■ちょっとした疑問や質問はここに書いてね118■

このエントリーをはてなブックマークに追加
1ご冗談でしょう?名無しさん
前スレ
■ちょっとした疑問や質問はここに書いてね117■
http://science6.2ch.net/test/read.cgi/sci/1253316072/

質問する前に
教科書や参考書をよく読もう
http://www.google.com/ などの検索サイトを利用し、各自で調べること
宿題を聞くときは、どこまでやってみてどこが分からないのかを書くこと。
丸投げはダメだからね
(丸投げ君は完全無視。答えるだけ無駄。)

質問に対する返答には、何かしらの返答ちょうだいね
★書き込む前に>>2の注意事項を読んでね
★数式の書き方(参考)はこちら>>3
(予備リンク:>>2-10
荒らし厳禁、煽りは黙殺、忘れないうちに定期age
単発質問スレを発見したらこのスレッドへの誘導をよろしくね。

定番FAQ
英語最新版
http://math.ucr.edu/home/baez/physics/
旧版日本語訳
http://research.kek.jp/people/morita/phys-faq/
他にも参考にあるサイトなどあればぜひ。
例)http://ja.wikipedia.org/
※wikipedia内の説明はすべてが正確なわけではありません。
このスレでの受け答えもそうですが。相互に補完しつつ精度を高めましょう。
2ご冗談でしょう?名無しさん:2009/10/07(水) 20:00:27 ID:??? BE:255611693-DIA(346366)
書き込む際の注意

1.)
板の性格上、UNIX端末からの閲覧も多いと推察されます。
機種依存文字(ローマ数字、丸数字等)は避けて下さい。

2.)
以下のような質問に物理板住人は飽き飽きしているので、たぶん無視されます。
しないで下さい。
「相対性理論は間違っています」「量子力学は間違っています」
「宇宙論は間違っています」「シュレディンガーの猫は変です」
「永久機関を作りました」「タイムマシンについて教えて」
「どうして〜?」関連(→「どのようにして〜?」と質問すること)
「なぜ〜?」関連(たいてい、物理学の対象ではないため)

「どうして・なぜ」:http://academy6.2ch.net/philo/
(哲学板・雑談板のほうがむいている場合が多いです。)
雑談は雑談スレで:http://science6.2ch.net/test/read.cgi/sci/1181386663/

3.)
宿題を聞くときは、どこまでやってみてどこが分からないのかを書くこと。
丸投げはダメよ。丸投げに答えるのもダメよ。 丸投げを相手にする人はお馬鹿さん。
せめてポインタを示す程度に留めましょう。

4.)
厨房を放置できない奴も厨房
3ご冗談でしょう?名無しさん:2009/10/07(水) 20:00:42 ID:??? BE:605894988-DIA(346366)
数式の書き方(参考)
●スカラー:a,b,...,z, A,...,Z, α,β,...,ω, Α,Β,...,Ω,...(「ぎりしゃ」「あるふぁ〜おめが」で変換)
●ベクトル:V=[v1,v2,...], |V>,V↑,vector(V) (混同しないならスカラーの記号でいい。通常は縦ベクトル)
●テンソル:T^[i,j,k...]_[p,q,r,...], T[i,j,k,...;p,q,r,...]  (上下付き1成分表示)
●行列  M[i,j], I[i,j]=δ_[i,j]  M=[[M[1,1],M[2,1],...],[M[1,2],M[2,2],...],...], I=[[1,0,0,...],[0,1,0,...],...]
(右は全成分表示。行または列ごとに表示する。例:M=[[1,-1],[3,2]])
●転置行列・随伴行列:M ',tM, M†("†"は「きごう」で変換可) ●行列式・トレース:|A|=det(A), tr(A)
●複号:a±b("±"は「きごう」で変換可)
●内積・外積・3重積:a・b, a×b, a・(b×c)=(a×b)・c=det([a,b,c]), a×(b×c)
●関数・数列:f(x), f[x] a(n), a[n], a_n
●平方根:√(a+b)=(a+b)^(1/2)=sqrt(a+b) ("√"は「るーと」で変換可)
●指数関数・対数関数:exp(x+y)=e^(x+y) ln(x/2)=log[e](x/2)(exp(x)はeのx乗、lnは自然対数)
●三角比:sin(a), cos(x+y), tan(x/2)
●絶対値:|x|  ●共役複素数:z~ ●ガウス記号:[x] (関数の変数表示と混同しないよう注意)
●階乗:n!=n*(n-1)*(n-2)*...*2*1, n!!=n*(n-2)*(n-4)*...
●順列・組合せ:P[n,k]=nPk, C[n,k]=nCk, Π[n,k]=nΠk, H[n,k]=nHk ("Π"は「ぱい」で変換可)
4ご冗談でしょう?名無しさん:2009/10/07(水) 20:01:11 ID:??? BE:426020459-DIA(346366)
数式の書き方続き(参考)
●微分・偏微分:dy/dx=y', ∂y/∂x=y_x ("∂"は「きごう」で変換可)
●ベクトル微分:∇f=grad(f), ∇・A=div(A),∇xA=rot(A), (∇^2)f=Δf ("∇"は「きごう」,"Δ"は「でるた」で変換可.)
●積分:∫[0,1]f(x)dx=F(x)|_[x=0,1], ∫[y=0,x]f(x,y)dy, ∬[D]f(x,y)dxdy, ∬[C]f(r)dl ("∫"は「いんてぐらる」,"∬"は「きご

う」で変換可)
●数列和・数列積:Σ[k=1,n]a(k), Π[k=1,n]a(k) ("Σ"は「しぐま」,"Π"は「ぱい」で変換可)
●極限:lim[x→∞]f(x) ("∞"は「むげんだい」で変換可)
●図形:"△"は「さんかく」 "∠"は「かく」 "⊥"は「すいちょく」 "≡"は「ごうどう」 "∽"は「きごう」
●論理・集合:"⇔⇒∀∃∧∨¬∈∋⊆⊇⊂⊃∪∩"は「きごう」で変換
●等号・不等号:"≠≒<>≦≧≪≫"は「きごう」で変換
5ご冗談でしょう?名無しさん:2009/10/07(水) 20:03:24 ID:???
989 名前:ご冗談でしょう?名無しさん 投稿日:2009/10/07(水) 19:47:00 ID:???
やっぱり分かってない奴が分かったふりして、適当に日本語でおkとかメコスジって言ってるんだな・・・
6ご冗談でしょう?名無しさん:2009/10/07(水) 20:15:39 ID:???
1000 名前:ご冗談でしょう?名無しさん 投稿日:2009/10/07(水) 20:00:28 ID:???
受け売りとWikipediaでよくこれだけ書けるわ。面の皮の厚さには感心する。
つか、これだけしか書けまいがな。

↑基本すら分かってない奴が何偉そうに書いてんだこいつ
死ねよ
分かってない奴に受け売りとWikipediaとか書かれる筋合いねーわ
7ご冗談でしょう?名無しさん:2009/10/07(水) 20:27:10 ID:???
■ちょっとした疑悶や膣悶はめこすじに掻いてね69■
8ご冗談でしょう?名無しさん:2009/10/07(水) 20:50:37 ID:???
>>前スレ972
「波束」の意味が不明と言ってるんじゃなくて、「*1個の光子の*波束」が意味不明だと言ってるのに、
波束の意味を延々と解説されても「ご苦労さま。で、それが何か?」としか言いようがないんだが。

まぁおまいの勘違いの手がかりが得られたのでよしとしよう。それは
>これは、単色光に近い状態(=干渉性の高い状態)。
この部分。単色光に近いことと干渉性の高いことはイコールではない。
やたら「波長が同じ」という仮定を強調するのは何でかなと疑問だったけど、
波長がそろっていることとコヒーレントがイコールだと勘違いしてる、という
ことなら話はわかる。

たとえばナトリウムランプは単色光に近いけど(正確には二成分だけど)、
全然コヒーレントじゃない。だから、いくら携帯の波長が同じと仮定したとしても
それだけでは干渉性が高いことにはならない。

ちなみに、ナトリウムランプ中のナトリウムの圧力は0.1Paだそうで、そこから
ナトリウムの平均距離をざっと見積もると、偶然ながらナトリウムランプの波長と
同程度になる。ちょうど、携帯の波長と同程度離した携帯の話と同じ状況に
なっている。だからおまいの主張どおりならナトリウムランプは集団運動とやらで
1かたまりの発信源としてふるまい、コヒーレントにならなきゃいけないはずだが
そうなってはいない。つまり前スレ>>925の論は成り立っていない
9ご冗談でしょう?名無しさん:2009/10/07(水) 20:57:18 ID:???
>>6
スレも新しくなったのに必死だなww

というか、おまえ自分で受け売りとwikiって書いてるんだぞ?
そこは理解しとかないとかなり恥ずかしいぞ
10ご冗談でしょう?名無しさん:2009/10/07(水) 21:01:11 ID:???
>>9
スレ立って早々に荒らしに触るな、バカ者
11ご冗談でしょう?名無しさん:2009/10/07(水) 21:17:12 ID:???
>>8
ナトリウム原子は不規則に分布し、かつ動き回るが
2つの携帯はほぼ静止している。話は同じではないと思うが。

レーザーのような場合は、BE統計が効くから例外だが、
通常は、干渉を起こすのは、個々の光子レベルの話だよ。
たとえば、ヤングの実験とかで、一秒に1個ずつしか
スリットを通らないようにしても、長時間露光して写真をとったり
すれば、スクリーン(フィルム)に干渉模様はできる。

ナトリウム灯を用いても、ヤングの実験で干渉模様はできるよ。
あらかじめ、単スリットを通して、その後に2つ穴のスリットを
通せばね。

単スリットを通せば、光路差をそろえることができるからだが、
これは今の場合、携帯が静止しているから不要。

ナトリウム灯でヤングの実験をするとして
Na原子気体を思いっきり高温にしてやると
ドップラーシフトで波長が広がるから、干渉模様はぼやける。
これは、携帯の場合の話で、変調により周波数が拡がりをもつことに
相当する。
12ご冗談でしょう?名無しさん:2009/10/07(水) 21:24:54 ID:bgYwvAvH
紙を、まるめて、音声を流すとき、音の波は、紙でどう変わるのでしょうか?
13ご冗談でしょう?名無しさん:2009/10/07(水) 21:42:35 ID:???
>>11
>ナトリウム原子は不規則に分布し、かつ動き回るが
影響があるほど動き回るのかよ。動くのが嫌ならLEDでもいいぞ。
半導体レーザーとして作られたものは別だけど、普通のLEDの光は
コヒーレントじゃない。固体だから、発光源の間隔はナトリウムランプより
ずっと近い。おまいの主張どおりならLEDはより集団運動とやらをしやすいはず。

>通常は、干渉を起こすのは、個々の光子レベルの話だよ。
誰かそれを否定したか?で、1個の光子の波束って何?

>あらかじめ、単スリットを通して、その後に2つ穴のスリットを
>通せばね。
後出しジャンケンで条件を付け加えられても困るのだが。

>単スリットを通せば、光路差をそろえることができるからだが、
やっぱわかってない。単スリットを通すとコヒーレントになるのは
光路差がそろうからではなく位相がそろうから。光路差がそろったって
位相がバラバラではコヒーレントにはならない。携帯が静止したって
位相までそろうわけじゃないから、2台の携帯の電波はコヒーレントではない
14ご冗談でしょう?名無しさん:2009/10/07(水) 22:11:14 ID:???
>>13
君が何を主張しているのかがやっと見えてきたよ。
光子の話は本筋ではないようだから、古典電磁気でいくね。
君の言いたいのは、思いっきり簡単にすると、こういうことでしょ。

ある場所に
携帯1から届く電磁波による電場を f1(t) = E1 sin(ωt)
携帯2から届く電磁波による電場を f2(t) = E2 sin(ωt+φ)
とする。
位相差φはランダムだから、合成電場のエネルギー|f1+f2|^2の
時間平均から干渉項は落ちる。

でもね。いまは、位相差φがランダムという仮定は成り立たないでしょ。
だから、干渉項が残る。極端な話ω≒1Hzの場合を考えてみて。
1秒に1回の振動だ。2つの波源の位相がどうであれ、しばらくの間、
同期して振動するでしょ。

実際は変調により携帯によってωは微妙に違い、時間変化もする。
だから、干渉で強め合う場所や弱めあう場所は一定ではなく、
空間的・時間的に移動する(うなる)が、ωにくらべてその移動は
ゆっくりだ。

このうなりの移動まで長時間平均してしまえば、もちろん、
干渉はならされて消えるよ。

1個の光子の波束については、
有限の広がりをもった電磁場の1粒子状態という意味だ。
これは平面波状態の重ねあわせで書ける。
15ご冗談でしょう?名無しさん:2009/10/07(水) 22:29:49 ID:???
>>14
携帯電話の電波ならωはGHz位だと思うが、
> 空間的・時間的に移動する(うなる)が、ωにくらべてその移動は
> ゆっくりだ。
> このうなりの移動まで長時間平均してしまえば、
ここの「ゆっくり」や「長時間」はどれくらいの時間スケール?
16ご冗談でしょう?名無しさん:2009/10/07(水) 22:32:29 ID:???
>極端な話ω≒1Hzの場合を考えてみて。
だから後出しジャンケンで条件を加えられても困るのだが。
2台の普通の携帯の電波がコヒーレントかという話のはずでは?

>2つの波源の位相がどうであれ、しばらくの間、
>同期して振動するでしょ。
そのことをさしてコヒーレントとはふつー言わないが、おまいがそのことをさして
コヒーレントと称しているらしいことは理解した。

きちんと定義された用語はその意味で使ってくれないと話が通じるわけがない
17ご冗談でしょう?名無しさん:2009/10/07(水) 22:43:23 ID:???
だんだん >>16が見苦しくなってきた。
18ご冗談でしょう?名無しさん:2009/10/07(水) 22:46:12 ID:???
そろそろ質問スレに戻そうぜ
19ご冗談でしょう?名無しさん:2009/10/07(水) 22:47:35 ID:???
どう見ても見苦しいのは条件をコロコロ変えて何とか間違ってないことにしようと
取り繕っている側だろう
20ご冗談でしょう?名無しさん:2009/10/07(水) 23:03:24 ID:???
>2台の普通の携帯の電波がコヒーレントかという話のはずでは?
え?それどうでもよくね?
21ご冗談でしょう?名無しさん:2009/10/07(水) 23:23:19 ID:???
>>16, 19
昨晩書いたことだけど、順番に考えてみてくれ。
ある場所の電磁場のエネルギー密度は>>14の|f1+f2|^2ね。

ここで、ω=0の場合を考えると、静電場だから
単純にクーロンの法則で重ねあわせてよい。つまり、干渉がある。

つぎに、ω=0.001 Hz の場合を考えると、ほとんど静電場だから
やはり、干渉がある。

つぎに、ω=0.01 Hzの場合を考えると、静電場に近いから、
やはり干渉があるだろう。こうやってωを大きくしていく。

もちろん、ωが大きくなるにつれ、電場の振幅E1, E2は距離の2乗に
反比例する形から、徐々に、距離の1乗に反比例する形に変わっていくよ。

そうやって、ωを大きくしていくとき、どこかで、人間がωを一定に保つ
のが難しいところがやってくる。原子が出す光(可視光)とかは、
共鳴空洞などで工夫しないなら、一定に保てないと言えるだろう。
ここまでくると、2つの波源の初期位相はバラバラで、干渉は消える。
君の言い方をするなら、アンテナから出る各光子の位相は
まったくそろってない(コヒーレントでない)。

でもね、携帯の電磁波ではω〜1Gzで、そこまでいってないんだよ。
ωを人間の技術で1%以内に保つことは可能。だから、コヒーレントな
光です。干渉します。変調していても、100振動くらいの間は位相が
揃っていると考えていい。
22ご冗談でしょう?名無しさん:2009/10/07(水) 23:36:06 ID:???
>>21
つまり1マイクロ秒くらいの「長時間」積分すれば干渉はならされて消えると考えてよろしいか?
23ご冗談でしょう?名無しさん:2009/10/07(水) 23:38:59 ID:???
>>21
あなたは、コヒーレントという言葉の意味がまるっきり、わかってない。

ω=0.001Hzであっても、「今山の奴」「今谷の奴」「今山と谷の中間の奴」
「その他大勢」がいろいろあって、いろんな可能性があったら、干渉が見える
はずがない。

あなたは頭の中で勝手にω=0.001Hzなら、みんなが山の状態から始まると
決めつけてしまっているが、自分でもその勝手な思いこみに気付いてない。
24ご冗談でしょう?名無しさん:2009/10/07(水) 23:41:27 ID:???
>>20
もともとの質問がどうでもいいと。そりゃすまんかったな
25ご冗談でしょう?名無しさん:2009/10/07(水) 23:43:03 ID:???
>>21
>ωを人間の技術で1%以内に保つことは可能。だから、コヒーレントな光です。
周波数がそろってるだけじゃコヒーレントじゃねぇよ。何度も言わすな
26ご冗談でしょう?名無しさん:2009/10/07(水) 23:44:32 ID:???
>>22
変調の程度によるけど、うなりの周期の10倍の時間として
振動数のゆらぎが1%くらいなら100マイクロ秒、
0.1%なら10マイクロ秒くらいの「長時間平均」で
干渉項は消えると思います。
27ご冗談でしょう?名無しさん:2009/10/07(水) 23:45:26 ID:???
ωを一定に保つだけでコヒーレントが達成されるなら苦労はしない
28ご冗談でしょう?名無しさん:2009/10/07(水) 23:50:24 ID:???
>>26
振動数が揃っただけじゃダメだと、いったい何人の人に何回言われたらわかるの?
29ご冗談でしょう?名無しさん:2009/10/07(水) 23:57:39 ID:???
>>22>>26
相手が電子機器だということを忘れるなよ。
回路にとってマイクロ秒オーダは十分長い時間である可能性はある。
ペースメーカーがどうだかは知らんけどね。
30ご冗談でしょう?名無しさん:2009/10/08(木) 00:01:05 ID:???
>>23
それが、可視光領域での経験から君の頭が固まってる、て言ってるの。
たしかに別の原子から出る自然放出光の位相は揃ってないよ。

でも、いま考えているのは、1GHzのアンテナ2個から出る電波
の干渉の計算だ。 これは古典電磁気で十分に計算できる問題。
ある瞬間にアンテナ1の電子たちが上へ寄っており、かつ、
アンテナ2の電子たちが上へ寄っているなら、出る電波の位相は揃うよ。

あと、アンテナからの電波の放射は、個々の電子から出るというより
アンテナ全体から出ると思った方がいい。

アンテナは数センチ、波長は40cmだから、アンテナ全体から出るといっても
事実上、点からの放射だけどね。
31ご冗談でしょう?名無しさん:2009/10/08(木) 00:01:56 ID:???
>>27
だから、君は可視光領域で仕事してるんでしょ?
光は波長によって全然、性質が違うんだよ。
32ご冗談でしょう?名無しさん:2009/10/08(木) 00:09:24 ID:???
>>30
アンテナ1とアンテナ2の電子はどうして一緒に上に登るの??
33ご冗談でしょう?名無しさん:2009/10/08(木) 00:10:37 ID:???
>>30
別々の携帯電話から出る電波の話だった筈だが、
なんでそれらのアンテナの電子が連動するのよ?
34ご冗談でしょう?名無しさん:2009/10/08(木) 00:13:42 ID:???
>>32
それぞれの携帯の発振器がアンテナの電子がゆさぶっているから。

これは可能な1つの状況。

アンテナ1の電子が上へ、アンテナ2の電子が下へ(位相がπずれた)
という状況もまた、別の可能な1つの状況。

いずれにしろ、位相差は一定時間(100〜1000振動の時間くらい)
は保たれる。これは変調の程度やωのコントロール精度によって違う。
35ご冗談でしょう?名無しさん:2009/10/08(木) 00:14:38 ID:???
>>33
同じ長さの振り子を天井からぶら下げて振ったら、
同じように揺れるだろ。
36ご冗談でしょう?名無しさん:2009/10/08(木) 00:15:25 ID:???
いや、時と場合による。>>35
37ご冗談でしょう?名無しさん:2009/10/08(木) 00:16:12 ID:???
>>29
そうかも知れないね。ちょっと気になるかな。
38ご冗談でしょう?名無しさん:2009/10/08(木) 00:17:03 ID:???
>>36
可視光レベルの振動数なら、おれもこんなことは言わないよ。
1 GHzだから、こう言ってるの。
3929:2009/10/08(木) 00:23:05 ID:???
>>24
問題になってるのは期待値じゃなくて最悪値のはず。
そこを忘れてないかい?

量子論的なコヒーレンシの概念は統計的な意味を包含している。
その意味でコヒーレントじゃないから
統計的な平均として強めあうことはない。
しかし、観測された1回の試行において、
強めっている状況に相当する電界強度が観測されることはありうるよ。
40ご冗談でしょう?名無しさん:2009/10/08(木) 00:31:37 ID:???
>>23
その点は理解していますよ。
時間座標の原点をずらせば、>>14で議論できます。
41ご冗談でしょう?名無しさん:2009/10/08(木) 00:34:23 ID:???
アンテナ1とアンテナ2の時間座標の原点を別に取るわけ?
42ご冗談でしょう?名無しさん:2009/10/08(木) 00:38:40 ID:???
>>前スレ967
分かりやすい解説ありがとうございます。電子は軽い粒子だと思っていたので
すが、それでもまだ重いほうだったのですね。てっきり真空では電子と陽電子
が出来たり消えたりしているものと解釈しておりました。
対生成で発生しうる他の粒子についてもう少し調べてみようと思います。
あとニュートリノに関する知識が皆無であることに思い至ったので、その辺の
勉強もしてみようと考えております。

皆様ありがとうございました。またいずれ知恵を借りに来ることがあるかもし
れませんが、その時は宜しくお願いします。
43ご冗談でしょう?名無しさん:2009/10/08(木) 00:39:23 ID:???
>>30
素人質問で恐縮だが……

波長40cmに対してアンテナ長数cmは無視できないほど大きく、
とても点には近似できないのでは?

それと、アンテナ長を短く見積もって3cmとし、電界が光速で伝わる
と仮定しても、電界がアンテナの端から端まで伝わるのに0.1n秒かかる。
1GHzの電波の振動周期1n秒の1割にも達するんだが、この影響は?
44ご冗談でしょう?名無しさん:2009/10/08(木) 01:03:07 ID:???
>>43
上へマイナスが行ったときには、下にプラスができていて、
重心はやはり真ん中にある。アンテナ長は波長の10%くらい
ですから、アンテナがうんと短くて、そのかわり両端に生じる
プラスとマイナスが多い場合(電気双極子放射)とほとんど同じです。

ずれの主要項は、電気4重極子放射ですが、波長程度の距離で
双極子放射の1000分の1くらい。(1/1000 = 0.10^3)
これは距離の3乗で落ちます。
遠方では、距離の1乗で落ちる電気双極子放射とくらべて、無視できます。
45ご冗談でしょう?名無しさん:2009/10/08(木) 01:04:46 ID:???
>>41
アンテナ1の初期位相がゼロになるように
時刻の原点を決めるだけだよ。
アンテナ2の初期位相φは、なにかの値になる。
46ご冗談でしょう?名無しさん:2009/10/08(木) 01:22:03 ID:???
>>44
スマソが、素人の俺にはまったく理解できないし、納得もできない。

より実態に即してアンテナ長を(1/4)λ(波長の1/4の長さ)とすると、
アンテナの下端の電界(の振動)がアンテナの上端に伝わるのに、
最低でも(1/4)T(振動周期の1/4)の時間がかかることになる。
すると、アンテナの上端の電子はアンテナの下端の電子よりも
どうがんばっても(1/2)πだけ位相が遅れてしまうのではないか?

重心が動いてようが止まっていようが、電磁波を発生させるのは
電子の運動なんだから全然関係が無いのではないか?

それと、アンテナ長(1/4)λの仮定に従えば、ズレの主要項は
1/256≒1.6%にもなり、コヒーレント性を論じるにあたってとても
無視できない大きさだと思うのだが。
47ご冗談でしょう?名無しさん:2009/10/08(木) 01:25:51 ID:???
>>46
>1/256≒1.6%
間違えた orz
正しくは

 1/64≒1.6%
48ご冗談でしょう?名無しさん:2009/10/08(木) 01:51:39 ID:???
>>46
振動数が同じ三角関数の重ね合わせは、
同じ振動数の三角関数だよね。位相がずれるだけで。

A1 sin(ωt+θ1) + A2 sin(ωt+θ2) + .... + A_N sin(ωt+θ_N)
= A sin(ωt + θ)

アンテナ内の場所による到達電場の位相のずれθ1, θ2, ... θ_Nは
時刻tによらないのだから、アンテナの大きさは気にしなくていい。
電子がアンテナ内で同じ周期で上下に振動していることの結果だけど。
49ご冗談でしょう?名無しさん:2009/10/08(木) 02:02:20 ID:???
>>43
アンテナ内で電子はそろって(同じ位相で)振動しているのだから、
回折格子みたいなものだと思えばいいのでは。

点で近似できるかどうかは本質的でない。
アンテナ1各部から出る電波の重ね合わせにより、遠方のある点の電場が
f(t) = E1 sin(ωt+θ)

のような形になることが言えれば十分。

アンテナ1各部というのは、じゅうぶんに多数の電子を含むが、
波長よりはじゅうぶんに短い部分を指す。
50ご冗談でしょう?名無しさん:2009/10/08(木) 02:10:07 ID:JIZ7T8ai
マイコンで障害物回避ロボットを作るに辺りキャタピラ型の車型ロボットをつくっています。
しかし、加速度センサだけで直進できるようにすること、障害物回避を行おうとしているのでタイヤ、モータの回転数などわからず
どう運動方程式をたてて方向修正すればいいのかわかりません。
よければ直進を目標としている際の軌道修正するための運動方程式をつくるのか教えていただけませんか、サイトなどでも結構です。
ちなみに制御はPID使います。
51ご冗談でしょう?名無しさん:2009/10/08(木) 02:11:25 ID:???
携帯からコヒーレント光が出てると聞いてやって来ました
52ご冗談でしょう?名無しさん:2009/10/08(木) 02:35:15 ID:???
>>48
やっぱり、素人の俺にはまったく理解できないし、納得もできない。

その式は、平面波や厳密な点光源のように一次元的に扱える場合にしか成り立たないのでは?
異なる位相の波が異なる場所((1/4)λは到底無視できない)から放射されているケースなので、
その式は無意味ではないか?そもそも波数ベクトルの項が無い。

波数ベクトルも考慮すれば、位置によってθの値はバラバラになってしまい(極端な例を挙げれば、
アンテナの上端の近辺と下端の近辺では、アンテナからの距離が同じでもθはまるで違う値だ)、
とうていコヒーレンスとは言えないのではないか?

>>49
電界の伝搬速度に上限がある以上、アンテナ内の電子が異なる位相で振動しているはずだが、
という素朴な疑問なのだが。回折格子に喩えると、場所毎に溝の大きさが違う回折格子の様なものでは?

以降は、前述の理由で納得できない。波長程度の距離では、位相の揃いようがないと思うが。
53ご冗談でしょう?名無しさん:2009/10/08(木) 03:15:51 ID:???
>>52
波数ベクトルk・x_iの項は、θ_iに含まれてる。

アンテナ内の場所iによる位相のずれ(発振器からの距離が違うなど)と、
アンテナ内の場所iから観測点までの距離による位相のずれk・x_iを
合わせたものがθ_iだと考えてくれればいい。

このθ_iは、場所iにはよるが、時刻tにはよらないでしょ。
だから>>48の三角関数の合成みたいな式で計算できて
ひとつの三角関数にまとまる。
(実際には積分計算。アンテナ内の電子の位相のずれの項のところが
少し違うが、よくある単スリットの場合の計算と同様。)

結局、はじめの問題は、長さ数センチのアンテナ2本からの1GHzの
電波の放射を古典電磁気学で計算してよいかどうか、という問題
なんだと思うが、おれは答がYesだと思うから、干渉がおきるという立場です。
(変調がある場合は、時間的・空間的に、うなる)

54ご冗談でしょう?名無しさん:2009/10/08(木) 03:26:55 ID:???
気の毒な奴だ
55ご冗談でしょう?名無しさん:2009/10/08(木) 03:33:26 ID:???
>>54
具体的にいってごらん。
56ご冗談でしょう?名無しさん:2009/10/08(木) 03:41:39 ID:???
レーザーとかやってる人は、横波の光子しかイメージできない
だろうけど、こういうアンテナからの双極子放射とかは、
アンテナ近傍では電子集団から出る縦波の光子(クーロン相互作用)が
効いていて、まず、準静電場的な電場ができて
それが変化して電磁誘導で磁場ができて、その繰り返しで遠方へ伝わっていく。
原子から出る可視光領域の光の場合とは、イメージがかなり違うと思う。
57ご冗談でしょう?名無しさん:2009/10/08(木) 04:14:37 ID:???
横波や縦波の光子ってなに?
58ご冗談でしょう?名無しさん:2009/10/08(木) 04:25:00 ID:???
運動量(進行方向)に偏光方向(電場の方向)が直交しているのが横波、
平行なのが縦波。

縦波は、光子として検出器にかかることはない。
電荷間の引力を仮想的に媒介するだけ。
59ご冗談でしょう?名無しさん:2009/10/08(木) 04:33:19 ID:cAISUX0V
ローレンツ変換についての質問です。
ベクトル、テンソルなどの場合、足のついてる数によってどのような変換か分かる
(例えばV^iなら、V^i=(L^i_j)V^jのように。)のですが
体積要素dVなどは、その系のエネルギーをEとして
EdV=E'dV'のように変換します。これはどのようなテンソルだと思えば良いのでしょうか?
EdVでスカラーなのでしょうか?
60ご冗談でしょう?名無しさん:2009/10/08(木) 04:33:45 ID:???
なるほど。
で、縦波や横波の光子ってなに?
61ご冗談でしょう?名無しさん:2009/10/08(木) 04:46:08 ID:???
>>59
ε_{μνλρ} p^μ dx^ν dx^λ dx^ρ なら擬スカラーのように見える
62ご冗談でしょう?名無しさん:2009/10/08(木) 04:52:58 ID:cAISUX0V
>>61
ありがとうございます。表記法の問題ということでしょうか?
ε_{μνλρ} p^μ dx^ν dx^λ dx^ρという書き方だと、
足は全部つぶれていますね。
運動量空間dP/E=dP'/E'も同じようにレビチビタテンソルを使って書けますでしょうか?
63ご冗談でしょう?名無しさん:2009/10/08(木) 06:05:14 ID:ZNFReSFy
すいません、物理の本について質問なんですがオークションでランダウの場の古典論と力学を買いました、東京図書で1966年刊行のやつです。これはいくらなんでも古いですか?刊行が新しいちくま書房とかのランダウとかの方がいいでしょうか?
内容は変わるんでしょうか?
64ご冗談でしょう?名無しさん:2009/10/08(木) 07:30:13 ID:???
>>50

マルチ氏ねよ
65ご冗談でしょう?名無しさん:2009/10/08(木) 08:25:06 ID:???
溶液の温度があがると電気伝導度が大きくなる理由を教えて下さい。
66ご冗談でしょう?名無しさん:2009/10/08(木) 08:35:47 ID:???
>>57
> 横波や縦波の光子ってなに?

横波は普通だが、縦波は一時期話題になった「スカラー電磁波」じゃないの?
要するに、と系
67ご冗談でしょう?名無しさん:2009/10/08(木) 10:21:11 ID:???
>>62 線形ではないのでテンソルを用いるのには難がある。でも
ローレンツ不変性は以下より明白。
まずd^4p=dEdpxdpydpzがローレンツ不変なのはいいと思う。
で、E^2-p^2=m^2がローレンツ不変でありかつ両辺の微分を
とればEdEd^3p=(EdE-pdp)d^3p=mdmd^3pなのでd^4p=mdm×(dp/E)。
で、我々の考える状況ではdm=0なのでdp/Eがローレンツ不変になる。
>>61が紹介したやつは場のもつエネルギー運動量の流れを微分形式で
書いたもの、と解釈できるので、これは3つの脚ε_{μνλρ} p^μ
をもった3階のテンソル。
68ご冗談でしょう?名無しさん:2009/10/08(木) 10:33:47 ID:???
>>66
やっぱりそっち系か。物理そっちのけで狂信的に干渉を主張するあたり何かおかしいと感じてた。
69ご冗談でしょう?名無しさん:2009/10/08(木) 10:59:19 ID:???
>>68
話の流れは分からんが、ちゃんとあるよ。
ゲージ粒子 縦波
とかで検索すればいい。
70ご冗談でしょう?名無しさん:2009/10/08(木) 11:31:36 ID:???
完全に自由空間での光子は横波成分しかないけど、放射源に近いところでは縦波成分は出てくる。
のでアンテナ直近の話で縦波云々自体はと系ではない。そんな縦波成分残りまくりの領域で
コヒーレンスだと言い張るのがと系。そもそもそんな領域でコヒーレンシーがうまく定義できるのか?
71ご冗談でしょう?名無しさん:2009/10/08(木) 12:42:29 ID:010Xb7gd
>>67
詳しくありがとうございます。
教えていただいたdp/Eがローレンツ不変量になる証明方法は知っています。
幾つかの本にも同じような方法が書いてあったのですが、
どうしてもテンソル表記でないことに違和感を感じてしまいます。
全ての物理量はテンソル、ベクトル、スカラーのいずれかで書けるのではなかったのですか?
線形ではない、というのは体積要素が微小量の3乗になっている、ということですよね。
微分形式で書けば、線形でないものでも
座標変換に対する変換性があらわに示されるのですか?
(微分形式はよく分かりません。すみません。)
72ご冗談でしょう?名無しさん:2009/10/08(木) 13:20:23 ID:???
電磁波に縦波があるのは文句はないが、縦波のあるなしとコヒーレンスは
どう関係するの???
縦波だろうが横波だろうが、コヒーレントなのはコヒーレントだし、
コヒーレントじゃないものはコヒーレントじゃない、と思われるのだが。
>>70
73ご冗談でしょう?名無しさん:2009/10/08(木) 14:08:21 ID:???
もちろん、一般の波の場合は縦波だからインコヒーレントとはいえない。
誤解させてすまない。

あくまで電磁場の場合に、縦波成分が残っているような領域では
そもそも波長や位相自体がうまく定義できないだろうに、
コヒーレンシー云々を議論できるのだろうか、という話。
74ご冗談でしょう?名無しさん:2009/10/08(木) 14:10:58 ID:???
>>74
d^4p δ(p^2-m^2) のように書けばローレンツ不変性はまあ明白
7574:2009/10/08(木) 14:12:20 ID:???
× >>74
>>71
76ご冗談でしょう?名無しさん:2009/10/08(木) 14:21:36 ID:???
>>70
> 放射源に近いところでは縦波成分は出てくる。

はいはい、ソースソース
論文参照でもいいから
77ご冗談でしょう?名無しさん:2009/10/08(木) 15:56:40 ID:???
78ご冗談でしょう?名無しさん:2009/10/08(木) 16:07:34 ID:???
>>77
「光子の質量がゼロでないなら」まで読んでやめた
79ご冗談でしょう?名無しさん:2009/10/08(木) 16:32:42 ID:???
80ご冗談でしょう?名無しさん:2009/10/08(木) 16:34:25 ID:???
>>78
確かに自分でぐぐってみたら??な情報ばかりだった。すまない。
でも、「光子の質量がゼロでないなら」というのはよいキーワード。

アンテナ近くの厳密な話は手に余るのでもっと簡単な例をあげると、
例えば静電場はE≠0だけどp=0。したがって形式的にはm^2=E^2-p^2≠0で
質量を持っているかのように記述できる。こういう仮想光子が質量を持つかの
ように振る舞う(オフシェルとかいろいろな言い方はあるけど)、というのは
どんな場の理論の教科書にも載ってるような話(のはず)

真空中でならもちろん電磁場は必ず横波になるけど、アンテナ近くで
真空中とはみなし得ない領域ではその限りではない

逆に真空中に限らず電磁場は横波という証明があるのなら見せて欲しい
81ご冗談でしょう?名無しさん:2009/10/08(木) 16:35:54 ID:???
白装束のいうスカラー電磁波とかの話とは全く無関係です。念のため
あれは真性の「と」
82ご冗談でしょう?名無しさん:2009/10/08(木) 20:00:02 ID:???
日本語でおk
83ご冗談でしょう?名無しさん:2009/10/08(木) 21:13:22 ID:KM+o/Gz/
マックスウェルの方程式の解はどういう式で表されますか?
y=○○のような形になりますか?
微分連立方程式を解くということですか?
独立変数と従属変数が複数あるのですよね?
x=○ y=○ みたいな。
84ご冗談でしょう?名無しさん:2009/10/08(木) 21:15:58 ID:???
え?
85ご冗談でしょう?名無しさん:2009/10/08(木) 21:16:53 ID:???
とんでもないのが来たな…
86ご冗談でしょう?名無しさん:2009/10/08(木) 21:19:09 ID:KM+o/Gz/
違うんですか・・すいません。高校物理に微分方程式がちょっと理解できる
ぐらいな者でして。どなたかマックスウェル方程式の解とは何かを
教えていただけませんか。
87ご冗談でしょう?名無しさん:2009/10/08(木) 21:28:41 ID:???
>>86
まず電磁気の教科書でも読んでマックスウェル方程式とは何かを勉強してくれ
88ご冗談でしょう?名無しさん:2009/10/08(木) 21:36:14 ID:KM+o/Gz/
>>87
ガウスの法則 電場 電荷の存在 発散
ガウスの法則 磁場 磁荷は存在しない 
アンペールの法則 電流の回りに磁界ができる
電磁誘導 電場の変化で電流が流れる
じゃなかったですか。式は省略で。
89ご冗談でしょう?名無しさん:2009/10/08(木) 21:37:38 ID:KM+o/Gz/
電場の変化で→磁場
90ご冗談でしょう?名無しさん:2009/10/08(木) 22:38:37 ID:???
ベクトルポテンシャルというものがいまいちよくわかりません

式で見ると何となくわかったような気はするんですが、物理的にはどういった意味を持つ量なんでしょうか、結局のところ
91ご冗談でしょう?名無しさん:2009/10/08(木) 22:39:55 ID:???
つまりポテンシャルだよ
92ご冗談でしょう?名無しさん:2009/10/08(木) 22:41:45 ID:???
流体力学勉強してこい
93ご冗談でしょう?名無しさん:2009/10/08(木) 22:46:48 ID:???
>>91
ベクトルポテンシャルとは空間そのものが持っているポテンシャルで、そのポテンシャルの移動が電磁場である、
みたいな感覚で理解してるんですが、この理解で問題ないんでしょうか
94ご冗談でしょう?名無しさん:2009/10/08(木) 23:50:01 ID:???
量子力学におけるハミルトン演算子の極座標表示ですが、一般に二種類で表されることがありますよね。
初めの項だけ書きますが、二次元の場合だと、Δu=∂^2u/∂r^2+・・・という表記とΔu=(1/r)(∂/∂r)(r(∂u/∂r))+・・・
という二通りです。
後者を展開すれば前者になるのは分かるのですが、後者で表記するメリットは何でしょう?
デカルトから極座標に変換すると直接後者にはならず、前者を出してから後者にまとめると言う作業が必要ですよね?
95ご冗談でしょう?名無しさん:2009/10/09(金) 00:02:41 ID:???
>>94
計算手順によっては2つめの方で出てくるよ
96ご冗談でしょう?名無しさん:2009/10/09(金) 00:19:26 ID:???
>>95
私がやってる方法は∂/∂x(∂u/∂x)を偏微分の連鎖律で開いて全てr,θ,φの偏微分で表記する方法なのですが、
これでは前者になりますよね。
後者を直接出す方法を教えてもらえますか?
9796:2009/10/09(金) 02:17:57 ID:???
>>95
すいません。自己解決しました。
微分を展開して打ち消すんじゃなくて、微分の中に入れて打ち消すのか・・・
9897:2009/10/09(金) 02:22:44 ID:???
>>95
あ、スミマセン。自己解決してませんでした・・・
99ご冗談でしょう?名無しさん:2009/10/09(金) 07:12:44 ID:jSwwrDpV
昨日マックスウェル方程式について質問したものですが
微分方程式を解くというのは微分を含まない式に変形するという
ことですよね。加速度から式を変形して時間を代入、距離を
導き出す。マックスウェル方程式はどうやって微分を
取り除くのですか?熱伝導方程式でも波動方程式でもなんでも
それをするでしょ。
100ご冗談でしょう?名無しさん:2009/10/09(金) 07:43:47 ID:AaX80PsI
ベクトルポテンシャルとは空間ポテンシャル、重力、場(場の理論で参照)とか、あとは、参考書読め。
101ご冗談でしょう?名無しさん:2009/10/09(金) 08:20:42 ID:AaX80PsI
過去スレの少しでたぞ。先生に謝っとけ
国産ウラン、現実味 低コストで海水から採取 原子力機構
http://sankei.jp.msn.com/economy/business/090628/biz0906281953004-n1.htm
海水の中からウランを採る
http://www.ies.or.jp/japanese/mini/mini_hyakka/29/mini29.html
海水からウランやバナジウムなどの有用希少金属を捕集できる技術を開発
http://www.jaea.go.jp/jaeri/jpn/publish/01/ff/ff39/tech02.html
海水からウランを回収する技術を開発しています
http://www.jaea.go.jp/jaeri/jpn/publish/01/ff/ff43/topics.html
モール状捕集システムによる海水ウラン捕集のコスト試算
http://wwwsoc.nii.ac.jp/aesj/publication/TAESJ2006/No.4/5_4_358-363.pdf
102ご冗談でしょう?名無しさん:2009/10/09(金) 08:38:35 ID:AaX80PsI
バカみたいだけど、ある茄子と、味噌で、味噌汁つくると、上級回復薬だ。食べ過ぎないようにね。
103ご冗談でしょう?名無しさん:2009/10/09(金) 09:04:22 ID:???
>>96-98
大学1,2年あたりだとちょっと荷が重いかも。

外微分とHodge作用素を使うと、ラプラシアンとかは座標系に依存しない形で書ける。
それを極座標系で書き下せばいい。
10465:2009/10/09(金) 10:48:28 ID:???
>>65をお願いします。

m(_ _)m
105ご冗談でしょう?名無しさん:2009/10/09(金) 11:36:45 ID:???
>>104
化学には詳しくないけど、物理の視点から言えば溶液が従う熱力学関数さえ分かれば線形応答で原理的には分かるはず
要するに、その溶液の熱力学的性質に大きく依存するんだけど

直観的にありうる描像としては温度が上がるとイオンが解離してキャリアが増えるとか考えられるが、
それだと溶液一般、特にNaClみたいにほぼ解離してる溶液は説明つかんね
106ご冗談でしょう?名無しさん:2009/10/09(金) 11:44:39 ID:???
温度が上がれば電子やイオンの移動度も上がるべ!?
107ご冗談でしょう?名無しさん:2009/10/09(金) 13:50:38 ID:PLnFCb1g
おまいら、大学3年生は、そろそろ就職活動しろよ。ちょっと遅いが。
108ご冗談でしょう?名無しさん:2009/10/09(金) 13:53:08 ID:???
■ちょっとした疑悶や膣悶はめこすじに掻いてね69■
109ご冗談でしょう?名無しさん:2009/10/09(金) 13:53:44 ID:???
おまえもちゃんとハロワ行けよ
110ご冗談でしょう?名無しさん:2009/10/09(金) 17:25:33 ID:???
>>98
円柱座標とか極座標とか具体的な場合について計算したいのなら
∂/∂r = (∂x/∂r)∂/∂x + (∂y/∂r)∂/∂y+ (∂z/∂r)∂/∂z
みたいのを計算して∂/∂xについて解いて、(∂/∂x)(∂/∂x)を計算すれば分かる
もっと一般的な議論がしたいなら>>103の通り。
111ご冗談でしょう?名無しさん:2009/10/09(金) 17:38:34 ID:???
>>106
本当?
金属なんか液化したってやっぱり温度上がると易動度下がりそうな気がするけど
112ご冗談でしょう?名無しさん:2009/10/09(金) 17:54:28 ID:oiaCCAKn
>>111
金属では電流に担い手は電子で、フォノンの振動により散乱されるから
温度が上がると電気伝導度は下がる

一方、溶液では電流の担い手はイオンで、金属イオンは水分子よりも重いから
水分子の振動による散乱は少ないんじゃないかな
113ご冗談でしょう?名無しさん:2009/10/09(金) 18:18:21 ID:???
俺なんか単純に解離度の問題だなと思ってたが、>>105読んでそうなのか?と思ったり、
これ以上は、各種イオン溶液の実験データをもって話をしないと定性的な議論じゃラチあかんと思う。
114ご冗談でしょう?名無しさん:2009/10/09(金) 18:21:46 ID:???
そもそも、どんな溶液でも温度が上がると必ず電気伝導度が大きくなるの?
115ご冗談でしょう?名無しさん:2009/10/09(金) 18:48:04 ID:???
ガラスは液化すると良導体になるな。
116ご冗談でしょう?名無しさん:2009/10/09(金) 18:58:48 ID:PLnFCb1g
新型インフル、患者報告数51%増 北海道などで大幅増
 国立感染症研究所は9日、4日までの1週間に全国約5千カ所の
定点医療機関から報告された
インフルエンザ患者数は3万765人、1医療機関当たり6・40人となり、前週(2万365人)より
51%増加したと発表した。ほとんどが新型で、この1週間で約33万人が感染したとみられる。(産経新聞)
[記事全文]

・ <新型インフル>受診患者33万人…9月28日〜10月4日 - 毎日新聞(10月9日)
・ インフル患者、大幅増加=5道府県で警戒レベル超−感染研 - 時事通信(10月9日)
学校休校にしませんか。
gakkou
117ご冗談でしょう?名無しさん:2009/10/09(金) 19:27:41 ID:jSwwrDpV
マックスウェル方程式の
>>99
ですけどどなたか分かる方いませんでしょうか。
118ご冗談でしょう?名無しさん:2009/10/09(金) 19:45:47 ID:???
昔からの疑問だったんだけど、ニュートンリングに代表される
光の干渉は、コヒーレント光でなくても観測されるのは何故?
高校物理の取り扱いだと、位相が強め合うか弱めあうかで判断しているから、
光がコヒーレントでないと干渉が起きないと考えられてしまうのだけど・・・。
119ご冗談でしょう?名無しさん:2009/10/09(金) 19:59:21 ID:???
>>117
方法はいろいろあってそれぞれ利点と欠点があったりするから
場合によって使い分けられたりしている。

1つのよく知られた方法は、
(ローレンツゲージの)四元ベクトルポテンシャルというのを導入して
それについての方程式の形で書くと、
源のある波動方程式とだいたい同じ形になってますよというやつ。
120ご冗談でしょう?名無しさん:2009/10/09(金) 20:10:23 ID:???
>>117
積分する
121ご冗談でしょう?名無しさん:2009/10/09(金) 20:10:32 ID:ERmq/ktZ
レベル超とは、すさまじい。
122ご冗談でしょう?名無しさん:2009/10/09(金) 20:10:50 ID:???
>ニュートンリングに代表される光の干渉
あれ普通には単色光じゃないですよね。
雑多な波長が混ざってる昼色光ですよね。
そもそもで
位相・干渉で説明するのが乱暴なんじゃないですか?
123ご冗談でしょう?名無しさん:2009/10/09(金) 20:46:41 ID:???
>>117,99
あてずっぽうでも何でも条件を満たす関数が見つかりゃいい。
二次式を勘で因数分解したりするでしょ。それと同じ。
124ご冗談でしょう?名無しさん:2009/10/09(金) 20:50:48 ID:???
うはwwwwwwww夢がニュートンリングwwwwwwwwwwwwwww
125ご冗談でしょう?名無しさん:2009/10/09(金) 20:57:10 ID:???
>>118
波長がバラバラだから虹色のグラデーションになるし、自身の干渉だから
位相が揃ってんの当たり前だろ。
126ご冗談でしょう?名無しさん:2009/10/09(金) 20:59:49 ID:???
>>125
何を言っているんだ?おまえは
127ご冗談でしょう?名無しさん:2009/10/09(金) 21:24:33 ID:???
群速度v=∂ω/∂kを変形して、
v=∂(ω/hbar)/∂(k/hbar)=∂E/∂pを、ハミルトン方程式q'=∂H/∂p
と解釈することは可能ですか?(つまり、粒子の速度=群速度と解釈)
量子力学だとハミルトニアンは演算子で、エネルギーとは別物なので
EをHと読み替えた部分が怪しいかなと思うのですが。
128ご冗談でしょう?名無しさん:2009/10/09(金) 21:26:04 ID:???
>>126
なんで理解できないんだ?おまえは
129127:2009/10/09(金) 21:26:21 ID:???
間違えました。
ハミルトン方程式を使っているので、古典力学でした。
しかし、古典力学でも、ハミルトニアン≠エネルギーだと思うので
そこの部分が正しいかどうか教えてください。
130ご冗談でしょう?名無しさん:2009/10/09(金) 21:29:04 ID:???
>>128
日本語でおk
131ご冗談でしょう?名無しさん:2009/10/09(金) 21:32:22 ID:???
>>130
読解力が残念
132ご冗談でしょう?名無しさん:2009/10/09(金) 21:54:48 ID:???
>>127
なにこのアホw
133ご冗談でしょう?名無しさん:2009/10/09(金) 22:02:45 ID:???
>>125
波長がバラバラだから虹色のグラデーションになるし、
>波長がバラバラで虹色のグラデーション?

>自身の干渉だから 位相が揃ってんの当たり前だろ。
当たり前っていうからには証明しろよ
134ご冗談でしょう?名無しさん:2009/10/09(金) 22:07:06 ID:???
>>133
なにこのバカ?なんでこんな必死なの?
135ご冗談でしょう?名無しさん:2009/10/09(金) 22:09:08 ID:???
世の中、光の話題が一般的になって、割と科学の素養がある人でも
電波の事を知らない方が増えているのでしょうか?
136ご冗談でしょう?名無しさん:2009/10/09(金) 22:09:34 ID:???
>>134 シラネ
ニュートン環の現物見たことないんだろ
137ご冗談でしょう?名無しさん:2009/10/09(金) 22:12:17 ID:???
単なるプリズム、、、
138ご冗談でしょう?名無しさん:2009/10/09(金) 22:18:27 ID:???
プリズムは光の干渉を利用したものではないぞ
139ご冗談でしょう?名無しさん:2009/10/09(金) 22:24:58 ID:???
必死と言われましても・・・

>>125
波長がバラバラだから虹色のグラデーションになるし、
>波長がバラバラで虹色のグラデーション?

>自身の干渉だから 位相が揃ってんの当たり前だろ。
当たり前っていうからには証明しろよ

これにきちんと答えないとw
140ご冗談でしょう?名無しさん:2009/10/09(金) 22:28:21 ID:???
おいおい
適当に虹色のファンデーションとか書いたんじゃねーだろな?
141ご冗談でしょう?名無しさん:2009/10/09(金) 22:31:07 ID:???
で、群速度v=∂ω/∂kと書いた基地外はどこへ消えたんだ?
142ご冗談でしょう?名無しさん:2009/10/09(金) 22:32:20 ID:???
>>140
グラデーションだろwwwwwwwwwwwwwww
143ご冗談でしょう?名無しさん:2009/10/09(金) 22:36:15 ID:???
なに、おまいらニュートンリングの現物見たことないんじゃなかろうな?
144ご冗談でしょう?名無しさん:2009/10/09(金) 22:36:45 ID:???
グラデーションとかどうでもいいわ
145ご冗談でしょう?名無しさん:2009/10/09(金) 22:37:43 ID:???
うはwwwwwwww夢がニュートンリングwwwwwwwwwwwwwww
146ご冗談でしょう?名無しさん:2009/10/09(金) 22:39:49 ID:???
>>141
↓このへんにいるんじゃないか
http://en.wikipedia.org/wiki/Group_Velocity
147ご冗談でしょう?名無しさん:2009/10/09(金) 22:44:15 ID:???
アホな群速度書いてる奴とニュートンリング書いてる奴が同一事しか分からないんだがw
http://ja.wikipedia.org/wiki/%E7%BE%A4%E9%80%9F%E5%BA%A6
148ご冗談でしょう?名無しさん:2009/10/09(金) 22:49:46 ID:???
>>141は阿呆なの?
149ご冗談でしょう?名無しさん:2009/10/09(金) 23:03:15 ID:???
>>127
>群速度v=∂ω/∂kを変形して、
>v=∂(ω/hbar)/∂(k/hbar)=∂E/∂pを、ハミルトン方程式q'=∂H/∂p
>と解釈することは可能ですか?(つまり、粒子の速度=群速度と解釈)

これ書いた奴はとりあえず頭逝ってるわw
まず、群速度v=∂ω/∂kって何だ?
変形も糞もねぇ、最初から脳内
ハミルトニアンだあ?w
だからてめーは馬鹿なんだよ
脳腐ってるだろ
基地外死ねよ
150ご冗談でしょう?名無しさん:2009/10/09(金) 23:07:42 ID:???
>>139
池沼はほっといてやれよ
151ご冗談でしょう?名無しさん:2009/10/09(金) 23:29:53 ID:jSwwrDpV
>>119
四元ベクトルポテンシャルと波動方程式の導出、ゲージ変換から素粒子理論
への流れも少しはわかるんですがマックスウェル方程式の解の
全体像です知りたいのは。

>>120
四つの式に変数はいくつあるのか。ただ一つの式を積分するのとは
違うのかなあって思ってるんですが。
152ご冗談でしょう?名無しさん:2009/10/09(金) 23:34:09 ID:SC97PAXm
物理はそこにあるものにしか対処できない。だから嫌い。
女はそこにあるもの以外のものがある。だから好き。
君たちは視野がせまいのだよ、>>1-151を読んだけど。
なにを言いたいかというと、内容がないって言いたい。つまらない。だから失礼する。
153ご冗談でしょう?名無しさん:2009/10/09(金) 23:35:38 ID:???
>>151
> 四つの式に変数はいくつあるのか。
それが分からないならやはり>>87
154ご冗談でしょう?名無しさん:2009/10/09(金) 23:44:37 ID:jSwwrDpV
>>153
ttp://homepage2.nifty.com/eman/electromag/solution.html
こちらで言ってることはなんとなくわかるんですが
この掲示板でもっとわかりやすく教えてくれる人がいないとかと思いまして。
155ご冗談でしょう?名無しさん:2009/10/09(金) 23:46:58 ID:???
>>151
>流れも少しはわかるんですが
>>154
>こちらで言ってることはなんとなくわかるんですが
「〜はわかるんですが」とわざわざ見栄をはる香具師は実はわかってないの法則だな
156ご冗談でしょう?名無しさん:2009/10/10(土) 00:01:00 ID:???
>>149は日本語が不自由なの?>>127の日本語もやさしくないけど。
つうか
>まず、群速度v=∂ω/∂kって何だ?
>>149が低能であることの証明にしか見えないんだけれど、もしかして
日本語が不自由なだけかなあ、と思ったりもして。
157ご冗談でしょう?名無しさん:2009/10/10(土) 00:13:36 ID:???
>>156
群速度v=∂ω/∂kって何だ?
158ご冗談でしょう?名無しさん:2009/10/10(土) 00:23:35 ID:???
で、虹のグラデーションはどうなったんだ?
159127:2009/10/10(土) 01:27:17 ID:???
どなたか分かる方、回答お願いいたします。
160ご冗談でしょう?名無しさん:2009/10/10(土) 01:30:58 ID:???
>>156
日本語があまりよくないですか?
もう一度、質問をまとめてみます。

v=∂ω/∂kという式がありますが、これは群速度の定義式なのか、
何か別のものから導かれるものなのかが知りたいんです。
v=∂E/∂pと変形し、エネルギーをハミルトニアンと読み替えれば
ハミルトン方程式そのものと解釈することが可能ではないか?というのが自分の考えですが
これは正しいでしょうか?
161ご冗談でしょう?名無しさん:2009/10/10(土) 02:02:14 ID:???
>>160
>>146,147
162ご冗談でしょう?名無しさん:2009/10/10(土) 03:02:43 ID:???
>>160
>>127は普通に理解できるまともな日本語

>まず、群速度v=∂ω/∂kって何だ?
とあるように>>149が頭弱いだけ
163ご冗談でしょう?名無しさん:2009/10/10(土) 04:26:49 ID:eM6JQ6l6
わかりやすい参考書なかった?
164ご冗談でしょう?名無しさん:2009/10/10(土) 05:37:11 ID:???
>>160
>v=∂ω/∂kという式がありますが、これは群速度の定義式なのか、
>何か別のものから導かれるものなのかが知りたいんです。
そういう質問が出るということは群速度を分かってないんだと思う
一応それは群速度なる概念の定義式だが、その式は物理的には何を意味してるのか、波動の教科書で復習しろ

そうすれば、群速度なる描像が成立する領域では、半古典的にそれを粒子の運動方程式とみなせることが理解できると思う
165ご冗談でしょう?名無しさん:2009/10/10(土) 06:31:27 ID:eM6JQ6l6
超聖水
166ご冗談でしょう?名無しさん:2009/10/10(土) 09:00:55 ID:???
ビッグバンのエネルギーはどこから生まれたの?
無とか訳わかめ
167ご冗談でしょう?名無しさん:2009/10/10(土) 09:57:26 ID:JwQsVdiU
お釈迦様がいったではないか。
色即是空。とな。
168ご冗談でしょう?名無しさん:2009/10/10(土) 12:00:58 ID:69c0R8ec
この地球がある宇宙は、ループしているんだよ。マターがその証拠。発生の途中を観察すれば、変動値で理解できる。と、ある論文にかいてあったなぁ。
169ご冗談でしょう?名無しさん:2009/10/10(土) 12:56:46 ID:???
>>164
波動をすっかり忘れていたので復習しなおし、
群速度の定義は波束の移動から自然に出てくることは理解できました。

エネルギーをハミルトニアンと読み替える部分ですが、
量子力学だと、ハミルトニアンは演算子で、エネルギーは固有値なので別物ですよね?
古典力学の場合は同じだと思っても問題ないのでしょうか?
170ご冗談でしょう?名無しさん:2009/10/10(土) 13:11:47 ID:69c0R8ec
粒子で考えてみて
171ご冗談でしょう?名無しさん:2009/10/10(土) 13:25:18 ID:69c0R8ec
ハミルトニアン粒子なんちゃって。
172ご冗談でしょう?名無しさん:2009/10/10(土) 13:27:11 ID:69c0R8ec
ハミルトニアン粒子なんちゃって。
173ご冗談でしょう?名無しさん:2009/10/10(土) 13:31:08 ID:69c0R8ec
重複質
174ご冗談でしょう?名無しさん:2009/10/10(土) 13:34:24 ID:69c0R8ec
3つ重複させたほうが良かったか。
175ご冗談でしょう?名無しさん:2009/10/10(土) 13:34:34 ID:???
>>169
量子力学のハミルトニアン演算子に対応するオブザーバブルが全エネルギー。
古典力学のハミルトニアン(つまり全エネルギー)を量子化したのがそのハミルトニアン演算子。
176ご冗談でしょう?名無しさん:2009/10/10(土) 13:42:13 ID:69c0R8ec
アメリカのナッツが、364まで重複質させたから、まあ、既出だ。
177ご冗談でしょう?名無しさん:2009/10/10(土) 13:48:03 ID:???
>>175
それは分かってはいるんですが・・・。
178ご冗談でしょう?名無しさん:2009/10/10(土) 14:21:50 ID:69c0R8ec
ハードラックは、きゅうり、果物(個人差による)、マンゴー、ライチを、2、3日おきに食べること。服部栄養士学校、ビタミンの学問より。
179ご冗談でしょう?名無しさん:2009/10/10(土) 14:30:49 ID:69c0R8ec
京都大学は気づいたな。
180ご冗談でしょう?名無しさん:2009/10/10(土) 14:39:04 ID:69c0R8ec
まほろさんのOP知ってる?疑問がとけるよ。
181ご冗談でしょう?名無しさん:2009/10/10(土) 16:06:46 ID:???
正準方程式の粒子的描像から
波動的な見方へ行きたければ
ハミルトン・ヤコービ(Hamilton-Jacobi)の方程式でぐぐれば?
182ご冗談でしょう?名無しさん:2009/10/10(土) 16:12:55 ID:69c0R8ec
ハミルトン方程式ですか。
183ご冗談でしょう?名無しさん:2009/10/10(土) 16:17:19 ID:???
意味不明先生の破壊力は強力だな
争いごとも治まっちまうぜ
184ご冗談でしょう?名無しさん:2009/10/10(土) 16:34:57 ID:bWNOE607
320 :名も無き冒険者 [↓] :2009/10/10(土) 15:05:27 ID:HU7Lszd0
60km/hで走る戦車が、いきなり片方のキャタピラを逆回転させたらどうなるんだ?

物理スレ識者の意見をききたいです。よろしく。

大航海時代Online 戦闘系スレ 76海里
http://jfk.2ch.net/test/read.cgi/mmo/1253616416/350-
185ご冗談でしょう?名無しさん:2009/10/10(土) 16:45:46 ID:???
>>184
履帯と地面の摩擦による。要するに状態次第。
摩擦の小さい状況なら履帯を空転させながら徐々にカーブしつつ、戦車は回転。
摩擦の大きい状況なら超信地旋回しながら横転する可能性大。

この中間のどこか。
186ご冗談でしょう?名無しさん:2009/10/10(土) 17:02:48 ID:???
>>185
即レスありがとうございます。
187ご冗談でしょう?名無しさん:2009/10/10(土) 17:21:22 ID:???
現実的には「キャタピラが外れる」の可能性が大。
188ご冗談でしょう?名無しさん:2009/10/10(土) 17:54:50 ID:???
現実にはキャタピラ社製の履帯以外はキャタピラと呼べないわけだが
189ご冗談でしょう?名無しさん:2009/10/10(土) 18:06:17 ID:???
現実にはPS3だってファミコンと呼ばれる。
190ご冗談でしょう?名無しさん:2009/10/10(土) 18:07:39 ID:???
はいはい、ホッチキスもマジックテープも使うなよ。
191ご冗談でしょう?名無しさん:2009/10/10(土) 18:10:03 ID:???
>>189-190
質問に関係ないチャチャ入れに構うなよ。
192ご冗談でしょう?名無しさん:2009/10/10(土) 18:20:31 ID:???
>>169
> 量子力学だと、ハミルトニアンは演算子で、エネルギーは固有値なので別物
という言い方をするなら、古典力学でも、ハミルトニアンは関数で、エネルギーはその値なので別物
193ご冗談でしょう?名無しさん:2009/10/10(土) 18:34:49 ID:???
>>192
>ハミルトニアンは関数で、エネルギーはその値なので別物
そこの部分が不安だったのですが、
それでは、群速度の定義v=∂E/∂pのEを位置と運動量の関数とみなせば
q'=∂H(q,p)/∂pと考えても問題ないですか?
194ご冗談でしょう?名無しさん:2009/10/10(土) 18:44:40 ID:???
量子力学で古典的な粒子に対応するのは波束で、
波束の群速度が古典的な粒子の速度に対応する。
位置の期待値の時間発展は古典描像に一致するから
>>160は正しい
195ご冗談でしょう?名無しさん:2009/10/10(土) 19:01:43 ID:???
>>193
解析力学のHamiltonianは、エネルギーを一般化座標・一般化運動量で表したもの
だから、群速度の定義を正準方程式の片割れとみなすには、
偏微分がどの変数を固定してるのか注意しないと混乱して誤った結果を導いてしまう
たぶんこの場合は大丈夫だけど

あと、そもそも量子力学でdq/dtなんて物理量はないから、当然これは古典力学が成立するときだけ
これは波束の概念が使えるときに相当する
196ご冗談でしょう?名無しさん:2009/10/10(土) 19:37:16 ID:???
>>195
>量子力学でdq/dtなんて物理量はない
ハイゼンベルグ描像なら普通にありだが
197ご冗談でしょう?名無しさん:2009/10/10(土) 22:17:34 ID:???
>>194-195
ありがとうございました。
確かに、偏微分するときに固定してるほうの変数に注意が必要ですね。
198ご冗談でしょう?名無しさん:2009/10/10(土) 23:11:31 ID:p0aAz8Gw
大きいコマを回してみる
すると中心から外側にいくほどスピードが増していき、
ある場所で光速になる。
じゃ、それより外側どうなってるの?
199ご冗談でしょう?名無しさん:2009/10/10(土) 23:52:12 ID:???
>>198
ニュートン力学の剛体の話なら光速関係ない
200ご冗談でしょう?名無しさん:2009/10/11(日) 00:36:04 ID:???
>>198
激しく外出事項だからスルーすべきかもしれんが一応簡単に言っとくと、
どんな理想化したコマ(剛体という)であっても外側が光速に達する事は
出来ない。つまり角速度が一定に保てなくなる。剛体コマであってもアメ
のようにグニャっとなる。中心付近の回転の方が速くなってる訳だから、
コマはだんだん渦巻きパンみたいにトグロを巻いていくだろう。
201ご冗談でしょう?名無しさん:2009/10/11(日) 00:42:15 ID:???
>>200
グニャッとなるのは剛体じゃない
202ご冗談でしょう?名無しさん:2009/10/11(日) 00:50:15 ID:???
・相対論の下では剛体は存在できない
・相対論の下では剛体はグニャっとなる
203ご冗談でしょう?名無しさん:2009/10/11(日) 00:55:21 ID:???
>>202
存在しない物がグニャッとなるとか意味不明
204ご冗談でしょう?名無しさん:2009/10/11(日) 01:37:20 ID:???
現実にはそんな遠心力に耐える固体は作れないから、面倒なこと考えなくてもいいよ。
205ご冗談でしょう?名無しさん:2009/10/11(日) 02:29:20 ID:???
剛体があれば無限量の計算が一瞬で出来る。
206ご冗談でしょう?名無しさん:2009/10/11(日) 05:14:05 ID:QBPAWc1V
無限超えの桁があるのだから、無限超えにあるだろう。
207ご冗談でしょう?名無しさん:2009/10/11(日) 07:10:15 ID:???
無限超えって…
208ご冗談でしょう?名無しさん:2009/10/11(日) 08:28:16 ID:QBPAWc1V
無限超の桁って、自然事象をどう観察して出来たと思う?数学にはあるが、物理ではどうでしょう?
209ご冗談でしょう?名無しさん:2009/10/11(日) 08:29:09 ID:QBPAWc1V
皆で、京都賞受賞しましょう。
210ご冗談でしょう?名無しさん:2009/10/11(日) 08:40:51 ID:Te+/5aYz
おれは、ノーヘル賞でいいよ。
211ご冗談でしょう?名無しさん:2009/10/11(日) 09:48:00 ID:???
俺はignoreされてればいいよ
212ご冗談でしょう?名無しさん:2009/10/11(日) 09:51:41 ID:QBPAWc1V
>>211
光の限界速度かんがえればいいよ。
213ご冗談でしょう?名無しさん:2009/10/11(日) 16:54:28 ID:hzlWzM0R
http://373news.com/_nie/qa/071211.php

これおかしくね?www 気流の問題だと思うんだが。
214ご冗談でしょう?名無しさん:2009/10/11(日) 17:06:43 ID:???
いや合ってると思うよ。
しかし、良い質問だな。こんな所に気体の科学が隠れていたとは。
215ご冗談でしょう?名無しさん:2009/10/11(日) 17:09:56 ID:???
214はバカ
216ご冗談でしょう?名無しさん:2009/10/11(日) 17:22:02 ID:???
>>213
おかしいね。
断熱膨張が起きたとしても人間の感覚で捉えられるほどの温度差になるわけない。
温度境界層が吹き飛ばされるかどうかの違いだと思う。
217ご冗談でしょう?名無しさん:2009/10/11(日) 18:08:04 ID:???
>214
マジで言ってるのかよ。
温度計で比べてみろ。
218ご冗談でしょう?名無しさん:2009/10/11(日) 18:09:02 ID:???
ふと疑問に思ったのですが、何故メガホンを使うと声が大きくなるのですか?
219ご冗談でしょう?名無しさん:2009/10/11(日) 18:13:28 ID:???
>>218
音がメガホンの向いた方向に集中するから。
掌でも効果がある。
220ご冗談でしょう?名無しさん:2009/10/11(日) 18:14:16 ID:???
>>213
おかしいね
断熱膨張が起きるほどなら、断熱圧縮が起きてるはずの口腔内はどんだけ
高温になるんだっつーの。40℃は軽いはずだけど、んなこたムリだ。

ゆえにデマカセ。
この教授も頭で考えた仮説を検証せず垂れ流してるだけ。
221ご冗談でしょう?名無しさん:2009/10/11(日) 18:14:29 ID:???
エネルギーが散逸する範囲を小さく限定することで、音声のエネルギーが特定の領域に効果的に届くからです。
アンテナも同様の原理で、電磁波を特定の幾何学的領域に限定して送信できるように形状が工夫されています。
メガホンは、言わば「音声のアンテナ」です。
222ご冗談でしょう?名無しさん:2009/10/11(日) 21:31:23 ID:???
テスト
223ご冗談でしょう?名無しさん:2009/10/11(日) 22:38:39 ID:???
>>220 ちょっと自分で試すだけで何が正しいか分かりそうなもんだがねえ。
余程頭でっかちの人なのか?
そもそも肺の中で1気圧だったのが一旦高圧になって出ていって減圧する時に
温度低下が生じるなら、一瞬の間に断熱圧縮で起こった温度上昇を打ち消すだけの
熱の移動が口の中で起きなきゃならないことになるのにねえ。
224.:2009/10/11(日) 23:43:53 ID:???
225ご冗談でしょう?名無しさん:2009/10/12(月) 01:46:54 ID:???
小学生の理科くらいの質問で丁度いいな
このアホスレはw
226ご冗談でしょう?名無しさん:2009/10/12(月) 09:10:16 ID:6o0UeLiD
時間と波の関係で、空間が異なったら、時間の流れも変わるんでしょうか?波を波で打ち消した場合、時間はどうなるのでしょうか?それとも、時間はやはり幻想なんでしょうか?
227ご冗談でしょう?名無しさん:2009/10/12(月) 09:14:38 ID:???
>時間と波の関係で
そもそもこれが何を指してるのかわからんぞ
228ご冗談でしょう?名無しさん:2009/10/12(月) 09:33:51 ID:???
>>226
訊きたい内容が第三者にわからないとスルーされるよ
229ご冗談でしょう?名無しさん:2009/10/12(月) 10:02:25 ID:6o0UeLiD
時間の波が、時間の波、別の時間の波に打ち消された場合、時間の進み方はどうなるのでしょうか?
時間流 時間渦巻きがあるのでしょうか?
230ご冗談でしょう?名無しさん:2009/10/12(月) 10:06:19 ID:6o0UeLiD
時間の波が、時間の波、別の時間の波に打ち消された場合、時間の進み方はどうなるでしょうか?
時間流、時間巻、時間渦巻きはあるのでしょうか?
231ご冗談でしょう?名無しさん:2009/10/12(月) 10:18:20 ID:???
時間の流れを水の流れでイメージしているんみたいね。
だから、水の流れで説明できるならその時間の流れはある、で、
SF的な時間世界を構築してるのね。

それが時間の本当の姿だ、はひょっとしたらあるかもしんない。
でも、そのことを誰かに聞いても、ここだけのことじゃなくて、
今現在では誰もまじめには答えてくれないよ。

だから、それが真実だと信じるなら、
誰にも頼らず、君自身だけで、その時間論を作りあげるしかないんだな。

頑張ってね。オレは応援するから。(微)
232ご冗談でしょう?名無しさん:2009/10/12(月) 10:18:27 ID:???
すまんがオカ板にでも逝ってくれ
233ご冗談でしょう?名無しさん:2009/10/12(月) 11:24:25 ID:6o0UeLiD
1980年代の論文を書いてないない方達だ
234ご冗談でしょう?名無しさん:2009/10/12(月) 12:32:24 ID:6o0UeLiD
時間×速さ=距離 がヒントかな。おまいら解る人書いてぉ。
これも、幻想だが。
235ご冗談でしょう?名無しさん:2009/10/12(月) 13:09:51 ID:???
たった三日の連休でもこーゆーのが湧くのか
236ご冗談でしょう?名無しさん:2009/10/12(月) 13:53:07 ID:???
いつもの常連さんだろ、相手するなよ。
237ご冗談でしょう?名無しさん:2009/10/12(月) 13:55:22 ID:???
いつもの壊れたヒトね
238ご冗談でしょう?名無しさん:2009/10/12(月) 14:45:14 ID:LTWaRaHt
違います。
ゲテもの目当ての釣り師です。
239ご冗談でしょう?名無しさん:2009/10/12(月) 15:22:59 ID:???
誰もオマエになんか釣られてねーよ。
バカにしてからかってるだけだよ。www
240ご冗談でしょう?名無しさん:2009/10/12(月) 15:27:23 ID:???
真・スルー 何もレスせず本当にスルーする。簡単なようで一番難しい。
偽・スルー みんなにスルーを呼びかける。実はスルーできてない。
予告スルー レスしないと予告してからスルーする。
完全スルー スレに参加すること自体を放棄する。
無理スルー 元の話題がないのに必死でスルーを推奨する。滑稽。
失敗スルー 我慢できずにレスしてしまう。後から「暇だから遊んでやった」などと負け惜しみ。
願いスルー 失敗したレスに対してスルーをお願いする。ある意味3匹目。
激突スルー 話題自体がスルーの話に移行してまう。泥沼状態。
疎開スルー 本スレではスルーできたが、他スレでその話題を出してしまう。見つかると滑稽。
乞食スルー 情報だけもらって雑談はスルーする。
質問スルー 質問をスルーして雑談を続ける。
思い出スルー 攻撃中はスルーして、後日その思い出を語る。
真・自演スルー 議論に負けそうな時、ファビョった後に自演でスルーを呼びかける。
偽・自演スルー 誰も釣られないので、願いスルーのふりをする。狙うは4匹目。
3匹目のスルー 直接的にはスルーしてるが、反応した人に反応してしまう。
4匹目のスルー 3匹目に反応する。以降5匹6匹と続き、激突スルーへ。
241ご冗談でしょう?名無しさん:2009/10/12(月) 15:27:50 ID:???
お前が釣られてるじゃないか
242ご冗談でしょう?名無しさん:2009/10/12(月) 15:28:04 ID:???
失敗スルーwww
243ご冗談でしょう?名無しさん:2009/10/12(月) 15:33:53 ID:???
>>239
今日から君は失敗スルー君だ
244ご冗談でしょう?名無しさん:2009/10/12(月) 15:38:47 ID:???
バカがばれると「釣りだ」と言って誤魔化そうとするのはバカの常套手段
釣られもなんとかスルーもないよ
245ご冗談でしょう?名無しさん:2009/10/12(月) 15:49:13 ID:???
現在、激突スルー状態へ移行
246ご冗談でしょう?名無しさん:2009/10/12(月) 18:21:46 ID:6o0UeLiD
ブラックホールが宇宙自身を吸い込まないのはなぜ
247ご冗談でしょう?名無しさん:2009/10/12(月) 19:15:20 ID:???
>>246
オマエの頭がブラックホールだ
バレバレ馬鹿の誤魔化しが「釣りだ」で今回も終わりになるだけさ
そんなことさえ読めない阿呆も>>240-243みたいにいるけどな
ま、ブラックホール頭に飲み込まれるのはブラックホール頭だけさ
248ご冗談でしょう?名無しさん:2009/10/12(月) 19:39:24 ID:???

 >>247=>>239は確定的に明らかだなwwww
249ご冗談でしょう?名無しさん:2009/10/12(月) 19:42:04 ID:???
うむ同意。
同じ奴に釣られてるな。

これは恥ずかしい。
250ご冗談でしょう?名無しさん:2009/10/12(月) 19:46:08 ID:???
馬鹿の誤魔化しを「そうだそうだ」とするやつは馬鹿。
今度はテメエが次の馬鹿になって必死にバレた馬鹿を誤魔化す、ってさ。
阿呆が。w
251ご冗談でしょう?名無しさん:2009/10/12(月) 19:53:15 ID:???
馬鹿なのか阿呆なのかはっきりしてくれ
252ご冗談でしょう?名無しさん:2009/10/12(月) 19:58:50 ID:???
>>246
宇宙自体がBHだから
253ご冗談でしょう?名無しさん:2009/10/12(月) 19:59:56 ID:???
必死な人が一名いるみたいだけど…
254ご冗談でしょう?名無しさん:2009/10/12(月) 20:05:15 ID:???
>>253
オマエを入れて3名だよ。w
255ご冗談でしょう?名無しさん:2009/10/12(月) 20:49:19 ID:???
>>247
コピペにマジレスカコワルイ
256ご冗談でしょう?名無しさん:2009/10/12(月) 21:26:14 ID:???
メガホンの質問答えていただきありがとうございました。
257ご冗談でしょう?名無しさん:2009/10/13(火) 00:58:58 ID:???
お願いします。
微小座標変換x' = x + δxを考える時、1次近似で
∂f'(x')/∂x' = ∂f(x)/∂x
としているのを様々な本で見かけるのですが、証明しようとしても上手くいきません。
これはどうしてそうなるのでしょうか?
258ご冗談でしょう?名無しさん:2009/10/13(火) 01:08:06 ID:???
εを微小量とすると、

ε+ε^2=ε
1/(ε+ε^2)=1/ε(1-ε)=1/ε
f(a+ε)=f(a)+εf'(ε)
259ご冗談でしょう?名無しさん:2009/10/13(火) 01:14:15 ID:89Z7Kqh1
こんばんわ。お願いします。
楕円の断面2次モーメントの公式I=(π/4)×ba^3というのがあると思うんですけど、
この途中式(過程)を教えてほしいです。
x軸に対する断面2次モーメントがI=x^2dA
で、xを三角関数に変形して、求めるということはなんとなく
わかるんですが、どう変形していったら公式のようになるのかわからないです・・・。
260257:2009/10/13(火) 02:04:29 ID:???
>>258
レス有り難う御座います。
しかしこの結果をどう使えばよいのでしょうか?
済みませんがもう少しヒントを頂けないでしょうか?
261ご冗談でしょう?名無しさん:2009/10/13(火) 02:07:38 ID:7Etx+MX+
論証などで、反論できないなら、教授なんてやめてしまえ、このバカども。
262ご冗談でしょう?名無しさん:2009/10/13(火) 02:11:29 ID:7Etx+MX+
論理、証明、これができないなら、物理などやめてしまえ。
263ご冗談でしょう?名無しさん:2009/10/13(火) 02:17:22 ID:7Etx+MX+
波理論も知らないのか。海外の論文もっと読め。
264ご冗談でしょう?名無しさん:2009/10/13(火) 02:31:14 ID:kJ6OJJt5
http://ja.wikipedia.org/wiki/%E8%A7%92%E9%81%8B%E5%8B%95%E9%87%8F
ここのwikiの次元のとこって間違ってますか?
たぶん、ML^2T^-1かなと思ったんですが・・・
265ご冗談でしょう?名無しさん:2009/10/13(火) 02:35:06 ID:???
>>264
wikiが間違ってるね
266ご冗談でしょう?名無しさん:2009/10/13(火) 02:45:39 ID:kJ6OJJt5
>>265 サンキューです
267ご冗談でしょう?名無しさん:2009/10/13(火) 02:48:24 ID:vVptiDcm
>259なんですけど、調べても全然分かんないです・・・。
教えてください。
268ご冗談でしょう?名無しさん:2009/10/13(火) 02:54:08 ID:7Etx+MX+
おまいらも、世界トップレベルに入れ。世界で認められる先生になれ!!
269ご冗談でしょう?名無しさん:2009/10/13(火) 05:32:00 ID:7Etx+MX+
円運動量について書け
270ご冗談でしょう?名無しさん:2009/10/13(火) 07:40:26 ID:7Etx+MX+
東工大では、常識
271ご冗談でしょう?名無しさん:2009/10/13(火) 07:46:15 ID:7Etx+MX+
パリ大学に推薦状だしてやるよ。
272ご冗談でしょう?名無しさん:2009/10/13(火) 08:27:25 ID:???
リアルでハブにされたり虐められてる奴の典型だな。
居丈高なレスが並んでいるが、物理の資質を感じさせるものはない。
273ご冗談でしょう?名無しさん:2009/10/13(火) 08:51:12 ID:???
>>259
楕円の式はx=b・cosθ, y=a・sinθ。(a、bが普通とは逆)
x軸回りの断面二次モーメントはI = ∫y^2dA 。
dAはx軸からyだけ離れた位置の帯状の部分の面積で、横幅は2x=2b・cosθ、縦はdy=a・cosθdθ。
この置き換えをしてθについて-π/2からπ/2まで積分。

274ご冗談でしょう?名無しさん:2009/10/13(火) 09:22:49 ID:???
数学板で聞いたら物理板で聞けと突っ返されたので

物理系のB4です。
3階の極性c-テンソルに磁化をかけることで4階の軸性i-テンソルに変換させるという研究をしています
3階の極性c-テンソルの時点で磁化の効果は考慮されているのですが、より取り扱いを簡単にするために4階の軸性i-テンソルにする必要があるのです

要は変換によりテンソルの対称性が保存されればいいのですが、磁化の効果(磁化の方向等の情報)を残した上でテンソルを変換する変換群が見つかりません。
出来るだけ数学的に厳密にやりたいのですが、何か良い方法は無いでしょうか
275ご冗談でしょう?名無しさん:2009/10/13(火) 10:04:55 ID:???
>>274補足
テンソルTについて
T[pqr]=a[ip]a[jq]a[kr]T[ijk]

T[pqrs]=a[ip]a[jq]a[kr]a[ls]T[ijkl]
に変換したいと言うことです
276ご冗談でしょう?名無しさん:2009/10/13(火) 10:46:01 ID:???
>>274 これだけじゃ情報不足。そもそも脚の数変えるなんてできない。
解釈としては磁場依存性の部分の脚があり、当初のやつでは(磁場を
固定しているので)その部分が見えなくなってる、ってことだろうけど、
そしたら当初の奴の変換性が君の書いたようなものではあり得なくなる。
ので、こちらとしても答えようが無い。
277257:2009/10/13(火) 11:23:13 ID:???
どなたか>>257お願い出来ないでしょうか?
∂f'(x')/∂x' = ∂(f(x) + δf(x))/∂(x + δx)
= (∂f(x) + ∂δf(x))/(∂x + ∂δx) ≒ ∂f(x)/∂x
とすればつじつまは合いますがインチキですよね…
278ご冗談でしょう?名無しさん:2009/10/13(火) 12:28:09 ID:???
>>276
ですよね…。
俺も其処で苦しんでるんで。
やっぱ定性的な議論は無理か…定量的な議論で我慢しよう
279ご冗談でしょう?名無しさん:2009/10/13(火) 13:00:17 ID:???
波はベクトルで表せますか?
280.:2009/10/13(火) 19:00:58 ID:???
メイプル最古ぉぉぉぉ!!!!!!
281ご冗談でしょう?名無しさん:2009/10/13(火) 20:39:22 ID:???
>>279
波数ベクトルというモノがあってだな…
282.:2009/10/13(火) 20:59:01 ID:???
メンテ長いなーーーっ
283ご冗談でしょう?名無しさん:2009/10/13(火) 23:10:48 ID:???
簡単なリニアモーターカーを作りたいと思っています。
http://viploader.net/ippan/src/vlippan022466.jpg
↑このような構造で作ろうと思っているのですが、
それぞれの磁石の極を教えていただけないでしょうか
284ご冗談でしょう?名無しさん:2009/10/13(火) 23:14:37 ID:???
>>283
まずモーターがどういう原理なのか勉強したら?
285ご冗談でしょう?名無しさん:2009/10/13(火) 23:15:07 ID:???
何を考えてるのか分からないけど、永久磁石だけだったら無理だよ?
286ご冗談でしょう?名無しさん:2009/10/13(火) 23:22:05 ID:???
>>283
浮く模型は作れてもリニアモーターはソレじゃないから
287ご冗談でしょう?名無しさん:2009/10/13(火) 23:24:16 ID:???
てっとり早くやりたいなら地上一次の誘導リニアが簡単だろうね
材料さえあれば1時間で完成する
288ご冗談でしょう?名無しさん:2009/10/13(火) 23:25:13 ID:???
磁気浮上とリニアモーターは元々は別の原理だからな
289ご冗談でしょう?名無しさん:2009/10/13(火) 23:51:49 ID:???
>>277
x' = x + ε(x)
y = f(x) = f'(x')とおく。

ふつうにチェインルール(合成関数の微分)を使って、
ε'(x) が微小と仮定すれば言えると思うけど。

y = f(x) = f'(x')として
∂y/∂x = ∂x'/∂x ・ ∂y/∂x'
= ∂(x+ε(x))/∂x ・ ∂y/∂x'
= (1 + ε'(x)) ・ ∂y/∂x'
≒ ∂y/∂x'

まあ、ε(x)が小さくてもε'(x)は小さくないとか言い出せば
成り立たないけどね。物理でε(x)が微小と言えば、微分も込めて
微小と仮定していることが多いでしょう。
290ご冗談でしょう?名無しさん:2009/10/14(水) 00:10:22 ID:???
八木アンテナはなぜ魚の骨のように、数本の棒を平行に配置しているのか。
指向性を持たせるためだ。

指向性とは、ようするに干渉の利用だ。

このアンテナを送信に使えば、方向によって送信される電波強度は変わるだろう。

何日か前の、携帯コヒーレント坊やたち。
上記の魚の骨の意味を熟考してみてほしい。
291ご冗談でしょう?名無しさん:2009/10/14(水) 00:18:49 ID:???
また変なのキター
292283:2009/10/14(水) 01:49:34 ID:???
>>284-288
ありがとうございます
どうやら勘違いをしていました。
私が作成したかったのは、磁気で浮上走行する模型だったようです。
293257:2009/10/14(水) 03:55:24 ID:???
>>289
レス有り難う御座います。
示して下さった計算の最後の行の≒では微小項の一次を落としていると思うのですが、
私が見た本(佐藤勝彦の相対性理論のp102、greinerのfield quantizationのp41)ではどちらでも、
この近似を一次近似で成り立つとしています。
また、greinerの方ではx' = x + δxの時必ずしもf'(x') = f(x)ではなく、f'(x') = f(x) + δf(x)と仮定しています。
チェーンルールでやってみるとどうしても一次の項が残ってしまうようなのですが…。
294ご冗談でしょう?名無しさん:2009/10/14(水) 08:09:58 ID:en2TEoJF
励起について質問です
エネルギー準位E0、E1、E2という準位があったとして(E0<E1<E2)
E0の状態の電子にエネルギーがE1<E'<E2の光をあてたら電子の励起はどうなりますか?
E1まで励起されてE'-E1はどうなりますか?
295ご冗談でしょう?名無しさん:2009/10/14(水) 08:49:13 ID:???
>>292
磁気浮上を実現するには、ちょっと横にずれたり傾いたとき元の位置に戻るように
磁極を配置する必要がある。その辺の工夫が面白いところなのでがんばってね。
296ご冗談でしょう?名無しさん:2009/10/14(水) 08:53:44 ID:nlmDCH6o
>>294
>エネルギーがE1<E'<E2の光をあてたら
ノーベル賞貰えるよ。
297ご冗談でしょう?名無しさん:2009/10/14(水) 10:11:38 ID:???
>>288
リニアモーターの意味分かってない奴多いよね
298ご冗談でしょう?名無しさん:2009/10/14(水) 10:21:08 ID:???
>>296
例えば水銀原子に太陽光あてれば
エネルギーがE1<E'<E2の光はあると思いますが
299ご冗談でしょう?名無しさん:2009/10/14(水) 11:08:01 ID:CQYHym0r
>>297
磁気浮上も分かってないの多いし
基本的にものの仕組み分かってないの多すぎるね
2chだからしょうがないけど
300ご冗談でしょう?名無しさん:2009/10/14(水) 14:24:40 ID:???
>>294
エネルギーE'がE1-E0<E'<E2-E0の光をあてたら、
という意味だよね?

それは励起の確率がおそろしく小さいから、考えなくていい。
時間に依存する摂動論のところで
フェルミのゴールデンルールとか、勉強したでしょ。
301ご冗談でしょう?名無しさん:2009/10/14(水) 17:00:18 ID:???
冷蔵庫内に冷蔵庫用脱臭剤を入れておいたら、食品自体に脱臭剤の化学物質的な臭いが移りました。
その食品は身体に害はありますか?味はとても嫌な感じです。

液体の化学物質が身体に悪いように気体の化学物質(を取り込んでるってことですよね?)もヤバいですよね?

言いたいことが伝わると嬉しいです。

アホ内容、アホ文章ですみません。
302ご冗談でしょう?名無しさん:2009/10/14(水) 17:20:42 ID:en2TEoJF
静電ポテンシャルはなぜマイナスがついているのでしょうか?
303ご冗談でしょう?名無しさん:2009/10/14(水) 17:23:21 ID:???
そういう類の脱臭剤はなるべく食品から離して置くものじゃないのかな?
詳しくは脱臭剤の裏面表記とかみたりサイトで調べるとかのほうがいいんじゃないの?
成分とかがわかればまだ具体的なことも言えるけど、情報がないからなんともアドバイスできない。
304ご冗談でしょう?名無しさん:2009/10/14(水) 18:10:23 ID:???
>>303
回答ありがとうございます
特に離しておいて下さいとの表記もなかったと思います。
成分…
カテキン、他は忘れてしまいました。
305ご冗談でしょう?名無しさん:2009/10/14(水) 18:31:16 ID:???
>302
変だな、俺のはプラスだけど
306ご冗談でしょう?名無しさん:2009/10/14(水) 18:42:36 ID:kztA2te0
9×4はどうして36なんていう小さい値になるの?
307ご冗談でしょう?名無しさん:2009/10/14(水) 18:44:41 ID:???
何度も見たことのある釣り
308.:2009/10/14(水) 19:07:28 ID:???
戦争反対
戦争反対
戦争反対
戦争反対
戦争反対
戦争反対
戦争反対
戦争反対
戦争反対
戦争反対
戦争反対
戦争反対
戦争反対
戦争反対
九条九条九条九条九条九条九条九条九条
九条九条九条九条九条九条九条九条九条
九条九条九条九条九条九条九条九条九条
九条九条九条九条九条九条九条九条九条
九条九条九条九条九条九条九条九条九条
309ご冗談でしょう?名無しさん:2009/10/14(水) 19:11:28 ID:???
電磁気力はクーロンの法則の中にεがあり、
空間にどんなモノがあるかで力の強さが変わるみたいですが、
重力は変わらないんですか?
F=GmM/R^2の中に、2物体間の空間にどんなモノがあるかで変わる変数は
ないですよね?

初歩的な質問ですんません
310ご冗談でしょう?名無しさん:2009/10/14(水) 19:52:54 ID:???
>>309
電磁気力も、本当は全部ε_0で変わらないよ。
絶縁体(=誘電体)の中にある電荷のうち、
一部の正と負の電荷が互いに逆向きに少しだけずれる効果を
便宜的にεで現しているだけで、
それらの電荷もちゃんと含めるならε_0だけでいい。

重力は、常に引力だから、そういう便宜的な扱いが
そもそもできないのかもね。
311ご冗談でしょう?名無しさん:2009/10/14(水) 20:03:54 ID:???
半減期の長い放射性元素は、あんまり分裂しないので安全に感じるのですが、この考えはあっていますか。
もし間違っていたらどこが間違っているか教えてください。
312ご冗談でしょう?名無しさん:2009/10/14(水) 20:31:33 ID:???
>>311
量、濃度による。
ウラン235のように連鎖反応を起こす場合もあるし、半減期が長いと長期間放射線を出し続けることになる。

チェルノブイリ原発の事故を見れば半減期数億年だからといって安全でないことは分かるだろう。
313309:2009/10/14(水) 20:48:51 ID:???
>>310
なるほど、解答ありがとうございます
314ご冗談でしょう?名無しさん:2009/10/14(水) 22:09:18 ID:e7eixSvf
周囲との摩擦が無い状態では、
物体の回転は永久に続きますか?
宇宙空間のようなところ。
315ご冗談でしょう?名無しさん:2009/10/14(水) 22:13:54 ID:???
>>314
常識的な質量の物体なら摩擦がなければ、ずっと回転してるよ。

中性子星クラス以上になると別だけどね。
316ご冗談でしょう?名無しさん:2009/10/14(水) 22:20:02 ID:MoOpowbP
人間は宇宙空間で静止した状態から回転できる。
回転体の内部運動によって止まることも、逆回転も可能。
317ご冗談でしょう?名無しさん:2009/10/14(水) 22:31:14 ID:???
>>316
回転はできんよ。角運動量保存則があるからね。
向いてる方向は任意に変更できるけど。
318ご冗談でしょう?名無しさん:2009/10/14(水) 22:38:17 ID:MoOpowbP
>>317
んー。。。どうやらそうらしい。スンマソン
319ご冗談でしょう?名無しさん:2009/10/14(水) 22:43:43 ID:MoOpowbP
しかしあれだろ、ずーっと向きを変えようとしてれば
結果的に回転してることになるだろ。疲れるけど。
320ご冗談でしょう?名無しさん:2009/10/14(水) 22:48:56 ID:???
角運動量がゼロのその過程を、回転と呼ぶならばそう。

普通は呼ばない。
321ご冗談でしょう?名無しさん:2009/10/14(水) 22:53:44 ID:???
向きを変えるには何かを逆向きにまわさなきゃならない。

体から分離できるものを持ってればそれをまわせばいいけど、なければ
体をねじるしかないので限度がある。
322ご冗談でしょう?名無しさん:2009/10/14(水) 22:55:13 ID:???
と思ったが、手をぐるぐる回せば体は逆回転できるな。
323ご冗談でしょう?名無しさん:2009/10/14(水) 22:57:22 ID:MoOpowbP
そうすか・・・
いや こいつはちょっと面白いな〜
これってどーして「回転」とはならないのかなー・・・。
方向転換作業をぶるぶると微分的に連続すれば回転と見分けがつかなくなんじゃねーかな・・?

これって角運動量の保存則とどうやって整合性を保つんだろ・・。
324ご冗談でしょう?名無しさん:2009/10/14(水) 22:59:14 ID:???
おもしろい問題だけど、おまえら、いすを壊すなよw
325ご冗談でしょう?名無しさん:2009/10/14(水) 23:02:31 ID:???
これって、若田さんでも、体の向きはかわるのか?
326ご冗談でしょう?名無しさん:2009/10/14(水) 23:03:55 ID:???
外力無しに方向転換は普通に出来る。
曲がった空間なら、外力無しに加速も出来る。

http://physics.technion.ac.il/~avron/about.html
327ご冗談でしょう?名無しさん:2009/10/14(水) 23:13:34 ID:???
>>323
手をぐるぐる回し続けても、身体のゆっくりした回転とで角運動量の総和はゼロ。
回転してることにはならない。

身体の屈伸と、手脚の回転でヨー、ロール、ピッチいずれの方向へも
方向変換可能。

ガン○ム世界のAmbacシステムの基本
328ご冗談でしょう?名無しさん:2009/10/14(水) 23:20:26 ID:MoOpowbP
>>327
んだから
「ぐるぐる、ぴたっ。ぐるぐる、ぴたっ。・・」って作業をさ、すごいすばやくやるんだよ。
そしたら時計の針みたいにチクタク回転するように見える・・・でしょ?
これは「回転」とは呼べないの??なぜかしら。不思議よね〜
329ご冗談でしょう?名無しさん:2009/10/14(水) 23:29:36 ID:MoOpowbP
いやまて そんなめんどくさいことしなくてもだな、
若田さんがカプセルに入ってぐるぐる手を回せば若田さんの胴体は回転するだろ?
このカプセルが若田さんの胴体と一致して回転するなら、
カプセルはやっぱり回転することになるんじゃねえの?
330ご冗談でしょう?名無しさん:2009/10/14(水) 23:31:59 ID:???
カプセルは逆に回るだろ、ちゃんと保存されてる
331ご冗談でしょう?名無しさん:2009/10/14(水) 23:39:44 ID:MoOpowbP
>>330
おおそうさね!

そうか、じゃあ>>316でだいたいオッケーじゃんかよー。

され角運動量も保存されたことだし、安心してもー寝るわい。
332ご冗談でしょう?名無しさん:2009/10/14(水) 23:49:19 ID:???
若田さんの前か前の前の日本人宇宙飛行士がISSで手足バタバタさせながら
方向転換する動画があったな。
333ご冗談でしょう?名無しさん:2009/10/15(木) 07:47:28 ID:???
無重力下では、液体の拡散係数や熱伝導が正確に測定できるのですか?


「米国空軍は定常的な観測で数mm以上のスペースデブリをほとんどカタログ化している」という
記述を正しいと思うのですが、後で調べたら米国空軍ではなくてNORADが正確な組織名でした。

これはやはり間違いとされるのでしょうか?
334ご冗談でしょう?名無しさん:2009/10/15(木) 08:22:35 ID:???
>>333
> 無重力下では、液体の拡散係数や熱伝導が正確に測定できるのですか?

両方とも重力に影響を受ける物理定数ではない。

> 「米国空軍は定常的な観測で数mm以上のスペースデブリをほとんどカタログ化している」という
> 記述を正しいと思うのですが、後で調べたら米国空軍ではなくてNORADが正確な組織名でした。
>
> これはやはり間違いとされるのでしょうか?

違う組織なので間違い。
中の人は互換性があったりもするけどね。
335ご冗談でしょう?名無しさん:2009/10/15(木) 10:13:13 ID:IfwVJANJ
このサイトってどうよ?
ttp://heart.15jam.net/
登録してみようか迷ってるんだがw
336ご冗談でしょう?名無しさん:2009/10/15(木) 11:15:03 ID:5rIOlFGg
保存力であることを確かめるにはどうすればいいですか?
337ご冗談でしょう?名無しさん:2009/10/15(木) 11:33:11 ID:???
>>336
もしf=-grad φとかけるなら
rot f = 0
となっているはず。
338ご冗談でしょう?名無しさん:2009/10/15(木) 15:44:58 ID:1KlDgx24
量子力学の質問です。

立方体(一辺L)に閉じこめられた電子の波動関数についてですが
波動関数が
ψ=Cexp(ikr)
とした場合と
ψ=Csin(kx)sin(ky)sin(kz)
とした場合

量子数が
ψ=Cexp(ikr)とした場合
n_x=0,±π,±2π...
でψ=Csin(kx)sin(ky)sin(kz)とした場合
はnが整数になります。
これはなぜなのでしょうか?
いま考えたら後者はなぜnが負ではいけないのでしょうか?
339ご冗談でしょう?名無しさん:2009/10/15(木) 16:13:04 ID:???
波動の問題で端が両方固定されている問題でこの条件から(端から端の長さL)
f=Asin(nπ/L)x
となります。条件はnが正の整数らしいのですが、なぜ負はだめなのですか?
340ご冗談でしょう?名無しさん:2009/10/15(木) 16:30:33 ID:???
+nと-nが同じ状態を表すから、負のnも認めてしまうと状態数を二倍に誤ってカウントしてしまう。
341ご冗談でしょう?名無しさん:2009/10/15(木) 16:32:46 ID:1KlDgx24
>>340
量子力学以外の波動の問題ではなぜnは正の整数なんですか?
342ご冗談でしょう?名無しさん:2009/10/15(木) 16:59:35 ID:???
量子だろうが古典だろうが同じものを2回数えるな。
343ご冗談でしょう?名無しさん:2009/10/15(木) 21:56:31 ID:???
どなたかこれの動作原理を教えてください
http://www.nicovideo.jp/watch/sm8482198
または
http://d.hatena.ne.jp/video/niconico/sm8482198
344ご冗談でしょう?名無しさん:2009/10/16(金) 00:20:10 ID:???
V=(a,b,c)とすると
∇Vと∇・Vはそれぞれ
(da/dx,db/dy,dc/dz) (ベクトル)
(a*d/dx+b*d/dy+c*d/dz) (スカラー)

これであってるでしょうか?
345ご冗談でしょう?名無しさん:2009/10/16(金) 00:33:11 ID:???
あってない
346ご冗談でしょう?名無しさん:2009/10/16(金) 00:37:06 ID:???
>>344
ぜんぜんだめ。
∇・V = div V = ∂a/∂x + ∂b/∂y + ∂c/∂z (スカラー)
V・∇ = a*∂/∂x + b*∂/∂y + c*∂/∂z (微分演算子)
あと∇Vは一般にはテンソルだけど、文脈がよくわからん。
347ご冗談でしょう?名無しさん:2009/10/16(金) 10:01:58 ID:???
tr(∇V) = divV ?
348ご冗談でしょう?名無しさん:2009/10/16(金) 12:32:54 ID:oQqUIdeL
>346

ありがとうございます。

ナビエストークス方程式を利用して流速から圧力を求めるプログラムを作ろうとしているんですが、
記号の意味がなかなか理解できず、聞く相手も身近にいないので
ここで質問した次第です
349ご冗談でしょう?名無しさん:2009/10/16(金) 12:33:52 ID:oQqUIdeL
あと6を左右逆にした文字はなんと読みますか?
350ご冗談でしょう?名無しさん:2009/10/16(金) 12:59:03 ID:???
偏微分記号∂のことならデル、ラウンドディーとかだろ。
351ご冗談でしょう?名無しさん:2009/10/16(金) 13:11:12 ID:???
八月頃、一般者(高校生とその親御さん)を対象に、X線γ線で見る宇宙と言う題目で宇宙について講義をしました。

そして最後にアンケート的なものを行ったのですが、
そのアンケートに専門用語が多くて話が分かりづらいと言うものが多数でした。

私的には専門用語は全く使ってないつもりだったのでかなり意外でした。

そこで質問なんですが、どういう言葉が専門用語にあたるのかを教えて下さい。同じ経験あるかたや、逆に講義聞く側の人いたら是非教えて下さい。

私的に重力、速度などは専門用語とは思っておりません。
352ご冗談でしょう?名無しさん:2009/10/16(金) 13:17:36 ID:???
ガンマ線やX線は勿論、波長や電磁波って単語ですら一般人の高校生その父兄にとっては専門用語。
進学校の理系とかなら、兎も角ね。
353ご冗談でしょう?名無しさん:2009/10/16(金) 13:18:27 ID:???
>>351
そっちから具体的に列挙してくれないと。
物理的に何を指してるのか明確なものはすべて専門用語だろ。当然、重力も速度も専門用語。
354ご冗談でしょう?名無しさん:2009/10/16(金) 13:55:54 ID:???
速度は専門用語だろうな。
「速さと向き」って言わないと一般の人に分からないだろ。
355ご冗談でしょう?名無しさん:2009/10/16(金) 13:58:40 ID:???
一般人に話すのに専門用語を多用する人って
その人自身がその専門用語の定義を正しく理解していない
って場合が往々にしてあるな。
356ご冗談でしょう?名無しさん:2009/10/16(金) 14:02:19 ID:???
>>355
そいつが元々一、般人に毛が生えた程度の場合にはね。
357ご冗談でしょう?名無しさん:2009/10/16(金) 18:41:39 ID:???
専門用語とかどうでもいい
358ご冗談でしょう?名無しさん:2009/10/16(金) 19:05:27 ID:+mumxz9o
粒子の平均の散乱時間がτだとしたら衝突から次の衝突までの時間はどのくらいになりますか?
359ご冗談でしょう?名無しさん:2009/10/16(金) 19:10:16 ID:???
>>358
> 平均の散乱時間
の定義は?
360ご冗談でしょう?名無しさん:2009/10/16(金) 19:15:03 ID:+mumxz9o
>>359
ぼくの持っている本に
平均の散乱時間がτならば衝突から次の衝突まで時間は2τであると書いてあるますがなぜなのかわからないのです。
361ご冗談でしょう?名無しさん:2009/10/16(金) 20:11:34 ID:???
それが散乱時間τの定義と考えるのが自然。
もうちょっと正確に表現すれば、
「衝突から次の衝突までの”平均”時間が2τ」⇔「”平均の散乱時間”をτと定義する」
362ご冗談でしょう?名無しさん:2009/10/16(金) 20:22:26 ID:???

要は「任意の時点から次の衝突までの衝突時間の平均」てことね。
363ご冗談でしょう?名無しさん:2009/10/16(金) 20:25:06 ID:S9+Lbg0N
>>290

この人はどっちの人だろう?

普通に設計されたアンテナは、全体が共振状態になるから、出てくる電波は位相が揃ってる、と言うか、一つの位相なんだ。
一般の光は、多数の電子などが非同期に発するからコヒーレントにならない。それを同期させるのがレーザー。

だから、アンテナから出てくる電波は、光の用語で言えばコヒーレントなんだよ。
364ご冗談でしょう?名無しさん:2009/10/16(金) 20:42:51 ID:sUSAWx+E
>>361
ごめんなさい。
まだわかないです。

平均の散乱時間がτなら衝突から次の衝突までの平均時間はτにならないのでえしょうか?
散乱時間は衝突した瞬間から次の衝突の間の時間じゃないんですか?
365ご冗談でしょう?名無しさん:2009/10/16(金) 20:54:45 ID:sUSAWx+E
また別の表現で
衝突から衝突までに要する時間は衝突が起こるたびにちがう。
よって簡単のためこの異なる衝突時間をある平均の衝突時間τとおく。
366ご冗談でしょう?名無しさん:2009/10/16(金) 20:59:50 ID:???
>>364
>>362読んでもわかんないの?
367364,365:2009/10/16(金) 21:01:01 ID:sUSAWx+E
>>361
ごめんなさい。
>>362でなんとなくつかめました。
「任意の時点から次の衝突までの衝突時間の平均」で定義するなら話がわかりました。
368ご冗談でしょう?名無しさん:2009/10/16(金) 21:19:27 ID:???
衝突から衝突までに要する時間は衝突が起こるたびにちがう。
よって簡単のためこの異なる衝突時間をある平均の衝突時間τとおく。

「任意の時点から次の衝突までの衝突時間の平均がτ」

ってなんかおかしくないか?
任意の時点=衝突した時点てすると
「衝突した時点から次の衝突までの衝突時間の平均がτ」
となり
「衝突から次の衝突までの”平均”時間が2τ」
と矛盾しないか?
369ご冗談でしょう?名無しさん:2009/10/16(金) 21:23:42 ID:???
>>368
> 任意の時点=衝突した時点てすると

かなり頭悪くね?
370ご冗談でしょう?名無しさん:2009/10/16(金) 21:25:16 ID:???
>>369
じゃあこのケースはどう説明すればいいんだ?
371ご冗談でしょう?名無しさん:2009/10/16(金) 21:48:48 ID:???
ある瞬間(=今)に無作為に選んだ1個の粒子に注目する。この粒子が
・最後に衝突してから今までの平均時間がτ
・今から次に衝突するまでの平均時間もτ
→ 衝突から衝突までの平均時間は 2τ
てことでは?
372ご冗談でしょう?名無しさん:2009/10/16(金) 22:27:35 ID:???
>>370
「このケース」が何を指してるのかようわからんが、

「衝突から衝突までの平均時間」
=「任意の時点から直前の衝突までの衝突時間の平均」+「任意の時点から次の衝突までの衝突時間の平均」

なんじゃねーの?そして、時間の対称性から右辺の二項は等しい。
373ご冗談でしょう?名無しさん:2009/10/17(土) 00:57:46 ID:???
宇宙環境利用に関する記述について誤っているものはどれか?

@落下実験施設は航空実験と比較して簡便であるが、微小重力環境は悪い。

A浮遊試料の位置を保持する方式として音波浮遊、電磁浮遊、静電浮遊がある。

B微小重力を利用して結晶生成を行う場合、表面張力勾配に起因して発生するマラゴンニ対流には注意が必要である。

C無重力下では、液体の拡散係数や熱伝導が正確に測定できる。

D宇宙機の質量中心においても、大気抵抗や太陽放射圧によって完全な無重力から外れて微小な残留重力が発生する。

Cが誤りだと思います。が、みなさんの見解はどうでしょうか?
374ご冗談でしょう?名無しさん:2009/10/17(土) 01:20:08 ID:???
初歩的な質問なんですが、円筒のピストンをゆっくり動かした場合エントロピーは変化しないというのはどういうことですか?
σはエントロピー、Cは定数、dx/dtは速度
dσ/dt=C+C'(dxdt)+C''(dx/dt)^2+.......
dx/dt=0のときdσ/dt=0よりC=0
dσ/dt≒C''(dx/dt)^2
とあるのですが、この式の意味が全く分かりません
もしかしたら抜けていたり、間違っていたりする部分があるのかもしれないのですが、どなたか分かる方お願いします
375ご冗談でしょう?名無しさん:2009/10/17(土) 01:39:52 ID:???
>>373 1だね
376ご冗談でしょう?名無しさん:2009/10/17(土) 01:46:14 ID:???
>>375
マジで?ソースは?
きぼうの実験よりも落下の方がいいの?
377ご冗談でしょう?名無しさん:2009/10/17(土) 02:11:16 ID:???
>>376
比較対象は「航空実験」だろ。
378ご冗談でしょう?名無しさん:2009/10/17(土) 02:18:50 ID:???
航空実験の何が悪いの?
重力を受けやすいということ?
379ご冗談でしょう?名無しさん:2009/10/17(土) 02:38:03 ID:???
>>374
どういう過程を考えているのか、文脈はないの?

たぶん、中身のエントロピーの変化率dσ/dtはピストンの速度vの
なめらかな関数だから、v=0の近くで
dσ/dt = c0 + c1 v + c2 v^2 + ....

と展開できる。
ここで、考えている過程の性質からc0=0とc1=0が言える。
よって、エントロピーの変化率の展開式はvの2次から始まるゆえ
dσ/dt = c2 v^2 + O(v^3)

みたいな議論なのでしょう。
過程がわからんから、c0, c1が消える理由もわからん。
380ご冗談でしょう?名無しさん:2009/10/17(土) 08:36:05 ID:???
>>374
よーわからんが、準静的な断熱過程ならエントロピーが一定なのは当たり前だぞ。
381ご冗談でしょう?名無しさん:2009/10/17(土) 11:48:30 ID:???
>>377
飛行機を正確に自由落下させるのが簡単だと思うのか?
382ご冗談でしょう?名無しさん:2009/10/17(土) 12:02:42 ID:P5bANymh
>>372
それなら平均自由行程は
l=v・2τ
になるの?
383ご冗談でしょう?名無しさん:2009/10/17(土) 12:42:00 ID:???
>>368,370

任意の日を選んだとき、その日が日曜日である確率 = 1/7

任意の日 = 日曜日とするなら

日曜日が日曜日である確率 = 1

で矛盾だなw
384ご冗談でしょう?名無しさん:2009/10/17(土) 12:46:41 ID:???
例えば、アメリカから日本へ音を送る場合、音をデータに変換して日本へ送って、
データを音に変換すれば、実際の音の速度よりも早く音を送るということは可能ですか?
また、直径10km程度の隕石が落下した場合、落下地点にはどれくらいの力がかかるんでしょうか?
10km程度の大きさの隕石で、地球に致命的な打撃を与えるほどのエネルギーが生じるというのが信じられません。
385ご冗談でしょう?名無しさん:2009/10/17(土) 12:47:06 ID:???
>>382 vが平均速度ならそうだろうさ
386ご冗談でしょう?名無しさん:2009/10/17(土) 12:54:38 ID:P5bANymh
>>385
そうなんですか?
本では散乱時間τとしておいて平均自由行程は
l=vτ
ってなってます……
387ご冗談でしょう?名無しさん:2009/10/17(土) 12:59:48 ID:???
>>384
国際電話って知ってる?
388ご冗談でしょう?名無しさん:2009/10/17(土) 13:05:41 ID:???
τの定義ぐらい調べろよ。

そして衝突がどういった確率過程なのか指定しないとその議論は無意味。
389ご冗談でしょう?名無しさん:2009/10/17(土) 13:14:41 ID:P5bANymh
>>372がτの定義
390ご冗談でしょう?名無しさん:2009/10/17(土) 13:36:01 ID:???
>>384
衝突する速度が半端じゃないからね。
391ご冗談でしょう?名無しさん:2009/10/17(土) 13:47:31 ID:???
>>384
>>10km程度の大きさの隕石で、地球に致命的な打撃を与えるほどのエネルギーが生じる

地球がどうなったら、「致命的な打撃」と言えるのかが問題だ。
392ご冗談でしょう?名無しさん:2009/10/17(土) 14:16:05 ID:???
10kmじゃあ人類文明が滅ぶだけで、天体としての地球はおろか生命すら存続するレベルだけどな。

「致命的」ってのは、こう言うのを言う
http://www.dotup.org/uploda/www.dotup.org270079.jpg
393ご冗談でしょう?名無しさん:2009/10/17(土) 14:18:22 ID:???
>>379,380
ありがとうございます
過程はよくわからないんですが、もう少し自分で調べてみます
394ご冗談でしょう?名無しさん:2009/10/17(土) 14:53:28 ID:E25kvlfs
ランダウ、量子力学(小教程)、ちくま学芸文庫p-230に関して質問です。
 要は第二量子化のことなのですが、占有数が独立変数の役割を果たすような
 数学的方法を作るにあたっての質問です。

->f_aをa番目の粒子に関する物理量の演算子としよう。すなわちそれはξ_a
 (座標やスピンの組)の関数の関数にだけ作用する。すべての粒子に関して対称な演算子
                F=Σ_a f_a
  を導入しよう。対称化された波動関数にかんして、この演算子の行列要素を定義しよう。
  まず最初に行列要素がゼロとならないのは、数N_1,N_2,…が変化しない遷移およびこれら
  の数のうちの一つが1だけ増加して他の一つが1だけ減少する遷移の場合だけであることが
  容易にわかる。(都合により一部改変)

  とあるのですが行列要素がゼロにならない理由がわかりません。
  生成消滅演算子が先に与えられているのならば話はわかるのですが、
  生成消滅演算子や場の演算子がでてくるのはこのはなしの次なのです。

  どうかよろしくお願いします。  
395ご冗談でしょう?名無しさん:2009/10/17(土) 15:45:41 ID:???
wikiの平均自由時間も
「衝突から衝突までの平均した時間を平均自由時間τという」
って定義しといて
導出で「衝突から次の衝突までの平均時間を2τとする」ってあるけどなにがちがうの?
396ご冗談でしょう?名無しさん:2009/10/17(土) 15:50:24 ID:kO/Fil70
躁鬱病の治し方は、薬で、躁鬱の波をなだらかにしていくこと 中級だけどね。治りかけが一番恐いから、気をつけてね
397ご冗談でしょう?名無しさん:2009/10/17(土) 15:53:11 ID:kO/Fil70
鬱の治し方は、薬で穏やかにしていくこと 中級下だけどね。何も知らないよりいいでしょう。これも、治りかけが一番恐いから、気を付けてね。
398ご冗談でしょう?名無しさん:2009/10/17(土) 16:03:49 ID:???
安全で、静かな日常生活が、一番。自然療法も取り入れるといいよ。
399ご冗談でしょう?名無しさん:2009/10/17(土) 16:13:04 ID:???
>>387
知ってますが、使ったことはありません
実際、音のデータを音を超えるスピードで送ることは可能なんでしょうか?
400ご冗談でしょう?名無しさん:2009/10/17(土) 16:16:38 ID:???
>>399
オリンピックの生中継とか見たことないの?
音や映像の信号を電波に乗せて、衛星経由で
0.5秒とかからずに届いている。

音速は340m/sくらいだから、地球の裏側2万キロのところ
からやってくるのに、音なら1分くらいかかるよ。
401ご冗談でしょう?名無しさん:2009/10/17(土) 16:18:37 ID:???
>>400
最後の一行でいろいろだいなしw
402ご冗談でしょう?名無しさん:2009/10/17(土) 16:22:07 ID:???
>>401
千倍みすったw
403ご冗談でしょう?名無しさん:2009/10/17(土) 16:27:35 ID:???
BCASのせいで遅れるのかしら
http://www.asahi.com/special/broadcast/OSK200309230007.html
404ご冗談でしょう?名無しさん:2009/10/17(土) 16:31:43 ID:???
>>395
たしかにおかしいな
405ご冗談でしょう?名無しさん:2009/10/17(土) 17:06:42 ID:???
衝突から次の衝突までの時間の平均 t1 の半分と
衝突するまでの平均時間 t2
なら、ずれる可能性があるね。

長い自由行路上に開始点が落ちる可能性が高いから
t2 > t1/2 かな。(t2 = t1 ?)
406ご冗談でしょう?名無しさん:2009/10/17(土) 17:21:57 ID:???
あくまでも自分の体験からですが、心身の健康のために体を動かす労働はとても良いですよ。
407ご冗談でしょう?名無しさん:2009/10/17(土) 17:25:14 ID:???
>>405の頭が残念
408ご冗談でしょう?名無しさん:2009/10/17(土) 17:28:11 ID:???
働きすぎるのも健康に悪いから、体には気をつけてね。
409ご冗談でしょう?名無しさん:2009/10/17(土) 18:50:02 ID:VAPA+7Ba
シュレディンガー方程式のところで正弦波の式で
 ψ=Nsin(2π/λ)x
っていう式がでますよね。

基本的なところですが、
どうして(2π/λ)になるのか教えてください。
410ご冗談でしょう?名無しさん:2009/10/17(土) 19:04:26 ID:JXBC58zY
sinの周期が2πで、波長の事をλと書くのが慣習だから。
411ご冗談でしょう?名無しさん:2009/10/17(土) 19:14:44 ID:p8xvJt6o
>>410
普通数学とかでsin2πxって感じで表わすじゃないですか?
物理の場合は(2π/λ)と覚えるしかないんですか?
412ご冗談でしょう?名無しさん:2009/10/17(土) 19:43:18 ID:???
(゚Д゚) ハア??
413ご冗談でしょう?名無しさん:2009/10/17(土) 19:49:20 ID:???
まずSchroedinger方程式の解は、

ψ(x) = A sin(kx) + B cos(kx)

だが、境界条件として ψ(0) = ψ(a) = 0 を課すと、 B = 0, ka = nπ が必要になる。結局、

ψ(x) = A sin(nπx/a)

となるが、このとき、 ψ(x + λ) = ψ(x) となる最小のλ、即ち波長を求めると、

nπλ/a = 2π

から、 λ = 2a/n となる。これを用いて ψ(x) を整理すると、 ψ(x) = A sin(2πx/λ) が得られる。
414ご冗談でしょう?名無しさん:2009/10/17(土) 22:49:50 ID:???
>>407
kwsk please.
415405:2009/10/17(土) 22:54:30 ID:???
t1は、衝突から衝突までの時間の平均ね。

t2は、ある粒子の運動の様子を長時間ビデオで撮ったとして、
任意の場所から再生したとしたときに、衝突するまでの時間
の平均。

なぜ t2 > t1/2 が成り立つかというと、再生開始場所は、
自由行程が長いパスの上に落ちる可能性が高いから。

おれ、頭おかしいかな?
416ご冗談でしょう?名無しさん:2009/10/17(土) 23:08:22 ID:???
任意の時刻から と言うより
無作為に選んだ時刻から とかの言い方のほうがいいと思うが

一定間隔で衝突するなら t1 = 2 t2
ポアソン過程なら t1 = t2
実際にはこの中間なんじゃないの
417ご冗談でしょう?名無しさん:2009/10/17(土) 23:11:20 ID:???
>>411
「xがλだけ変化すると位相が2π変化する」ってことを表現してる式だと理解できれば覚える必要はなくなる。
位相部分だけに注目すると2π×(x/λ)と書けるけど、x/λの部分が位相をどのくらいの「割合」で変化させてるか、
と理解できる。
例えば、xの変化がλの半分、つまりλ/2だったら、「割合」x/λは
(λ/2)/λ=1/2=50%
で、位相が半周分、つまり2π×(1/2)=πだけ変化する。
xの変化がちょうどλなら位相は一周分の2πだけ変化する。
λが分母にあることで、λを基本単位としてxがどのくらいの「割合」であるかをうまく表現できている。
418ご冗談でしょう?名無しさん:2009/10/17(土) 23:16:16 ID:???
>>414>>415
なんで

「衝突から衝突までの平均時間」
=「任意の時点から直前の衝突までの時間の平均」+「任意の時点から次の衝突までの時間の平均」

が理解できないのか。

例えがわからないのなら、実生活で「衝突」を「有感地震との遭遇」にでも置換してみ?
任意の時点から「有感地震との遭遇」までの時間の期待値は、「有感地震との遭遇」の間隔の
半分になるだろうが。これは長い間隔があっても一緒。

>>416
> ポアソン過程なら t1 = t2

等しいはずはないと思うが。
419405:2009/10/17(土) 23:49:43 ID:???
>>418
平均の取り方が違うんだ。
君の平均は、時間÷衝突回数でしょ。

もうひとつの平均の取り方がある。
無作為に選び出した時刻に、無作為にある粒子に注目する。
その後、その粒子がはじめて衝突するまでの時間の平均だ。

この場合、衝突までの時間が長い粒子が、無作為に取り出される確率が
高い。つまり、衝突から衝突までの時間を、その時間に比例した重みを
つけて平均しているわけ。

粘性率の計算とか熱伝導率の計算などでは、このような平均が
重要になる。
420ご冗談でしょう?名無しさん:2009/10/18(日) 04:08:53 ID:1znurSrs
統計力学でよくある「自然な変数」とはなんのことでしょうか?
421ご冗談でしょう?名無しさん:2009/10/18(日) 05:34:02 ID:???
SのU,V,NとかGのT,P,Nのことじゃね?
422ご冗談でしょう?名無しさん:2009/10/18(日) 05:45:08 ID:KZQ8NUH0
決定論に関する理解について、ご意見を下さい。

誤解がないように定義をしっかりやります。
決定論と言うと、宗教・道徳的な決定論(全てが決まっているなら何をやっても良い、救われる人かどうかは予め決まっている「予定説」)と、
ラプラスの悪魔に代表される決定論の2つの視点を想起されると思いますが、
ここでは後者に関して述べます。

決定論は不確定性原理と量子力学によって否定されたとすぐに説明する人もいますが、
否定されたのは、あくまで「人間が観測して未来を予測することが不可能である」ことが理論的に否定されているだけであり、
人間が把握できるか否か、予測できるか否かとは関係なく、
あらゆる現象が原因と結果の連鎖である限り、「予め未来は決まっている」ということで正しいでしょうか?

人間が未来を計算によって予測することは不可能であるが、それでも未来は決まっているのである。
その意味での決定論は有効である。ということで良いでしょうか?

仮にその決定論さえも否定されるのであれば、物理法則における因果律がどのような原因によって崩壊するのか指摘できなくてはならないと思います。
423ご冗談でしょう?名無しさん:2009/10/18(日) 05:47:46 ID:KZQ8NUH0
422の7行目を以下の文章に訂正します。

否定されたのは、あくまで「人間が観測して未来を予測することが不可能である」ことだけであり、
424ご冗談でしょう?名無しさん:2009/10/18(日) 06:14:55 ID:???
正しくない。
425ご冗談でしょう?名無しさん:2009/10/18(日) 06:18:24 ID:???
ある現象が「原因と結果の連鎖である」というのはどういうことか
「予め未来は決まっている」というのはどういうことか
物理の言葉に焼き直して(せめてなるべく近付けて)定義して、
さらにその定義の下でちゃんと
>あらゆる現象が原因と結果の連鎖である限り、「予め未来は決まっている」
という主張を導いてくれないことには、量子力学勉強しろとしか言えないね

一から全部説明してやる気にはならない
426ご冗談でしょう?名無しさん:2009/10/18(日) 07:45:32 ID:???
>>422
量子力学の世界じゃ「隠れた変数理論」と呼ばれる決定論だね。
隠れた変数理論が(アインシュタインらにより)どのように提起され、どの様に反証を受け、
因果関係も確率的にしか決まらないという考えが現在の主流となったかは、自分で調べてくれ。(私もそんなに詳しくないし)

最後に当たり前な事だけど、どんな科学理論も現実世界による検証を受ける、
将来 現在主流の量子力学を超えて決定論的理論が支持される可能性は勿論有る、そう考えてる人は少ないけど。
427ご冗談でしょう?名無しさん:2009/10/18(日) 09:45:11 ID:???
かなり決定論が支配してる思われることでも
カオスって発生するよね

428ご冗談でしょう?名無しさん:2009/10/18(日) 10:11:53 ID:???
>>427
そりゃそーだ。完全な決定論でもカオスは発生する。
429ご冗談でしょう?名無しさん:2009/10/18(日) 10:17:38 ID:6xEoN6Xx
カオスであろうが 結局その初期の微小差違の発生の由来は説明できない。

世界は「非決定」ですよ。
430ご冗談でしょう?名無しさん:2009/10/18(日) 10:28:27 ID:???
衛星やスパコン使っても天気予報ですら当たらんからな。
431416:2009/10/18(日) 11:27:25 ID:???
>>418
「衝突から衝突までの平均時間」というのが曖昧なんだけど。
@毎回の衝突を同じ重みとする平均
A無作為に選んだ時刻を含む区間(衝突から次の衝突までの区間)の長さの期待値

@とAは等しいとは限らない。
>>405>>415 の t1 は、文脈からして @の方だと解釈した。
Aは、
無作為に選んだ時刻から次の衝突までの時間の期待値 と
無作為に選んだ時刻から最も近い過去の衝突までの時間の期待値 の和に等しいだろう。
@とAが等しくならない簡単な例としては、ポアソン過程。

>>419
時間/衝突回数 の極限になるのは@の方であって、あなたが t1 と書いたものだと思うが。
432ご冗談でしょう?名無しさん:2009/10/18(日) 11:58:35 ID:???
多次元宇宙論って、ありますか?
433ご冗談でしょう?名無しさん:2009/10/18(日) 12:06:19 ID:???
>>399
国際電話は使ったことなくても、普通の電話 (携帯含む) くらいはつかったことあんだろ。
音速は (気温によるが) 340m/s くらいだから 1km 進むのに約3秒かかる。

10km 離れた場所と通話してて、もし伝達速度が音速を超えられないとしたら、

 こちらから話しかけてそれが相手に届くのに30秒
 相手が返事してそれがこっちに届くのにまた30秒

話しかけて返事が帰ってくるまでに最低で1分かかる。これじゃ使い物にならないな。
そういうものだと思って使えばそれなりに使えるのかもしれないが、現実の電話がそういうもの
ではないことくらいは知ってるだろ。

つか、なんでそれくらいのことを自分で計算してみないのかな?
434ご冗談でしょう?名無しさん:2009/10/18(日) 14:08:56 ID:KZQ8NUH0
みなさんご回答ありがとうございます。

>>429
それは人間が計測することができないと言っているだけじゃないのですか?
人間が計測することができないということしか説明できてないのではないですか?

>>430
人間には予測できないというのは理解していると書いたつもりでしたが。

>>426
私の誤解かもしれませんが、検証するとか反証するというのはあくまでも人間にそれができるか?という問題がありますよね?
その方向では決定論が否定されるという理屈はわかります。
私がお聞きしたいのは、反証主義からみれば私が述べるように人間が反証できないものは「科学」ではないのかもしれませんが、
人間が観測できる検証できるか否かに関わらず、
Aという事象によりBという結果が発生し、Bという結果が原因となってCという結果が発生するという
無限連鎖が続く限り、例えばこの連鎖のどれだけ先の結果であろうとも、Aという事象から辿りつく結果は同じではないでしょうか?
その意味で、世界の結果は予め決定していると考えるのです。

その考えは科学ではないし物理学でもないから回答仕様がないということになるでしょうか?

「Aという事象によりBという結果が発生する」と書きましたが、Bという結果が発生することについて
確率的にしかわからないというのも理解しています。
繰り返しになりますが、その理論もあくまで人間の理解では確率的にしかどうしてもわからないだけであって、
Bという結果が出たことに関して、何らかの理由があったわけで、その理由が何であろうとも、
その理由の先にまたその理由もあるわけで、原因から結果が生まれるという連鎖が破綻しない以上、
未来は決定しているのでは?という考えです。
435ご冗談でしょう?名無しさん:2009/10/18(日) 14:16:49 ID:???
人間の観測は「特別」で、単なる粒子の相互作用は「観測」にならないと主張する輩だよな、お前>>434
436ご冗談でしょう?名無しさん:2009/10/18(日) 15:37:52 ID:???
>>432
あなたの言うその「多次元宇宙論」をどういうものか定義してくれんと、それが有るか無いか言えない。
現在の(割と)主流の考え方では、この宇宙は10次元ぐらいだろうと予想されてるが確定ではない。
437レンガSTATIC:2009/10/18(日) 16:12:18 ID:7RDZQy9c
438ご冗談でしょう?名無しさん:2009/10/18(日) 16:43:27 ID:???
ヒッグス粒子ってそこらじゅうにあるものではないんですか?
ヒッグス粒子に衝突する確率=質量となにかで読んだのですが…
439ご冗談でしょう?名無しさん:2009/10/18(日) 17:02:45 ID:???
バリバリ先入観ありまくりじゃねぇか。

> 決定論に関する理解について、ご意見を下さい。
はどこへ行ったw
440ご冗談でしょう?名無しさん:2009/10/18(日) 17:55:40 ID:???
平均自由行程の話はアシュクロフトの演習問題に載ってた気がするな
441ご冗談でしょう?名無しさん:2009/10/18(日) 18:19:04 ID:???
>>434
だから決定論を物理の言語で定式化しないと物理の文脈で検証or反証が可能かどうか議論にならないっつーの
442ご冗談でしょう?名無しさん:2009/10/18(日) 18:23:06 ID:???
初出で文頭に「だから」を付ける人間は、相手を丸め込もうとする意図がある。
443ご冗談でしょう?名無しさん:2009/10/18(日) 18:36:31 ID:???
だからに気をつけて頑張ってね
444ご冗談でしょう?名無しさん:2009/10/18(日) 19:39:40 ID:???
>>438
ヒッグス粒子とヒッグス場を区別した方がいい

ヒッグス機構による質量の起源は真空(エネルギー的にもっとも低く安定な状態)において
ヒッグス場が非ゼロとなってゲージ対称性を破ることに由来する
粒子はあくまで真空からの励起なので、質量の起源がヒッグス粒子という言い方は正しくない

> ヒッグス粒子に衝突する確率=質量
という認識もおかしい
445ご冗談でしょう?名無しさん:2009/10/18(日) 21:38:41 ID:???
>>434
>Bという結果が出たことに関して、何らかの理由があったわけで

その様な理由が正に「隠れた変数」と呼ばれる物で、それが「有る」というのが隠れた変数理論です。
446ご冗談でしょう?名無しさん:2009/10/18(日) 21:42:43 ID:???
>>434
検証不可能なので、それは科学で議論する命題ではない。
447420:2009/10/18(日) 23:44:20 ID:1znurSrs
>>421
返答ありがとうございます。
448ご冗談でしょう?名無しさん:2009/10/19(月) 00:31:23 ID:???
>>422
Schrodinger 方程式は決定論的方程式だからそういう感じもするんだが
考えるべき要素は多い。

まず、測定を行うと波束の収縮でランダムネスが入るから、それも現象論的に扱うと
stochastic Schrodinger 方程式になってしまう。

しかし密度行列で記述することにするならその運動方程式(Master方程式)はやはり
決定論的になる。どこまでが原理的でどこからが現象論的なのかはよくわからんが。

別の見方として、宇宙全体の波動関数考えるなら測定はないから決定論的(人間も
原子の集まりだから波動関数で記述され、相互作用はあっても観測はない)。
しかしその解釈はわからない。

昔はやった量子宇宙論でその辺もよく話題になったけど、結局定説はできなかった
んじゃないかな。
449ご冗談でしょう?名無しさん:2009/10/19(月) 00:41:04 ID:???
>>434
君、○一教会の人か、○一教会に誘われてる人でしょ。
昔潜入した事有るんだが、理論の穴だらけな感じがまさにそんな感じなんだが。
450ご冗談でしょう?名無しさん:2009/10/19(月) 02:19:40 ID:WUzH8gxY
クラインゴルドン方程式はスピン0の粒子が従う方程式、とあるのですが
スピン0の粒子というのは、具体的にはπ中間子のことであると考えて良いのでしょうか?
また、正準量子化されていないφについて調べても、「古典的なクラインゴルドン場」としか
書いてないものが多いのですが、物理的には何を表しているのか教えてください。
451ご冗談でしょう?名無しさん:2009/10/19(月) 06:09:18 ID:LVwHy9qx
ttp://www5a.biglobe.ne.jp/~neo-neo/banana-scrt.htm

朝など、食欲のない時はバナナを食べるといいですよ。
452ご冗談でしょう?名無しさん:2009/10/19(月) 06:18:02 ID:???
       ┌┐
     ♪ ヽ \
   ♪    i ヽ \
        (゚Д゚ ) | <バナナンバナナンバーナーナ-♪(インフルエンザ対策のためにも)

       ⊂|  υ |
        i  i  |
       ヾ_,ノ _⊃
         ∪"
453ご冗談でしょう?名無しさん:2009/10/19(月) 06:20:17 ID:???
          .┌┐
   ♪     / /
     ♪ ./ / i
       | ( ゚Д゚) <バナナンバナナンバーナーナ-♪
       | ц   |つ
       |  i  i
       ⊂_ヽ_,ゝ
          "∪
454ご冗談でしょう?名無しさん:2009/10/19(月) 08:07:52 ID:yJgUDHw9
449は教会会員ですのでみんなさわってはいけません
455ご冗談でしょう?名無しさん:2009/10/19(月) 13:52:51 ID:???
>>436
ありがとうございます。
456ご冗談でしょう?名無しさん:2009/10/19(月) 13:54:08 ID:???
京都大は、重力船ぐらいつくれよ。OBつれてきてさ
457ご冗談でしょう?名無しさん:2009/10/19(月) 13:56:12 ID:PMk/3e0x
光は波だか粒だかよくわかってないみたいですが光は粒子でそれらが集まって波を形成している、てのはありえますか?
物理初心者がすいません
458ご冗談でしょう?名無しさん:2009/10/19(月) 14:02:11 ID:???
>>457
おお、ワタシも小学4先生の頃同じコト考えてたよ。
奇遇だなあ。
459ご冗談でしょう?名無しさん:2009/10/19(月) 14:02:57 ID:???
typo
×小学4先生
○小学4年生
460ご冗談でしょう?名無しさん:2009/10/19(月) 14:07:36 ID:PMk/3e0x
>>458
ですよねwwww

結局どうなんでしょうか
小学3先生ですが分かりやすくお願いします
461ご冗談でしょう?名無しさん:2009/10/19(月) 14:32:28 ID:???
波の性質と粒子の性質を併せ持つと考えるからそういう誤解が生まれるわけで、
波動粒子性というのはそういう新しい概念としてとらえないといけない。
462ご冗談でしょう?名無しさん:2009/10/19(月) 14:33:37 ID:???
>>450
スピン0なのはπ中間子だけではない。
http://pdg.lbl.gov/
463405:2009/10/19(月) 14:43:39 ID:???
>>450
Higgs粒子もスピンゼロ。
464ご冗談でしょう?名無しさん:2009/10/19(月) 14:51:16 ID:???
>>457
涙か粒子なんだか、よくわかってないんじゃなくて、
波でも粒子でもない、
あるモノ(量子化された電磁場の真空からの励起)だとわかってる。
この辺の理論は、1930年頃に確立したんじゃないかな。

そのあるモノは、状況によって、
古典的な波に見えたり、粒子に見えたりする。
465ご冗談でしょう?名無しさん:2009/10/19(月) 15:21:08 ID:???
>>460
おやすみ
466ご冗談でしょう?名無しさん:2009/10/19(月) 15:21:26 ID:???
量子力学の境界条件で質問です。
x<0、x>Lの領域が無限大のポテンシャルを持っている場合、
境界条件としてψ(x=0)=0,ψ(x=L)=0を採用しますが、ここでゼロにする必然性がわかりません。
相手は無限大のポテンシャルなんだから有限の値なら何でも良いのではないでしょうか

ご回答お待ちしています
467ご冗談でしょう?名無しさん:2009/10/19(月) 15:52:25 ID:???
>>466
エネルギーが有限の状態を求めているから
468ご冗談でしょう?名無しさん:2009/10/19(月) 17:05:59 ID:WUzH8gxY
>>462>>463
ありがとうございます。
量子化していない状態でのクラインゴルドン場は物理的意味を持たないのですか?
スカラー場を量子化すると、φはスピン0の粒子の場と解釈できるようですが
量子化されていないφについては意味を考えてはいけないのでしょうか?
非相対論→相対論→相対論+場の量子化
       ↑
      ここでの場の物理的意味を知りたいです。
469ご冗談でしょう?名無しさん:2009/10/19(月) 17:07:42 ID:???
>>468
「物理的意味」を定義してくれ
470ご冗談でしょう?名無しさん:2009/10/19(月) 17:13:53 ID:???
He4における超流動では全体の1割程度の原子しかBECを起こしていないと聞いたのですが、それは何故でしょうか?
また、超伝導現象において、どの程度の割合の電子がBECを起こしているのでしょうか?
具体的な数式や根拠、概念的な説明などどんなことでもいいのでご教授願います。
471ご冗談でしょう?名無しさん:2009/10/19(月) 17:52:12 ID:???
>>467
エネルギーが有限である以上、無限大のポテンシャルの前ではそれは0とみなせるということでしょうか
472ご冗談でしょう?名無しさん:2009/10/19(月) 18:38:30 ID:???
>>468
古典的な電磁場がどういう扱いだったか思い起こしてみたらいいんじゃね
それとあんま変わらんと思う
473ご冗談でしょう?名無しさん:2009/10/19(月) 19:01:53 ID:???
悪霊て、本当にいるんですね
http://science6.2ch.net/test/read.cgi/kikai/1014172518/
474ご冗談でしょう?名無しさん:2009/10/19(月) 20:09:00 ID:???
>>473
2chにはたくさんいるよ
475ご冗談でしょう?名無しさん:2009/10/19(月) 20:17:44 ID:???
>>468
He4原子の気体とかも、高温で、かつ、電離しないくらいの温度なら
クラインゴルドン方程式に従うと思って計算していいんじゃない?

E = √(p^2+m^2) ≒ m + p^2/(2m) + ...

となるから、非相対論的な方程式との対応もつくし。
負エネルギー解 E = -√ のほうは、手で捨てる必要があるけど。
それがいやなら、量子化するべし。

相対論効果が重要になる温度では、電離してしまうから、
あまり見ないだけなんじゃない?
476ご冗談でしょう?名無しさん:2009/10/19(月) 21:27:32 ID:???
量子化していない場を考えて嬉しいのって実際のところm=0のボゾンくらい?ちがう?
477ご冗談でしょう?名無しさん:2009/10/19(月) 22:21:25 ID:SGZh6ESz
とある大学の授業で床反力計を用いた実験をしています。

床反力計の上に被験者を立たせて垂直飛びをさせます。
その際の力の動きをディスプレイ上でモニターし観察しています。
そのデータをもとに被験者がジャンプしたときの初速を求め、
到達するであろう高さを求める…ということを計算で求めたいのですが、
いかんせん高校3年間生物しかやったことないド文系のものなので
公式等が分かりません。


ちなみに一度測ってみた数値だと、静止時には468Nの力がかかり、
飛び立つ瞬間には927Nの力がかかったため、加速度は9.6m/s^2だと計算できました。
※F=mgの式により被験者の体重を割り出し、F=mg+maの式を変形してma=F-mgとし、
a=(F-mg)/mとしました。


先生に質問したところ、飛び立つ瞬間から0.01秒ごとに(0.01秒単位でモニターできる機器だそうです)逐一計算するのだとか…。
「数値をエクセルに入れて関数組んだらスイッチポンやでwww」
とおっしゃっていましたがその積分をもちいた式がさっぱりです。

478ご冗談でしょう?名無しさん:2009/10/19(月) 22:23:00 ID:SGZh6ESz
>>477 ながすぎたらしく分割します。続きです。

例として
静止した状態の時間をt1とおき、その時の速さをV1とする。
t1=V1=0m/sec
0.01秒後の加速度a1を0.1m/sec^2とする。そのときをt2とし、速さをV2とする
t2のとき V2=V1+0.1m/sec^2*0.01=0.001m/sec
t3のとき a2=1m/sec^2になったとする
V3=V2+1m/sec^2*0.01=0.01m/sec

という式を示して頂いたのですが、これをどのように垂直飛びの式に当てはめていけばようのでしょうか。
上記の例題中にでてくる加速度は最初に書いたようにその瞬間の床にかかった力から逐一求めていかなければならないのでしょうか。

さらに、この式をエクセルで計算する場合はどのような命令を組めばよいのでしょうか
(スイッチポンになるやり方もできるのならば教えていただきたいです。)


提示する数値、内容が少なすぎるかもしれませんが、ここから解決の方法をご教授願えませんか?
長文失礼しました。よろしくお願いします。
479ご冗談でしょう?名無しさん:2009/10/19(月) 22:25:24 ID:???
できるのならとりあえずグラフうp
480ご冗談でしょう?名無しさん:2009/10/19(月) 22:37:28 ID:SGZh6ESz
>>479
印字していないので実物は用意できませんが、こんな感じという手描きのグラフではダメでしょうか。
481ご冗談でしょう?名無しさん:2009/10/19(月) 22:41:43 ID:???
どなたか>>470についてお願いします。
482ご冗談でしょう?名無しさん:2009/10/19(月) 22:42:55 ID:SGZh6ESz
http://imepita.jp/20091019/816520

こちらになります。
x軸が床にかかる力(N)、y軸が時間(sec)です
483ご冗談でしょう?名無しさん:2009/10/19(月) 22:44:24 ID:???
>>480
まあ何かグラフあったほうが説明しやすいと思っただけなのでなくてもいいです

力積を計算すればいいと思うけどそれでわかるかな
484ご冗談でしょう?名無しさん:2009/10/19(月) 22:53:10 ID:???
>>482
これ着地も含んでるね、まあ検証に使えるからいいか。
確認だけど、初速は足が床から離れた時点の重心の速度でいいのよね?
485ご冗談でしょう?名無しさん:2009/10/19(月) 22:54:18 ID:SGZh6ESz
力積を使うということは先生もおっしゃっていたのでわかるのですが、
どう使えばいいのかさっぱりなんです・・・
486ご冗談でしょう?名無しさん:2009/10/19(月) 22:57:28 ID:SGZh6ESz
>>484
データは着地までとりましたが計算はテイクオフの瞬間まででいいようです。
初速もそれでOKなはずです。
487ご冗談でしょう?名無しさん:2009/10/19(月) 22:57:29 ID:j0GR2MIA
大学で量子力学の基礎にあたる講義を受けいています。
この間出された課題がどうしてもわかりません。
 @ f(Xn)=pe^qn n=0,1,2,3,・・・・   p,qは定数
 A Σ(lnf(Xn)+1)d/dx f(Xn)

@をAの左辺に代入して0になることを示せ。
488ご冗談でしょう?名無しさん:2009/10/19(月) 22:57:52 ID:WUzH8gxY
>>472>>475
返信遅くなり申し訳ありません。
ありがとうございました。
489ご冗談でしょう?名無しさん:2009/10/19(月) 23:01:59 ID:???
http://www.dotup.org/uploda/www.dotup.org280635.jpg
この範囲の積分をして、Tを求める。

そうすれば mv0 = T - mgΔs なんじゃね。
490ご冗談でしょう?名無しさん:2009/10/19(月) 23:12:40 ID:???
>>486
こっちも図が下手でゴメン
t1が重心が静止した状態、t2が床から足が離れた瞬間ね
グラフの青く塗った領域の面積が、この間に床に加えた力積になる。
1g下での運動なので、この力積からmg(t2-t1)を引いた物が人の運動量変化(まあぶっちゃけm*初速)と等しい。
491ご冗談でしょう?名無しさん:2009/10/19(月) 23:14:35 ID:6zKduDEA
>>489
>>477です。諸事情により携帯から書き込みます

厚かましいようですが物理に関しては門外漢なので式の記号について解説いただければ幸いです



知りたいことを簡潔に書くと、
・テイクオフ時の初速の求め方
・それをもとにジャンプの最高到達点を求める方法
です。
お手軽かけますがよろしくお願いします。
492ご冗談でしょう?名無しさん:2009/10/19(月) 23:18:56 ID:???
>>481
ニ流体モデルとか
493490:2009/10/19(月) 23:22:55 ID:???
494ご冗談でしょう?名無しさん:2009/10/19(月) 23:24:54 ID:???
>>487
d/dx f(Xn)
xがないからどう微分していいかわからん
495490:2009/10/19(月) 23:26:18 ID:???
>>489
ん、それだと求める初速になんないんじゃね?
496ご冗談でしょう?名無しさん:2009/10/19(月) 23:29:43 ID:6zKduDEA
>>490
ありがとうございます
力積ですが、どんな数値があれば求められそうですか?
497490:2009/10/20(火) 00:00:01 ID:???
>>491
最高到達点も・・・これも重心位置でいいんだろうな?
でもきちんと出すには、ちゃんとグラフの積分しないといけないけど出来るか?
初速だけからでは正確な重心位置とはいえないはずなのに。でも「それをもとに」って書いてあるし、
この点指摘したら評価されるかも、そんな事ないか。

>>496
だからあんたのグラフから求めろと・・
498ご冗談でしょう?名無しさん:2009/10/20(火) 00:00:02 ID:???
どなたか>>471お願いします
499ご冗談でしょう?名無しさん:2009/10/20(火) 00:01:57 ID:???
>>498
有限井戸の解求めて極限とってみればいいじゃん
500490:2009/10/20(火) 00:03:09 ID:???
>>497の前段についてだけど
最初の静止状態と足が離れる時点での重心位置が変わらないと言う仮定を付ければ、
初速だけから重心の最高到達点を求めても大丈夫になるな。
人がジャンプする場合を考えると、そんなに悪くない仮定だし。
501ご冗談でしょう?名無しさん:2009/10/20(火) 00:09:47 ID:???
>>499
ああ、なるほど。
ありがとうございました
502ご冗談でしょう?名無しさん:2009/10/20(火) 00:11:13 ID:2hM/lcL8
みなさんありがとうございます

力積に関してはこちらも数値を提示していないので申し訳ないことになってますね…
これくらい自分でやらなきゃいけませんね

ちなみに授業で使うデータなので仮定や条件などは厳密すぎなくても大丈夫なようです
503ご冗談でしょう?名無しさん:2009/10/20(火) 06:59:50 ID:???
これって、どのくらいの出力?軍事転用できる
http://tsushima.2ch.net/test/read.cgi/newsplus/1255889581/l50
504ご冗談でしょう?名無しさん:2009/10/20(火) 07:06:29 ID:???
転用できるようだったら、鹿島に電話して、実用化させといて。佐藤研もお手伝いして
505ご冗談でしょう?名無しさん:2009/10/20(火) 07:09:17 ID:???
核を超える兵器があれば
非核三原則を守りつつ核抑止力とする事が出来る
506ご冗談でしょう?名無しさん:2009/10/20(火) 07:28:09 ID:FyFkxXUI
つまり、レーザーでブッラクホールが出来るの?
ブラックホールが出来ると、危ないんでしょ?
地球アポンになったら、どうするの?
非黒穴三原則を造って、やんちゃな研究者を縛った方がよくない?
507ご冗談でしょう?名無しさん:2009/10/20(火) 08:07:50 ID:???
つまり、メタマテリアルでブッラクホールが出来るの?
ブラックホールが出来ると、危ないんでしょ?
地球アポンになったら、どうするの?
非異物三原則を造って、やんちゃな研究者を縛った方がよくない?
508ご冗談でしょう?名無しさん:2009/10/20(火) 08:11:37 ID:???
>>505
核じゃないなら非核抑止力だろ、バカじゃね?
509ご冗談でしょう?名無しさん:2009/10/20(火) 08:13:06 ID:???
>>506-507
とっとと≫503のスレへ逝け、カス
二度書き込むな、ウジ虫
510ご冗談でしょう?名無しさん:2009/10/20(火) 08:14:55 ID:???
レーザーで、核汚染消える?消えるなら教えて
511ご冗談でしょう?名無しさん:2009/10/20(火) 08:15:27 ID:???
ブラックホールといやあバカが大騒ぎするから、ってだけのこと。
なんでもないよ

蛍光灯を光らせておくと消してもガラス管がぼやっとさせられることを実現!
すごい実験に成功した!! 
ブラックホール周辺で見られる現象と同じだっっっ!!!

大阪はバカだからねえ。。。。
512ご冗談でしょう?名無しさん:2009/10/20(火) 08:16:02 ID:FyFkxXUI
>>509
やんちゃで幼稚な危ない研究者君ですか?
513ご冗談でしょう?名無しさん:2009/10/20(火) 08:17:34 ID:???
>>512
やんちゃで幼稚な危ない小学生君ですか?
514ご冗談でしょう?名無しさん:2009/10/20(火) 08:21:08 ID:???
>>512
文系のバカ記者が書いた飛ばし記事に釣られてオタオタしてる
ゆとりクンの姿が見えるぜ。
515ご冗談でしょう?名無しさん:2009/10/20(火) 08:26:51 ID:???
>>512
二度と書き込むなというのも理解できんのか。
さすがに日本語も理解できん底辺バクテリアはひと味違う。

とっとと出てけ。
516ご冗談でしょう?名無しさん:2009/10/20(火) 08:31:29 ID:FyFkxXUI
何か気に触ったのかな? まさか図星だったの?
517ご冗談でしょう?名無しさん:2009/10/20(火) 08:53:18 ID:???
ホントの研究者ならバカをからかう時間なんかないだろ
518ご冗談でしょう?名無しさん:2009/10/20(火) 10:14:29 ID:???
地球人ごときがBH作ろうなんてまだ1世紀は無理だろjk
519ご冗談でしょう?名無しさん:2009/10/20(火) 10:32:51 ID:v33J/DCR
東大理系も、ブラックホール創っただろが。
520ご冗談でしょう?名無しさん:2009/10/20(火) 10:45:00 ID:v33J/DCR
ブラックホールに、核汚染物質入れればいいね。大ボケしました
521ご冗談でしょう?名無しさん:2009/10/20(火) 10:51:04 ID:???
使い捨てウランも、ポイ捨てね
522ご冗談でしょう?名無しさん:2009/10/20(火) 11:22:02 ID:v33J/DCR
でも。星系のバランス考えないとね。人の欲には限りないから。
523ご冗談でしょう?名無しさん:2009/10/20(火) 11:31:30 ID:v33J/DCR
今の高校2年生 最強ゆとりの場合は、研究施設や施設を閉鎖しないとね!!
524ご冗談でしょう?名無しさん:2009/10/20(火) 12:06:02 ID:???
>>508
核に対する抑止だろ
馬鹿じゃね
525ご冗談でしょう?名無しさん:2009/10/20(火) 12:17:05 ID:???
>>524
「『核抑止力』を理解していません」まで読んだ
526ご冗談でしょう?名無しさん:2009/10/20(火) 14:10:57 ID:???
核抑止、または核抑止力には2つの意味がある。ひとつは国家間の戦争を抑止するというものであり、もうひとつは核兵器の使用を抑制するというものである。
527ご冗談でしょう?名無しさん:2009/10/20(火) 17:20:33 ID:LdQmETwh
質問です。
ある物質の質量が同じ大きさの体積の水より重い場合、その物質を水に落としたとき、必ず沈みますよね?
528ご冗談でしょう?名無しさん:2009/10/20(火) 17:24:50 ID:???
別に。表面張力とかで浮けるし。
529ご冗談でしょう?名無しさん:2009/10/20(火) 17:42:34 ID:???
お椀や手こぎボートのような形状でも浮けるし。
530ご冗談でしょう?名無しさん:2009/10/20(火) 17:42:40 ID:???
水に浮かぶ一円玉とか
531ご冗談でしょう?名無しさん:2009/10/20(火) 17:55:37 ID:LdQmETwh
言い方を間違えました。

その物質を水に沈めた場合、浮き上がることはないですよね?

こっちでお願いします。
532ご冗談でしょう?名無しさん:2009/10/20(火) 18:06:48 ID:???
上昇する水流があれば浮き上がる
533ご冗談でしょう?名無しさん:2009/10/20(火) 18:08:10 ID:fCcwxmkJ
場の量子化に出てくる計算についての質問です。
φ=∫dk(a(k)e^{ikx})がエルミート演算子であるとき、
a(k)=a(-k)^†が成り立つ。というのを自分で計算してみると
φ^†=∫dk(a(k)^{†}e^{-ikx})=-∫dk'(a(-k')^{†}e^{ik'x}) (k=-k'と変換)
  =-∫dk(a(-k)^{†}e^{ikx})=φ=∫dk(a(k)e^{ikx})
となって、マイナスが出てきてしまうのですが、どこが間違っているのか教えてください。
534ご冗談でしょう?名無しさん:2009/10/20(火) 18:12:07 ID:???
バルサ材で作ったライトプレーンでも、日本海溝に沈めると二度と浮かばないんだぜ。
535ご冗談でしょう?名無しさん:2009/10/20(火) 18:17:08 ID:???
>>533
積分区間は?
536ご冗談でしょう?名無しさん:2009/10/20(火) 18:48:17 ID:???
>>525
ねぇ
>>526は読めた?
537ご冗談でしょう?名無しさん:2009/10/20(火) 19:18:18 ID:???
「Wikipediaから一言一句違わずコピペしてきました」まで読んだ。
538ご冗談でしょう?名無しさん:2009/10/20(火) 19:41:59 ID:???
なんの反論にもなってないな
それともWikipediaのこの文が間違ってるの?
539ご冗談でしょう?名無しさん:2009/10/20(火) 20:33:51 ID:???
ねえ、ウィキペのパクリを貼り付けたり、自分の主張を途中で変えたりして
恥ずかしくないの?
540ご冗談でしょう?名無しさん:2009/10/20(火) 20:38:24 ID:???
>>538
>>524とWikipediaの記述は食い違ってるな。
風呂敷広げるなよ。
541ご冗談でしょう?名無しさん:2009/10/20(火) 20:40:00 ID:???
質問かと思ったら何やってんだよw
542ご冗談でしょう?名無しさん:2009/10/20(火) 20:42:01 ID:???
次の質問どうぞ
543ご冗談でしょう?名無しさん:2009/10/20(火) 20:53:38 ID:???
どこが自分の主張変わってるんだよ
どこの記述が食い違ってるんだよ
核の使用をなるべく出来無くする
通常兵器も強力ならば核抑止になる

非核抑止って何?
馬鹿じゃね
544ご冗談でしょう?名無しさん:2009/10/20(火) 21:24:48 ID:???
「核を超える兵器があれば」という前提の話で核を抑止して何の意味があると思っているのだろう
頭悪すぎ
545ご冗談でしょう?名無しさん:2009/10/20(火) 21:39:54 ID:???
スレチなネタで必死になるなよ

次の質問どうぞ
546ご冗談でしょう?名無しさん:2009/10/20(火) 22:24:53 ID:???
随分な
論点ずらしだな
で何処が間違ってるの?
核抑止力に成る事には反論は無いの?

非核抑止力ってなんなの?
核抑止力を
核に対する抑止力なのに
核で行う抑止と思い込んでいただけなんだろ
547ご冗談でしょう?名無しさん:2009/10/20(火) 22:35:43 ID:???
よそでやれ
548ご冗談でしょう?名無しさん:2009/10/20(火) 23:05:00 ID:???
妬み野郎が多いねここは。
それに自分は平均よりも出来ると思い込んでいる人が大半だな
549ご冗談でしょう?名無しさん:2009/10/21(水) 00:58:43 ID:???
NG 非核 抑止
550ご冗談でしょう?名無しさん:2009/10/21(水) 01:29:58 ID:yFmlb26j
>>535
-∞から∞です。
だから符合がひっくり返るんですね。解決しました、ありがとうございました。
551ご冗談でしょう?名無しさん:2009/10/21(水) 01:58:34 ID:???
20世紀の天才物理学者達が理論のみを追求し、それによる人体影響などを知りたいが為に核実験に関わったのは残念だ
兵器に使われず、その危うさを抑制するような装置が研究されていれば、
すでに石油変わる次世代エネルギーになっていたかもしれんのにな
552ご冗談でしょう?名無しさん:2009/10/21(水) 02:06:35 ID:???
原爆に使われたウラン・プルトニウムを濃縮するために作られたのが、今でいうところの
「原子炉」。

分かるかな〜、分かんねぇだろうな〜(笑)。
553ご冗談でしょう?名無しさん:2009/10/21(水) 02:12:12 ID:???
兵器に使われたから、未だ石油に頼って原子炉が普及してねーんだよなあ

分かるかな〜、分かんねぇだろうな〜(笑)。
554ご冗談でしょう?名無しさん:2009/10/21(水) 02:18:11 ID:???
>>552
そらわからんわ、間違いだもんな。
555ご冗談でしょう?名無しさん:2009/10/21(水) 02:20:18 ID:???
核分裂どころか保存則すら全く意味が分かってない>>552みたいなアホはほっとけよw
556ご冗談でしょう?名無しさん:2009/10/21(水) 02:25:16 ID:???
>>552
原爆に使われたウラン・プルトニウムを濃縮するために作られたのが、今でいうところの
「原子炉」。

は?w
557ご冗談でしょう?名無しさん:2009/10/21(水) 02:39:16 ID:???
もともと核の使い道は、原子炉だけに使われる為だけでよかったのに
兵器として使われてしまったが為に、エネルギー供給資源として普及が進んでいないのが現状
あと、原子炉には制御棒が付いてて中性子数を調整し、臨界状態にならないようにしてるだけ
558ご冗談でしょう?名無しさん:2009/10/21(水) 02:54:12 ID:???
>>557
臨界にならないんだったら、原発はどうやって発電してんのかな?
559ご冗談でしょう?名無しさん:2009/10/21(水) 03:01:40 ID:???
「原子力は平和利用に限るべきだ」と言う主張だけならまだいいんだ、賛同する人もいるだろう。
(ただこのスレで主張すんのは、明らかにスレ違いだ)
でも言ってる事が間違いだらけじゃ、むしろその主張の足を引っ張るだけじゃないかな。
560ご冗談でしょう?名無しさん:2009/10/21(水) 03:05:47 ID:???
>>558
更に3行ほど書きたかったが、うざい池沼が沸くので消しただけw
1行目
臨界状態から上を制御し、電力として供給している
561ご冗談でしょう?名無しさん:2009/10/21(水) 03:09:28 ID:???
>>559
おまえが全部間違ってるしなww
俺は平和利用じゃなく、石油に代わる新しいエネルギーとして有効利用できたと言ってるんだ
兵器として使われてなければ、平和も戦争もない
562ご冗談でしょう?名無しさん:2009/10/21(水) 03:26:26 ID:???
>>552
馬鹿かこいつ

つーかこんなアホな考え奴に反論できず、
小学生並み以下の考えに踊らされてる2ちゃんが怖いわw
563ご冗談でしょう?名無しさん:2009/10/21(水) 03:32:45 ID:oHx3Rg5d
>>560
池沼って何ですか?
564ご冗談でしょう?名無しさん:2009/10/21(水) 03:33:29 ID:???
原爆さえなければ、核融合研究も進んでる
565ご冗談でしょう?名無しさん:2009/10/21(水) 03:34:36 ID:???
>>563
VIPで聞いて下さい
566ご冗談でしょう?名無しさん:2009/10/21(水) 03:42:59 ID:???
>>560-561
下手に取り繕っても、君の原子力の知識が誤りばかりなのは隠しようが無い。
567ご冗談でしょう?名無しさん:2009/10/21(水) 03:44:21 ID:???
568ご冗談でしょう?名無しさん:2009/10/21(水) 03:44:51 ID:???
だからこの手の馬鹿を相手にするなよ、
ここはまともな質問を相手にするスレだ。
569ご冗談でしょう?名無しさん:2009/10/21(水) 03:48:10 ID:???
>>568
アンカー付けず自演失敗だなw

反論するからには何が違うのか書けよ?低脳
別に俺は逃げないし、俺はおまえのアホ差がどれだけアホか知りたいだけ
570ご冗談でしょう?名無しさん:2009/10/21(水) 03:51:28 ID:???
>>552
原爆に使われたウラン・プルトニウムを濃縮するために作られたのが、今でいうところの
「原子炉」。
笑うわwwwwwwwwwwwwwwwwwwwwwwwww

はやく反論書けよ低脳君w
571ご冗談でしょう?名無しさん:2009/10/21(水) 04:04:39 ID:???
>>566
まともに反論できないおまえの知的レベルの低さは隠しようが無い。
572ご冗談でしょう?名無しさん:2009/10/21(水) 06:47:35 ID:???
>>568
>別に俺は逃げないし、

ならばあなたの住所、氏名、年齢、職業、電話番号をお書きくださいw
低脳さまww
573ご冗談でしょう?名無しさん:2009/10/21(水) 06:54:35 ID:???
夏休みでも始まったのか?
574ご冗談でしょう?名無しさん:2009/10/21(水) 07:19:29 ID:GLTWrxpr
世界トップレベルの頭脳が結集している物理板で、喧嘩はダメよ!!
575ご冗談でしょう?名無しさん:2009/10/21(水) 07:20:40 ID:???
>>552が言ってんのはシカゴパイルのことだべなぁ。

UCLAかどっかの大学のフットボール場の地下に作ったんだっけ?
576ご冗談でしょう?名無しさん:2009/10/21(水) 07:42:12 ID:???
>>555
一番最初の兵器級の核燃料はどうやって作ったの?
577ご冗談でしょう?名無しさん:2009/10/21(水) 08:03:39 ID:???
572を書いた者ですがアンカを間違えました。

>>568 ではなく >>569 でした!

568様の意向反した書き込みで尚且つ568様にご迷惑を
おかけしました。申し訳ございません。
578ご冗談でしょう?名無しさん:2009/10/21(水) 08:25:59 ID:???
そもそも>>505の「核を超える兵器があれば」という妄想から始まった話で、
どれだけスレを消費しようというのか。
579ご冗談でしょう?名無しさん:2009/10/21(水) 10:58:23 ID:???
数学、物理、シュミレーション、その他、調和して 進歩すると良い。
580ご冗談でしょう?名無しさん:2009/10/21(水) 10:59:04 ID:GLTWrxpr
三歩さがって、2歩あがる。
581ご冗談でしょう?名無しさん:2009/10/21(水) 11:00:27 ID:???
なかなか調和はとれんのよね
582ご冗談でしょう?名無しさん:2009/10/21(水) 12:13:11 ID:???
三歩さがって、2歩あがる タンタンとタンタンと。
583ご冗談でしょう?名無しさん:2009/10/21(水) 12:15:42 ID:???
この板でよく見るメコスジって何ですか?
メゾスコピック系の事?
584ご冗談でしょう?名無しさん:2009/10/21(水) 12:29:36 ID:???
メソスコピックって読まねえ?
585ご冗談でしょう?名無しさん:2009/10/21(水) 12:45:09 ID:???
>>583
そうです
研究に行き詰まって発狂してるんです
そっとしておいてやって下さい
586ご冗談でしょう?名無しさん:2009/10/21(水) 13:19:33 ID:???
物質の形成過程に、エネルギーはどう関わっているか教えてください。
587ご冗談でしょう?名無しさん:2009/10/21(水) 14:37:30 ID:???
磁力の伝達速度って光と同じですか?
588ご冗談でしょう?名無しさん:2009/10/21(水) 15:14:37 ID:???
>>587
そうです。
589ご冗談でしょう?名無しさん:2009/10/21(水) 15:15:48 ID:???
>>586
物質の形成過程ってなにを指しているの?
もっと具体的に。
590ご冗談でしょう?名無しさん:2009/10/21(水) 21:44:11 ID:???
じゃあ、水素でいいや。
591ご冗談でしょう?名無しさん:2009/10/21(水) 21:46:16 ID:j9VxnmMa
将棋の手数は数え上げられるかな?
592ご冗談でしょう?名無しさん:2009/10/21(水) 21:55:31 ID:???
場の量子論の本で、演算子を固有関数系でフーリエ展開するというのが
説明無しに突然出てきて困っています
他の本を見ても、「古典論との類推により〜」と書いてあるだけで、
こういう事が可能であるという事について特に説明していないようです
展開は、 Σ展開係数(演算子)×固有関数 という形になっていますが、
この演算子は、固有関数に対して作用しているのでしょうか?
計算の途中で 固有関数×演算子 と順序を変えたりしているように見えるのですが…
宜しくお願いします
593ご冗談でしょう?名無しさん:2009/10/21(水) 22:09:54 ID:???
>>590
水素か。

エネルギーって広すぎるんだよね。あらゆるものに関係するし。
エネルギーが関わるって、
どういう説明を期待しているの?

他の現象でエネルギーが関わっている具体例を挙げてみて。
594ご冗談でしょう?名無しさん:2009/10/21(水) 22:13:55 ID:???
>>592
演算子が作用するのはヒルベルト空間(フォックスペース)
であって、時空(R^4)上の複素数値関数の空間とか
時空上のベクトル束が作る空間ではない。

フォックスペースに作用する演算子が、時空の各点にのっているだけ。
595ご冗談でしょう?名無しさん:2009/10/21(水) 22:43:28 ID:???
( ´・∀・`)へー
596ご冗談でしょう?名無しさん:2009/10/21(水) 23:20:14 ID:???
レス有り難う御座います
作用する空間が違うので演算子と固有関数の順序は交換しても構わないという事でしょうか
しかし演算子のフーリエ展開において、展開係数となる演算子がどの空間に作用するものでなければならないかについても
何も書いていないのですが、展開前の演算子と同じ空間という事になるのでしょうか?
演算子の展開についてはフーリエ解析の本等にも全く載っておらず、本当にこういう事が可能なのか、
どういうルールがあるのか全く分からないのですが、個別に扱ってる分野があるのでしょうか
597ご冗談でしょう?名無しさん:2009/10/21(水) 23:37:27 ID:???
>>596
調和振動子の量子力学の行列表示はやったの?
598ご冗談でしょう?名無しさん:2009/10/22(木) 00:41:50 ID:???
多分、固有関数って言ってるのは空間依存性を持つ係数部分で
場の演算子の一部だな
599ご冗談でしょう?名無しさん:2009/10/22(木) 10:58:13 ID:???
>>588
サンキュー
600ご冗談でしょう?名無しさん:2009/10/22(木) 13:14:12 ID:zm/60VyZ
細い棒がその中心軸の周りで自転しているとします。
この場合、棒の軸が自転軸からわずかでもズレていると、棒には遠心力が働いて
ズレが一層大きくなり、それがさらに遠心力を増大させる・・・という悪循環?が
つづくので棒は遅かれ早かれ、自転軸に対して直角方向を向くようになると思われる
のですが、この考え方は正しいのでしょうか?
それが正しいとした場合、そのような棒の運動は時間の経過に対して可逆的なのでしょうか?
つまり棒の運動に適当な初期状態を与えてやれば、初めに回転軸に直角方向を向いていた
棒の軸が、徐々に向きを変えていって、やがて自転軸方向を向くというのようなことは力学的
に起こりえるのでしょうか?
ニュートン力学の運動方程式が時間反転に対して対称であることから考えると、起こり得そうな
気がしますが。
601ご冗談でしょう?名無しさん:2009/10/22(木) 13:26:06 ID:???
>>600
正しくない
602ご冗談でしょう?名無しさん:2009/10/22(木) 13:31:16 ID:zm/60VyZ
しかし棒に遠心力が働けば、棒の軸は直角方向に向きをかえるのではないですか?
603ご冗談でしょう?名無しさん:2009/10/22(木) 13:36:23 ID:???
家電板の、電気工事屋さんが答えます 第二号柱
http://gimpo.2ch.net/test/read.cgi/kaden/1243483952/
っていうスレで、コンセントのトラッキング現象から出た話で

45 名前:目のつけ所が名無しさん[sage] 投稿日:2009/06/06(土) 16:36:49
電流は抵抗の少ない所へ集中する。
それだけ。

みたいなレスがあるんだけど物理板的にどうよ?
604ご冗談でしょう?名無しさん:2009/10/22(木) 13:36:42 ID:S8WCdYLT
>>600
>のですが、この考え方は正しいのでしょうか?
経験的には正しい。
理論的には間違っている。
605ご冗談でしょう?名無しさん:2009/10/22(木) 13:41:08 ID:???
>>600
これはおわんにビー玉を入れとくとやがては真中に落ち着く
というのと本質的には同じじゃないか
摩擦が大きければ素直に真中に落ち着くけど
摩擦が小さければいったり来たりしてなかなか落ち着かない
摩擦がなければ位置エネルギーと運動エネルギーが入れ替わるだけで延々と続く
(この辺は運動方程式を立てて積分してみてほしい
減衰振動ってやつだ)

真中にあるビー玉を上に上げるのは摩擦が小さければ簡単だけど
また落っこちてくるだろう
(おわんだと上に平衡点がないけど不安定な平衡点ならほとんどの場合
たどり着いても通り過ぎるだけだろう)
摩擦が大きいとすぐに力尽きてまた真中に戻り出す

素直に直角方向を向くように見える場合は摩擦が大きい場合に相当するわけで
逆向きに動かすにはあらかじめ大きなエネルギーを与えないといけないだろう
摩擦(分子運動)にエネルギーを奪われるわけだから
606ご冗談でしょう?名無しさん:2009/10/22(木) 13:55:52 ID:???
>>600
この場合には、対称軸と自転軸は同じずれ角を維持したまま
対称軸が自転軸のまわりをぐるぐる回ると思う。
歳差運動でぐぐってみ?
地球の場合には、周期10か月の歳差運動があって、自転軸は
北極点(南極点)の周りを10メートルくらいのずれで動いているそうだ。
直観的な説明はおれにはよくわからん。

一般の剛体では、慣性モーメントの主軸のうち、
最大固有値と最小固有値に対応する軸からの回転軸のちょっとしたずれは
上のように安定。中間の固有値に対応する軸からの回転軸のずれは不安定
(激しく大きくなったり小さくなったりする)だそうです。
ゴールドスタインの第5章とか読んでみたら?
607ご冗談でしょう?名無しさん:2009/10/22(木) 13:58:08 ID:???
>>600
棒の慣性テンソルの主軸の慣性モーメントは3つ。そのうち最大か最小の軸のまわりの回転は安定。
608ご冗談でしょう?名無しさん:2009/10/22(木) 14:00:22 ID:???
>>602
遠心力という言葉で何を指しているのか、よくわからんが、
回転軸を倒そうとしたら、回転軸は力と直角方向にずれる。
これは回転軸をぐるぐる回す(歳差運動)わけで
回転軸が倒れるわけではない。
609ご冗談でしょう?名無しさん:2009/10/22(木) 14:00:27 ID:???
>>606
歳差運動は外力がかかり続ける場合。
>>600は初期の擾乱以外、自由運動を想定していると思われる。
610ご冗談でしょう?名無しさん:2009/10/22(木) 14:02:04 ID:zm/60VyZ
>>605
>これはおわんにビー玉を入れとくとやがては真中に落ち着く
というのと本質的には同じじゃないか

なるほど。
結局、摩擦がなければ、棒の軸は回転軸に収束したり離れたりを一定周期で繰り返すわけですね。
剛体の回転運動というのはなかなか複雑であることがわかりました。
ありがとうございます。
611ご冗談でしょう?名無しさん:2009/10/22(木) 14:04:26 ID:???
>>609
外力がない歳差運動もある。
612ご冗談でしょう?名無しさん:2009/10/22(木) 16:39:24 ID:???
放物運動で、解答を見てもわからなかったのですが、「・」とはどう計算すればいいのでしょうか
613ご冗談でしょう?名無しさん:2009/10/22(木) 17:04:41 ID:bOEa65PE
ポッカのキレートと牛乳1パックとで、80%能力が戻るよ。夏場暑かったから。
614ご冗談でしょう?名無しさん:2009/10/22(木) 17:10:08 ID:???
>>612
ただの積か
ベクトルどうしの内積のどっちか。
615ご冗談でしょう?名無しさん:2009/10/22(木) 17:29:40 ID:???
ごめんなさい、ここ最近やる気が出ずに勉強面で努力できていません。
期待させるようなことばっかり言って申し訳なかったです。
完全に放棄はしていませんが、前のように勉強自体に無我夢中になれない感じです。

あなたもお仕事大変だと思いますが、体には十分気をつけて無理せずに自分のペースで
続けられるように頑張ってくださいね。

616ご冗談でしょう?名無しさん:2009/10/22(木) 18:24:18 ID:???
でも、その代わりというのもなんですが、精神的に冷静さを取り戻す
ことができました。
休憩を挟んだら、また始めます。

だから心配はしないでください。
風邪には気をつけて、無理しないでくださいね。
617ご冗談でしょう?名無しさん:2009/10/22(木) 18:36:24 ID:???
>>615, 616
618ご冗談でしょう?名無しさん:2009/10/22(木) 18:52:05 ID:???
なにか違うモノが見えてるんでしょ
619ご冗談でしょう?名無しさん:2009/10/22(木) 20:18:22 ID:???
物理で行列計算が役に立つor立っている分野ってありますか?
620ご冗談でしょう?名無しさん:2009/10/22(木) 20:22:03 ID:kuQWbgtK
>>619 殆ど全て。とりわけ量子力学は行列力学というくらいだから行列
計算が基本。要は対角化できたもん勝ち。
621619:2009/10/22(木) 20:30:31 ID:???
>>620
回答ありがとうございます。

利用される理由はツールとして行列を使った方が簡単だからということでしょうか?
それとも本質的に行列でないと解けない等の理由で利用されているんですか?
622ご冗談でしょう?名無しさん:2009/10/22(木) 20:36:56 ID:???
「本質的に」と「ツールとして」の違いが分からん
623ご冗談でしょう?名無しさん:2009/10/22(木) 20:40:43 ID:kuQWbgtK
>>621 本質的だよ。量子力学の場合はシュレディンガー方程式にマップできるけど。
624619:2009/10/22(木) 20:45:46 ID:???
>>623
なるほど、勉強になりました。
ありがとうございました。
625ご冗談でしょう?名無しさん:2009/10/22(木) 21:36:41 ID:kUQpdXq+
材料力学の範疇だと思いますが、自緊処理って何でしょうか。
626ご冗談でしょう?名無しさん:2009/10/22(木) 23:06:51 ID:???
摂動論を含む量子力学の演習書って何が良いでしょうか?
問題と解答があると嬉しいです
和書と洋書のそれぞれについてお願いします
627592=596:2009/10/23(金) 00:52:00 ID:???
>>597
生成消滅演算子の行列表示等はやりましたが…
調和振動子の行列表現で演算子のフーリエ展開が出てくるのですか?

>>598
どうもよく分かりません…
演算子を展開するという事自体初めて見たのですが、どういうルールのもとに展開しているのでしょうか?
皆さん場の量子論に進む前にどこかでやっているのでしょうか?
それとも古典論との類推で分かるものなのでしょうか
628ご冗談でしょう?名無しさん:2009/10/23(金) 01:10:26 ID:XrmaCjq9
まぁ、場の演算子なんて、行列だと思えばよい。その固有値が古典場だな。
629ご冗談でしょう?名無しさん:2009/10/23(金) 01:18:25 ID:???
>>626
サクライとSakuraiでいいんじゃね?
630ご冗談でしょう?名無しさん:2009/10/23(金) 01:19:30 ID:???
>>627
演算子A,Bを任意の状態|ψ>に作用させてA|ψ>=B|>となったとすると
この二つの演算子は同じ写像(A=B)だよね。
固有状態{|ψ_i>}に位置を変数として持つ演算子φ(x)を作用させて
φ(x)|ψ_i>=Σ_j c_ij(x) |ψ_j>となったとすると、生成消滅演算子を使って
φ(x)=Σ_ij c_ij(x) a_j^†a_iと書けて演算子が「Σ 関数*演算子」という形に展開できる。
演算子の「フーリエ展開」というのはよく分からんが雰囲気はこんな感じのはず。
631ご冗談でしょう?名無しさん:2009/10/23(金) 01:22:33 ID:???
一行目書き間違い
×A|ψ>=B|>
○A|ψ>=B|ψ>
632ご冗談でしょう?名無しさん:2009/10/23(金) 01:32:58 ID:w8gOVhfO
熱気球が膨らむ(飛ぶのではない)原理は
熱せられた空気が軽くなり上昇することによって膨らむのか、熱せられた空気のかさが大きくなり気球を膨らませるのかどちらでしょうか?
633ご冗談でしょう?名無しさん:2009/10/23(金) 01:44:18 ID:???
>>627
無限自由度だからわからんのか
有限自由度でもわからんのかどっちだ?

例えば、n個の粒子がバネでつながっている(そのような力を及ぼしあう)系を考える場合、
●γγγ●γγγ●γγγ●γγγ●
各粒子の位置の演算子を(離散)フーリエ変換して得られる演算子を考えることがあるが
これだったらわかるのか?

それと、すでに指摘されてるが >>592>>596 の「固有関数」が何を指してるのかがよくわからない。
何の固有関数?
634ご冗談でしょう?名無しさん:2009/10/23(金) 01:54:02 ID:???
>>629
サクライって演習書だったの!?
635ご冗談でしょう?名無しさん:2009/10/23(金) 02:04:43 ID:???
>>632
同じ質量(あるいは分子数)の気体は、
圧力が同じなら、温度が高いほど体積が大きい。
636ご冗談でしょう?名無しさん:2009/10/23(金) 05:53:41 ID:???
>>632 後者
637ご冗談でしょう?名無しさん:2009/10/23(金) 06:17:02 ID:???
>>603だけど思いっきりスルーされているので帰ります
物理板じゃちょっと難しい話題だったみたいなので力学的なネタが出たときにまた来ます
638ご冗談でしょう?名無しさん:2009/10/23(金) 06:34:48 ID:???
>>625
材料内部に応力を残す様に成型する事により、耐圧・剛性・疲労強度などを向上させる事。
耐圧容器や砲身の耐圧改善、柱の座屈防止などに使われる。
639ご冗談でしょう?名無しさん:2009/10/23(金) 11:59:22 ID:naJnGxyA


厨房の物理の問題なんだが
斜面を下る台車の運動
てなんだっけ?
久しぶりに思いだしかんだが答えが思いだせん
640ご冗談でしょう?名無しさん:2009/10/23(金) 12:03:44 ID:???
>斜面を下る台車の運動
これだけで答えを出せる香具師がいたら見てみたいもんだ
641ご冗談でしょう?名無しさん:2009/10/23(金) 12:09:56 ID:naJnGxyA


>>640
すまん
なんか記憶が曖昧なんだ
642ご冗談でしょう?名無しさん:2009/10/23(金) 12:47:34 ID:???
ID:naJnGxyA
643ご冗談でしょう?名無しさん:2009/10/23(金) 12:50:08 ID:???
すまんがテレパスの上級者はココにはいない
644ご冗談でしょう?名無しさん:2009/10/23(金) 12:52:28 ID:???
わたしテレパスだけど、本人の記憶が曖昧なので無理でした
645ご冗談でしょう?名無しさん:2009/10/23(金) 14:33:28 ID:???
>>637
3行読むだけで問題がわかるように書けば?
646ご冗談でしょう?名無しさん:2009/10/23(金) 14:47:30 ID:???
煽れば答えてくれると思っていいのは小学生まで
647ご冗談でしょう?名無しさん:2009/10/23(金) 16:06:55 ID:???
質問してるように見えないし
へりくだる必要はないけど質問だとわかるように書いてくれ
648ご冗談でしょう?名無しさん:2009/10/23(金) 16:43:45 ID:kl1ckmrA
水はなぜ蒸発して水蒸気になると上へと行くんでしょうか?
空気より軽く、運動が活発なため、ということでしょうか?
649ご冗談でしょう?名無しさん:2009/10/23(金) 16:48:40 ID:???
>>648
分子量考えれば同体積の窒素や酸素より軽いのわかるだろ。
650ご冗談でしょう?名無しさん:2009/10/23(金) 16:50:20 ID:kl1ckmrA
>>649
ありがとうございます
651ご冗談でしょう?名無しさん:2009/10/23(金) 19:22:32 ID:DQfaESGD
傾斜角30°の斜面に98Nの荷物を起き、摩擦のない滑車を通して20Nの重りをつるした。
これ以上軽い重りをつけると物体は滑る。
(1)斜面の摩擦計数を求めよ
(2)何Nの重りをつるすと物体は上に滑り上がるか
※1Φ=30
※2滑車は傾斜の頂点についてるものとする
お願いします
652ご冗談でしょう?名無しさん:2009/10/23(金) 19:58:54 ID:???
653ご冗談でしょう?名無しさん:2009/10/23(金) 20:16:24 ID:???
地表付近でわざと爆発を起こして地面の内部構造を図る手法をなんといいますか?
654ご冗談でしょう?名無しさん:2009/10/23(金) 20:20:23 ID:???
>>653
人工地震による地下構造探査の事か?
655ご冗談でしょう?名無しさん:2009/10/23(金) 20:21:34 ID:???
>>651
>斜面に98Nの荷物を起き

質問の表現が既におかしい
656ご冗談でしょう?名無しさん:2009/10/23(金) 20:30:07 ID:???
教えてください。m(__)m
wikiなどを見ますと、物理学では時空や時間を概念として捉えていますが、
光に関しては概念とありません。光には概念として捉えることの出来ない何かがあるのですか?
よろしくお願いしますm(__)m
657ご冗談でしょう?名無しさん:2009/10/23(金) 20:30:31 ID:???
熱力学の第ゼロ法則について質問なんですけど
閉じた系でも熱い側(状態量が多い側)から寒い側(状態量が低い側)に
流れるほうがエントロピーは結局増大するということで間違ってないんでしょうか
658ご冗談でしょう?名無しさん:2009/10/23(金) 20:32:00 ID:DQfaESGD
>>655
わかれよカスw
解んねーなら解んないでちゅよってかいとけよw
これだからゆとりはwww
国語と道徳からやり直して来いよwwwwwwwww
659ご冗談でしょう?名無しさん:2009/10/23(金) 20:33:34 ID:DQfaESGD
係数0.341で78Nだろjk
暗算で出来るだろwww
660ご冗談でしょう?名無しさん:2009/10/23(金) 20:35:39 ID:???
>>656
光は実測できるから概念じゃないじゃん
661ご冗談でしょう?名無しさん:2009/10/23(金) 20:36:56 ID:KHFtXwSe
炎上期待アゲ
662ご冗談でしょう?名無しさん:2009/10/23(金) 20:42:05 ID:???
>>660
ありがとうm(__)m
物理学では実測出来ないものは概念としてとらえ、実測できるものは概念じゃないとするのですか。

663ご冗談でしょう?名無しさん:2009/10/23(金) 20:51:15 ID:???
スルーしとけ
664ご冗談でしょう?名無しさん:2009/10/23(金) 20:52:15 ID:???
マッチポンプか。低能だな。
665ご冗談でしょう?名無しさん:2009/10/23(金) 20:53:52 ID:???
>>658-659
解ったんならこのスレには2度と来んなよ。
ちなみに答えは間違えてるがな。
666ご冗談でしょう?名無しさん:2009/10/23(金) 20:56:16 ID:???
>>665 こういう手合いは相手にスンナ。
667ご冗談でしょう?名無しさん:2009/10/23(金) 21:00:49 ID:???
>>654
それです。
でその言葉でググったら「屈折法探査」でした。
ありがとうございました。
668ご冗談でしょう?名無しさん:2009/10/23(金) 21:03:32 ID:???
尿結石検査

似見えた
669ご冗談でしょう?名無しさん:2009/10/23(金) 21:21:45 ID:???
>>657
以下の言葉の意味が不明。
・熱力学第ゼロ法則
・状態量が多い(少ない)
670ご冗談でしょう?名無しさん:2009/10/23(金) 21:24:49 ID:???
>>669
熱力学の第零法則は熱力学の第零法則としか言いようが無いなぁ
状態量はエントロピーと置き換えてください
671ご冗談でしょう?名無しさん:2009/10/23(金) 21:26:13 ID:???
>>670
あ、熱平衡の状態に向かうってことです<第零法則
672ご冗談でしょう?名無しさん:2009/10/23(金) 21:32:35 ID:???
>>670
閉じた系で、各微小部分は平衡状態とみなせて
エントロピー密度が定義できるが
全体としては平衡状態にない場合、
熱伝導がおきれば、系のエントロピーは当然、増大する。
673ご冗談でしょう?名無しさん:2009/10/23(金) 21:53:57 ID:???
第零法則じゃなくて第2法則の話だな。
674ご冗談でしょう?名無しさん:2009/10/23(金) 21:54:31 ID:???
>>672
なんだかわかったようなわからないような説明ですが何となく感覚はつかめました。
サンクス、熱伝導による増大分を考えれば良いってことですか
675ご冗談でしょう?名無しさん:2009/10/23(金) 22:10:40 ID:???
>>673
要は第零法則と第二法則との関連性が知りたかったということです
676592=596:2009/10/23(金) 22:38:09 ID:???
>>628
>>630
皆さん有り難う御座います
丁寧に説明して下さってるのにもの分かりが悪くて済みません
630の生成消滅演算子を使ってそのように書けるというのがよく分からないのですが…

>>633
有限自由度の系でも同様に分かりません、場の量子論に固有の問題についての質問のように書いてしまって済みません
自分の読んでいる本では、まさにおっしゃっているバネの系について、
「古典論との類推により」とだけ断って演算子を展開しています
> >>592>>596の「固有関数」
こちらも分かりにくくて済みません
非相対論的な場の量子化の際に、場の演算子をシュレディンガー方程式を満たす固有関数系で展開するという話です
677ご冗談でしょう?名無しさん:2009/10/23(金) 22:48:10 ID:???
差し支えなければ本のタイトルを晒したほうがよくないか
678ご冗談でしょう?名無しさん:2009/10/23(金) 22:56:01 ID:???
>>638
ありがとうです。
容器などにどうやって応力残すのですか?
679ご冗談でしょう?名無しさん:2009/10/23(金) 23:11:25 ID:???
量子力学の質問です。

三次元の井戸型ポテンシャル(一辺Lの立方体)についてなのですが、
(T)箱の端の波動関数を0とする条件
つまりψ(L,y,z,)=ψ(x,L,z,)=ψ(x,y,L)=0
のとき

(U)周期的境界条件を条件にした場合
つまりψ(x,y,z,)=ψ(x+L,y,z,)=ψ(x,y+L,z)=ψ(x,y,z+L)
という条件のとき

とでエネルギー固有値を求めました。

すると(T)は
E=h^2/(8πm)・(π/L)^2・{(n_x)^2+(n_y)^2+(n_z)^2}
ただしn_x,y,zは0を含まない自然数。

(U)は
E=h^2/(8πm)・(2π/L)^2・{(n_x)^2+(n_y)^2+(n_z)^2}
ただしn_x,y,z=0,±1,±2...

となりました。明らかに(T)と(U)ではエネルギー固有値がちがってきます。
これはなぜなのでしょうか?
このほかのフェルミ波数等は同じ値をとるのにエネルギー固有値だけちがうというのはいいのでしょうか?
680ご冗談でしょう?名無しさん:2009/10/23(金) 23:14:33 ID:???
>>676
多分
1電子シュレディンガー方程式 → N電子シュレディンガー方程式
 → 第二量子化形式で書き直す → 馬の量子論
てやると分かると思う
途中が抜けてると思う
681ご冗談でしょう?名無しさん:2009/10/23(金) 23:20:03 ID:???
>>679
波長も違う
682ご冗談でしょう?名無しさん:2009/10/23(金) 23:23:11 ID:???
>>679
・境界条件が違うことはポテンシャルが違うのと同じくらい全然違う
・フェルミ波数云々は大きい系の極限だとそうなるだけ
683ご冗談でしょう?名無しさん:2009/10/23(金) 23:30:37 ID:Ohrxhao6
>>682
フェルミ波数はおおきいので同じになるけど細かくみると全然ちがうということですか。
境界条件だけでだいぶ変わるんですね。
ありがとうございました。
684ご冗談でしょう?名無しさん:2009/10/23(金) 23:32:31 ID:???
>>678
引張り強度の違う材料2層で容器を作り、中に降伏圧以上の圧力をかけるとか、
予め熱した外筒に内筒挿入するとか、
フィラメント巻くのは自緊とは言わんのだっけ?
685ご冗談でしょう?名無しさん:2009/10/23(金) 23:37:56 ID:???
>>684
>>678
>引張り強度の違う材料2層で容器を作り、中に降伏圧以上の圧力をかけるとか、
降伏圧以上の圧力かけると、応力がのこるの?それは二層で作らないとダメ?
686ご冗談でしょう?名無しさん:2009/10/23(金) 23:49:52 ID:???
焼き嵌めみたいに二つのパーツの間で応力を発生させるんじゃなくて
ひとつの材料で残留応力を残すのを自緊って呼ぶんじゃなかった?
687ご冗談でしょう?名無しさん:2009/10/24(土) 03:45:49 ID:???
中級の大きさのピーマン(完全無農薬の)、光水(日本のビールと大麦茶を割ったもの)、にんじん(完全無農薬の)、じゃがいも(北海道産)皮を厚めに切った中のもの、塩(ナトリウムの塩)少々、これを混ぜて、飲むと、朝鮮人が、日本人になるよ。
ワクチンを取ってみれば解る。
688ご冗談でしょう?名無しさん:2009/10/24(土) 03:57:57 ID:Zfgr6ZLO
ヘブライはしその葉っぱ食べてれば、治るよ。
689ご冗談でしょう?名無しさん:2009/10/24(土) 04:24:52 ID:???
>>669
>・熱力学第ゼロ法則

それはないわ
690ご冗談でしょう?名無しさん:2009/10/24(土) 06:38:38 ID:???
ゼロ法則はあるぜ
691ご冗談でしょう?名無しさん:2009/10/24(土) 07:54:29 ID:???
系1と系2が熱平衡にあり、系2と系3が熱平衡にあれば、系1と系3も熱平衡にある
692ご冗談でしょう?名無しさん:2009/10/24(土) 08:20:54 ID:???
論理学のイロハ、文系氏分かるかな?
693ご冗談でしょう?名無しさん:2009/10/24(土) 10:30:45 ID:???
>>689
山本義隆はその言葉使うね。一般的用語でないかもしれんが、駿台の山本の講義でで習ったとおも。
694ご冗談でしょう?名無しさん:2009/10/24(土) 11:04:51 ID:B+gKtOZj
京都大 テンプルフロート 初期実験成功。
695ご冗談でしょう?名無しさん:2009/10/24(土) 11:11:58 ID:rUnzii5N
よし! 今日のお昼は天丼にしよう!
696ご冗談でしょう?名無しさん:2009/10/24(土) 11:14:15 ID:???
>>693
一般的用語だと思うが。
http://ja.wikipedia.org/wiki/%E7%86%B1%E5%8A%9B%E5%AD%A6%E7%AC%AC%E9%9B%B6%E6%B3%95%E5%89%87
Wikipediaでは29の言語でこの項目があるぞ。
697ご冗談でしょう?名無しさん:2009/10/24(土) 11:16:10 ID:???
>>692
まて、>>691 は論理で出てくるわけじゃないぞ。熱平衡状態の性質を述べた物理的記述。
熱力学の公理。
698ご冗談でしょう?名無しさん:2009/10/24(土) 12:09:46 ID:B+gKtOZj
小学生のゆとり、養護院の治し方は、日本の教育に英語と道徳を入れるだけでいい。
699ご冗談でしょう?名無しさん:2009/10/24(土) 12:12:26 ID:???
ハア?
700ご冗談でしょう?名無しさん:2009/10/24(土) 12:38:18 ID:???
中学生のゆとり 養護院の治し方は、今の小学生の日本の教育に英語と道徳を入れて、教育を受けた後、高校受験の勉強を一年の最初からしてればいい。
701ご冗談でしょう?名無しさん:2009/10/24(土) 12:39:52 ID:???
国語もろくにできない段階で外国語とか噴飯モノ
702ご冗談でしょう?名無しさん:2009/10/24(土) 12:50:39 ID:B+gKtOZj
幼稚園は、自由闊達に遊んで、行事を入れればいい スイスレベルだけどね。
703ご冗談でしょう?名無しさん:2009/10/24(土) 13:01:08 ID:B+gKtOZj
うさぎ飛びは、膝、関節に、水がたまるから禁止。
704ご冗談でしょう?名無しさん:2009/10/24(土) 13:25:22 ID:???
高校は、大学に進学するんだったら、高校二年で、高校の学問を終えて、さらに高校二年の10月くらいから、高校の残りの学問と大学受験の勉強と平行して勉強し、高校三年生からは、大学受験の勉強をすればいい。普通の進学校の教育だよ。予備校も忘れずにね。
705ご冗談でしょう?名無しさん:2009/10/24(土) 13:31:16 ID:B+gKtOZj
極まると、休憩や休みの取り方で、合否が決まるから。
706ご冗談でしょう?名無しさん:2009/10/24(土) 13:36:55 ID:???
また猫野郎が湧いてきてるのか

Kitty Guyな。
707ご冗談でしょう?名無しさん:2009/10/24(土) 15:30:09 ID:???
>>676
有限自由度の場合については話は難しくない。
A, B を複素ベクトル空間V上の線形変換(演算子)、c を複素数とすると、
c A とか A + B もやはりV上の線形変換になることはわかるよな?

これを使えば、N個の演算子 q_1, q_2, …, q_N が与えられたら
(1/√N) 農[j =1〜N] exp(i2πjk/N) q_j (kは整数)
のような形もやはり演算子になることがわかる。これを Q_k と書くと
q_j = (1/√N) 農[k=1〜N] exp(-i2πjk/N) Q_k
となっている。この形は離散フーリエ展開そのものだ。
708ご冗談でしょう?名無しさん:2009/10/24(土) 15:46:50 ID:???
ここの板の人はどんな質問が来ても答えられるみたいだけどなぜ?
自分の場合、得意分野なら結構ディープな質問でも答えられるけど
古典力学とか電磁気なんて殆ど忘れてしまってるし
学部2年レベルの質問とかでも即答できる自信がない。
数学も自分の研究分野で使ってるのしか知らないし。
709ご冗談でしょう?名無しさん:2009/10/24(土) 15:49:24 ID:???
単にたくさんの人間が答えてるからだろう
中には優秀な奴もいるだろうけど
710ご冗談でしょう?名無しさん:2009/10/24(土) 15:49:29 ID:???
どんな質問でも、というのはウソだな
俺が質問する内容の5割は誰も返答してくれない
とはいえ、返答してくれるときは非常に参考になるわけだが…
711ご冗談でしょう?名無しさん:2009/10/24(土) 15:51:39 ID:???
>>710
何番?まあ俺にはわからんと思うが
712ご冗談でしょう?名無しさん:2009/10/24(土) 15:54:12 ID:???
>>711
いちいち探すの面倒だからアレだけど、ほとんど量子力学に関する質問
713ご冗談でしょう?名無しさん:2009/10/24(土) 16:09:30 ID:???
でも場の量子論とか相対論の質問とかにも返答がつくのはすごいと思う
とくに、一般相対論なんて真面目に勉強してる人の数自体ほとんど少ないじゃん。
714ご冗談でしょう?名無しさん:2009/10/24(土) 16:11:50 ID:???
「ほとんど少ない」ってどんな日本語だよ。
「ほとんどいない」「ごく少ない」こんなとこだろ。
715ご冗談でしょう?名無しさん:2009/10/24(土) 16:15:09 ID:EKjuv/VE
>>660
実測て測ることだから概念だよね。
716ご冗談でしょう?名無しさん:2009/10/24(土) 16:17:57 ID:???
場の量子論、相対論の学部レベルのことならまともな解答がくる
院レベルになると一気にスルーされる
717ご冗談でしょう?名無しさん:2009/10/24(土) 16:20:59 ID:???
院生なら教官に聞けってことか
718ご冗談でしょう?名無しさん:2009/10/24(土) 16:25:56 ID:???
院レベルの内容は専門の人しか知らないからな
単純に答えられる人間が少ない
719ご冗談でしょう?名無しさん:2009/10/24(土) 16:27:28 ID:3TEOnd9s
院生レベルの質問は、ちょっとした質問じゃなくね
720ご冗談でしょう?名無しさん:2009/10/24(土) 16:30:13 ID:???
複雑な式がいるような答はテキストじゃかけないしな。
721ご冗談でしょう?名無しさん:2009/10/24(土) 16:35:50 ID:???
質問が不正確なこともよくあるから
本の内容を聞くならタイトルぐらい書けばいいと思うな
722ご冗談でしょう?名無しさん:2009/10/24(土) 16:44:35 ID:???
708 名前:神も仏も名無しさん[sage] 投稿日:2009/10/24(土) 09:24:07 ID:R1y5G1jg
たとえば20cmという何か、があるわけではない。
そういう意味で20cmは概念。
時間もおなじ。

709 名前:神も仏も名無しさん[sage] 投稿日:2009/10/24(土) 09:29:27 ID:R1y5G1jg
まあ、定規が力を持ちすぎて、実際に20cmはそのものとして独立に単独で
存在する、と思い込んでいる人もおるかもしれないが、それはどうにもならん。
723ご冗談でしょう?名無しさん:2009/10/24(土) 18:17:15 ID:qHlXPVUV
ちょっとした疑問
COP30のエアコンを考えた。まず、COP6のエアコンのポンプの作動速度を5分の1にする。
次にそのポンプの数を5個に増やす。これで同じ冷暖房能力でCOP30のエアコンの完成です。
なぜそうなるか。ポイントは運動エネルギーは速度の2乗に比例する。5分の1の速度なら
25分の1のエネルギーですむ。何か指摘があればどうぞ。
724ご冗談でしょう?名無しさん:2009/10/24(土) 18:36:10 ID:???
セックスの時はゆっくり腰を使うてことですか
725ご冗談でしょう?名無しさん:2009/10/24(土) 19:03:23 ID:???
じゃあポンプ25個必要じゃないのですか?
726ご冗談でしょう?名無しさん:2009/10/24(土) 19:06:10 ID:???
一人の女性に男性25人でいいんじゃないか。
727ご冗談でしょう?名無しさん:2009/10/24(土) 22:28:05 ID:???
物性に関する質問です。
本来非磁性である固体が、不純物や表面の効果で磁性が発現する場合、
その原因が時間反転対称性や空間反転対称性の破れであるという議論をよく聞きますが、
対称性が破れと磁性はどのような関係にあるのでしょうか?
728ご冗談でしょう?名無しさん:2009/10/24(土) 23:52:00 ID:???
対称というと三角形や立方体みたいなのを思い浮かべそうだが一番対称性が高いのは球
つまり等方性を持った状態で、原子の磁気モーメントだったら勝手な方向をバラバラに
向いてる状態が一番対称性が高い
これが何らかの理由で一方を向くようになるのは対称性があがったのではなく下がっている
専門家ではないのでインチキ説明だが
729ご冗談でしょう?名無しさん:2009/10/24(土) 23:53:53 ID:???
時間的対称性の破れってエントロピー増大と似てるけどなにか関係あるの?
730ご冗談でしょう?名無しさん:2009/10/25(日) 00:01:11 ID:GR0sfckE
>>729
エントロピーは時間が経過するに従い増大するから。過去と未来が等価ではなくなる。


質問です。
モノポールが発見されたと聞いたのですが本当ですか?
731ご冗談でしょう?名無しさん:2009/10/25(日) 00:02:04 ID:GR0sfckE
>>729
>>730は検討はずれな応答でした。
732ご冗談でしょう?名無しさん:2009/10/25(日) 00:02:35 ID:???
>>730
ワタシは聞いてない。
そもそもあなたはどこで聞いたのか。
733ご冗談でしょう?名無しさん:2009/10/25(日) 00:02:41 ID:???
うそです。
734ご冗談でしょう?名無しさん:2009/10/25(日) 00:08:16 ID:GR0sfckE
>>732
http://science6.2ch.net/test/read.cgi/sky/1218278160/907-910
こんなページもありましたが
http://nanonet.mext.go.jp/modules/news/article.php?a_id=676

発見されれば大騒ぎになるはずなのにおかしいなと思ってはいました。
735ご冗談でしょう?名無しさん:2009/10/25(日) 00:49:07 ID:???
天井にヒモを吊してバネを結び、その下に質量のある物体を吊し、バネとヒモの質量を無視したとします。
物体を吊さない場合は当然ヒモの張力は0になると思います。

バネを挟まずに物体を吊した場合とバネを挟んで物体を吊した場合のヒモの張力は同じになりますか?
736ご冗談でしょう?名無しさん:2009/10/25(日) 00:52:34 ID:???
物体が静止していれば同じ
737592=596:2009/10/25(日) 01:13:49 ID:???
皆さん長らくお付き合い頂き本当に有り難う御座います

>>686
分かりました
N粒子シュレディンガーの理解が不十分だったかも知れないので復習中です

>>707
少し分かってきました
演算子の部分と残りの部分(無限自由度の例で言えば関数の部分)の順序については、
その関数は演算子が作用する空間の関数ではないので順序を気にする必要はないという事でしょうか
例えば>>676の最後に書いた例では、演算子が作用する空間はフォックスペースで、
固有関数の方は通常の量子力学のヒルベルト空間の関数である為に、前者が後者に作用する訳ではなく
書き順も気にする必要はないという事でしょうか
下から三行目辺り自分で書いていて怪しいのですが…
738ご冗談でしょう?名無しさん:2009/10/25(日) 01:40:09 ID:???
>>737
行列 Φ(x) を、別の行列 A(k) と関数 f(x,k) を使って
Φ(x) = ∫f(x,k) A(k) dk
と表す。
Φ(x) の性質によって、うまい f(x,k) と A(k) の選び方がある。
…てぐらいの話だと思うんだが、難しく考え過ぎでは?
739ご冗談でしょう?名無しさん:2009/10/25(日) 02:12:33 ID:???
>>737
>演算子が作用する空間はフォックスペースで、
>固有関数の方は通常の量子力学のヒルベルト空間の関数である為に、前者が後者に作用する訳ではなく
>書き順も気にする必要はないという事でしょうか

その理解でいいと思うよ。
(自由)場の「量子化」でやってることは、なんらかの波動方程式に従う場を
無数の独立な調和振動子の集まりと見て、各々の調和振動子を量子化する
ということ。

「シュレディンガーの波動方程式(量子論ではない)」を量子化する場合も同じ。
このあたりは朝永さんとか読むといい。
740 ◆ly/TAatdog :2009/10/25(日) 02:27:00 ID:???
>>737
>固有関数の方は通常の量子力学のヒルベルト空間の関数である為に
 量子力学の演算子が、位置座標の関数に対して作用するものだと思っていませんか?
だとすると、まずその理解を変えた方がよいでしょう。

 1粒子の量子力学でも、状態ベクトルそのもの と、
それを粒子の位置演算子の固有基底で展開した成分 は、別物と考えた方がよいです。
式で書くとすると、|ψ> = ∫f(x) |x> dx ( |x> は位置演算子の固有状態 )
といった感じ(ややいい加減だが)。
 あなたの言う「固有関数」は、|ψ> と f の区別ができていないように見える。
「波動関数」といった言葉は、このどちらを指すこともある感じなので要注意。
場の演算子の展開に使われたりするのは f の方。
741740:2009/10/25(日) 02:49:51 ID:???
 量子論の演算子というのは、(1粒子量子力学でも、場の量子論でも)
あくまで状態ベクトル空間に作用する線型写像であって、
|ψ> に作用することはあっても、f に対しては"スルー"です。
742ご冗談でしょう?名無しさん:2009/10/25(日) 07:06:50 ID:mN03Rar1
誘電率ε、電気伝導率σ、抵抗率ρの媒質をつめた容量Cの平行板コンデンサーの両極間に電流が流れるとき、抵抗Rを求めよ。


コンデンサーに電流は流れるの?
743ご冗談でしょう?名無しさん:2009/10/25(日) 07:17:38 ID:???
流れるよ
744ご冗談でしょう?名無しさん:2009/10/25(日) 08:30:09 ID:mN03Rar1
>>743
充電中は流れるのはわかるけど…電荷がたまってからでも流れるの?
745ご冗談でしょう?名無しさん:2009/10/25(日) 08:46:34 ID:???
「電荷がたまったとき」 ってどう意味で言ってるの?
「電流が流れなくなったとき」 の意味?
746ご冗談でしょう?名無しさん:2009/10/25(日) 08:54:41 ID:???
日本語でおk
747ご冗談でしょう?名無しさん:2009/10/25(日) 09:48:55 ID:???
>>746>>745宛ならとんだ馬鹿だな。
748ご冗談でしょう?名無しさん:2009/10/25(日) 09:56:47 ID:???
どう意味で言ってるの?(笑)
749ご冗談でしょう?名無しさん:2009/10/25(日) 10:00:52 ID:???
おまえの日本語もおかしいよ
750ご冗談でしょう?名無しさん:2009/10/25(日) 10:05:50 ID:???
>>742
ρσ=1だからρとσ片方でいいんじゃないか
具体的に解くなら板の面積と極板間の距離もほしい
それはともかく、電気を通す材料を使ってるわけだから普通に直流でも流れるだろう
インピーダンスと言わず抵抗というってことは直流抵抗を聞いてる気もする
問題文をよく読んでくれ
練習としてはインピーダンス(周波数に依存)を出しとけばいいんじゃないか
過渡現象を考えてもいいし
751ご冗談でしょう?名無しさん:2009/10/25(日) 10:22:59 ID:???
describe in English, it's ok
752ご冗談でしょう?名無しさん:2009/10/25(日) 10:53:42 ID:???
Decrivez en anglais, c'est ok.
753ご冗談でしょう?名無しさん:2009/10/25(日) 10:55:57 ID:???
안녕하십니까,ok
754ご冗談でしょう?名無しさん:2009/10/25(日) 11:10:39 ID:???
>>744
溜まっている電荷が変化しなくなっても(電圧が変化しなくても)流れるか?
という意味か?
理想的なコンデンサなら流れないが、
この問題では理想的でないコンデンサを扱っている。
755ご冗談でしょう?名無しさん:2009/10/25(日) 13:27:58 ID:???
並列回路ではなぜ電圧が等しくなるのでしょうか?
キルヒホッフの電圧則が関係しているのですか?教えてください
756ご冗談でしょう?名無しさん:2009/10/25(日) 13:33:23 ID:???
>>755
難しく考えるな。
お前んちの冷蔵庫と、俺んちのテレビも並列なんだ。
電圧が違ったら困るじゃないか。
757755:2009/10/25(日) 13:40:22 ID:???
うちの冷蔵庫は三相200ヴォルトです!
758ご冗談でしょう?名無しさん:2009/10/25(日) 13:52:12 ID:???
>>755
というか、導体(導線)の性質だと思ったほうがいいでしょうね。
抵抗のとても小さいモノと抵抗が大きいモノたちが接していて電流が流れていれば
抵抗の小さいモノの中ではほとんど電位降下がない(電位がほぼ一定)
ということです。

導線A --- 抵抗の大きいモノたち --- 導線B

Aの中では電位はほぼ一定、Bの中でも電位はほぼ一定だから
どの抵抗の大きいモノの両端の電位差もほぼ同じ。
759ご冗談でしょう?名無しさん:2009/10/25(日) 13:58:30 ID:???
イジング模型では内部エネルギーってどのように決まるのでしょうか
↑とか↓とか、何に作用されて変わるんですか?
760ご冗談でしょう?名無しさん:2009/10/25(日) 13:59:56 ID:NtzwHRJI
神の気まぐれ
761ご冗談でしょう?名無しさん:2009/10/25(日) 14:10:56 ID:???
揺らぎって奴だっけ?
762ご冗談でしょう?名無しさん:2009/10/25(日) 14:44:33 ID:???
>>759
ヒートバスからもらう(あるいはあげる)でしょ。
763ご冗談でしょう?名無しさん:2009/10/25(日) 15:11:33 ID:???
熱浴関係なく揺らぎはあるはず
764ご冗談でしょう?名無しさん:2009/10/25(日) 15:29:25 ID:???
なんで生物が誕生したんですか?
765ご冗談でしょう?名無しさん:2009/10/25(日) 15:32:21 ID:???
>>764

>>2
2.)
 以下のような質問に物理板住人は飽き飽きしているので、たぶん無視されます。
 しないで下さい。
 「相対性理論は間違っています」「量子力学は間違っています」
 「宇宙論は間違っています」「シュレディンガーの猫は変です」
 「永久機関を作りました」「タイムマシンについて教えて」
 「どうして〜?」関連(→「どのようにして〜?」と質問すること)

 「なぜ〜?」関連(たいてい、物理学の対象ではないため) ←
766ご冗談でしょう?名無しさん:2009/10/25(日) 17:56:25 ID:???
>>764
仮定だが、物質だけで脳(生物)を生成することは不可能だろう
おそらく宇宙の初期エネルギーの中にDNAを構築する素粒子が存在していたと考えられなくもない
767ご冗談でしょう?名無しさん:2009/10/25(日) 18:01:51 ID:NtzwHRJI
>>766
DNAが素粒子である可能性はないですか?
768ご冗談でしょう?名無しさん:2009/10/25(日) 18:09:26 ID:???
>>767
「仮定」って書いたんでこれ以上書くつもりはないし、俺の思想にすぎない
769ご冗談でしょう?名無しさん:2009/10/25(日) 18:09:50 ID:???

オカルトや「と」系はスルーで
770ご冗談でしょう?名無しさん:2009/10/25(日) 18:32:08 ID:???
>>765
>>769
馬鹿か?
発想がなければ物理学なんて永久に発展してねーよ
生物が何故誕生したのか?とか物理学的に重要な疑問だろ
理論的に成り立つか?それが実証できるか?の積み重ねで今の物理学があるんだろうが
771ご冗談でしょう?名無しさん:2009/10/25(日) 18:34:57 ID:???
>>770
荒らしは消えてくれないか
772ご冗談でしょう?名無しさん:2009/10/25(日) 18:39:25 ID:???
>>771
質問スレでテンプレ貼り付けたり、オカルトって言葉でぶったぎる奴の方が荒らしだと思うが?
773ご冗談でしょう?名無しさん:2009/10/25(日) 18:43:40 ID:???
>>772>>771
両者ともに、今はお互いの言い分の良いところを見直してみると良い。

>>771へ 実験事実によって証明された理論は正しい。
>>772へ 疑問を感じなくなったら、学問の進歩はありえない。これも正しい。

故に、相互が探求され、その新たな物理学を創発させてみるのも良い。
774ご冗談でしょう?名無しさん:2009/10/25(日) 18:46:00 ID:NtzwHRJI
>>773
嵐は止めてね。
775ご冗談でしょう?名無しさん:2009/10/25(日) 18:47:51 ID:???
生物板で聞けよ
776ご冗談でしょう?名無しさん:2009/10/25(日) 18:50:06 ID:???
私は、台風かね?物理学の領域は、専門性が非常に重要視されるが・・
しかしながら、いつも同じ教科書で、同じ講義をしているスタイルが物理学なのかね?

違うだろ?研究者たるもの・・・仮説を立て、実験事実によって証明されなければ評価されない。
それが基本ではないのか?
777ご冗談でしょう?名無しさん:2009/10/25(日) 18:51:13 ID:0isk+Rcs
ちょっと皆様に教えて頂きたいのです!!
「ミリカンの油滴実験」についてレポートを書くんですが、
どのような内容にすればいいか分かりません
その実験の意図と方法をただ調べてまとめればいいのでしょうか

ちなみに私は大学1年の理系ですが
高校では物理を履修していませんでした
778ご冗談でしょう?名無しさん:2009/10/25(日) 18:53:28 ID:NtzwHRJI
マジ実験しろ!優確実。
779ご冗談でしょう?名無しさん:2009/10/25(日) 18:56:36 ID:0isk+Rcs
それは無理でしょ〜

Wikiと辞書でなんとかイケますかね?
780ご冗談でしょう?名無しさん:2009/10/25(日) 18:57:13 ID:???
>>770
「なぜ?」と「どのようにして?」の違いが分かってないなw

>>777
実験の教科書の巻頭を読め

NGID:NtzwHRJI
781ご冗談でしょう?名無しさん:2009/10/25(日) 18:57:41 ID:???
>>777
ノーベル賞を取れるくらいの画期的な実験だった理由はなぜかを考えてまとめればいいと思う。
782ご冗談でしょう?名無しさん:2009/10/25(日) 18:58:44 ID:???
>>779
その手は不可能ではないが、判を押したように同じ内容のレポートが他人からも提出される
結果になってバレる。
783ご冗談でしょう?名無しさん:2009/10/25(日) 19:01:11 ID:???
>>779
これからでも、明日でもよい、大学の図書館に行けばよい。そこに、多分実験論文や検証論文が出ているはず。
それらを引用文献として参考にしながら、仮説・本論・反論・結論の形で書けばよい。それらをまとめて、アブストラクトを書けばよい。
それが普通の文系の場合レポートだそうだ。
784ご冗談でしょう?名無しさん:2009/10/25(日) 19:04:24 ID:0isk+Rcs
>>782
じゃ、じゃあ教科書とWikiと電子辞書とYahoo百科事典を
織り交ぜたらどうでしょか?!
785ご冗談でしょう?名無しさん:2009/10/25(日) 19:07:01 ID:???
>>784
それは、著作権の問題に触れるかも知れない。
結構、それらの媒体は見られているし、それらのコピーは現在問題になっている。

自分の言葉で書くのが一番良いわけ。理系ではオリジナリティを要求されるのからね。
786ご冗談でしょう?名無しさん:2009/10/25(日) 19:08:12 ID:0isk+Rcs
>>783
話が難しすぎて・・・
そんな大それたものでなくていいのですが・・・
たぶん論文読んでも意味がわからなさそうです(バカ
787ご冗談でしょう?名無しさん:2009/10/25(日) 19:10:38 ID:NtzwHRJI
ごたごた謂わずに、実験したら。
スプレーと空き缶と電源と顕微鏡があればOKだよ。
788ご冗談でしょう?名無しさん:2009/10/25(日) 19:11:42 ID:0isk+Rcs
>>785
でも何にも見ずに書くことは無理ですし・・・
自分の言葉でといっても
実験方法や結果などはどのみち同じ様な文章になりませんか?
789ご冗談でしょう?名無しさん:2009/10/25(日) 19:11:49 ID:???
>>779
ナレッジデータベースを作るIT技術の一手法と辞書で何をすんの?
790ご冗談でしょう?名無しさん:2009/10/25(日) 19:12:35 ID:???
>>786
だったら、教科書に書いてある項目を、箇条書きでまとめる。
自分が疑問に思ったことを、箇条書きに書き込み、疑問の内容を文献調査をして裏付ける。

どんな実験装置で、何を目的にしたのか?それはいつ、どこでをキチンと書き込めばよい。
791ご冗談でしょう?名無しさん:2009/10/25(日) 19:19:00 ID:???
>>777
は何を聞きたいの
出題者の意図?
792ご冗談でしょう?名無しさん:2009/10/25(日) 19:20:24 ID:???
>>789
その人はWikipediaをwikiって略しちゃうヒトなんだよ
793ご冗談でしょう?名無しさん:2009/10/25(日) 19:21:10 ID:???

 ミリカン味ぽん
794ご冗談でしょう?名無しさん:2009/10/25(日) 19:21:11 ID:0isk+Rcs
>>786
どんな実験装置で、何を目的にしたのか?それはいつ、どこでをキチンと書き込めばよい。

わかりました。そのポイントはおさえて書くことにします。
ですが、目的や方法はある程度理解できても
実験の結果によって導き出される式や計算の過程は
さっぱりわからないので「疑問に思ったこと」と言われても…
795ご冗談でしょう?名無しさん:2009/10/25(日) 19:23:53 ID:???
なんで高校で物理を履修してなかったようなヴァカが理系に進学できるの?
796ご冗談でしょう?名無しさん:2009/10/25(日) 19:26:11 ID:???
>>794
分かったふりをするよりは、分からないことをレポートに書いて置けば良いと思う。
分からなかったところは、後でキチンと数学と物理の書籍を読んで学んで欲しい。

遠回りのように見えるけれど、時間をかけてキチンと理解できるようになったとき、なんとなく苦労が報われるはず。
まだ学部の1年生ならば、そのくらいの遠回りをしても、まだ壁を越えるための時間はあるはずだから。
797ご冗談でしょう?名無しさん:2009/10/25(日) 19:32:38 ID:0isk+Rcs
入試には理科は2科目(私の場合は生物と化学)しか必要じゃなかったので。
しかも今の高校では理科3科目全て履修することはできません。

どうせ今の1年後期の物理の授業が終われば
もう物理に関わることは一生ないのでご心配なく。
798ご冗談でしょう?名無しさん:2009/10/25(日) 19:40:24 ID:???
暗記だけでなんとかなる生物と化学だけを履修した結果がこれだよ
799ご冗談でしょう?名無しさん:2009/10/25(日) 19:46:04 ID:???
物理も公式の意味と公式さえ暗記出来れば簡単に満点とれるだろ
800ご冗談でしょう?名無しさん:2009/10/25(日) 19:46:45 ID:???
とれませんよw
801ご冗談でしょう?名無しさん:2009/10/25(日) 19:48:16 ID:???
あまり、批判しない方が良い。物理でも、いくつかのタイプの学生や研究者が居る。

*手堅く論文をまとめて、資料を作り出すのが上手いタイプ → 省庁の研究所の技官タイプに多い。
*新しい仮説を提案することで、実験を誘導するタイプ → 理論物理学者
*観察及び実験データから、物理学的な考察によって、仮説を理論に同定するタイプ → 実験物理学者

故に、どのタイプなのかは・・それぞれのやり方(手法)で決まるはずなのだから。
802ご冗談でしょう?名無しさん:2009/10/25(日) 19:49:51 ID:???
わざわざ物理嫌いを増やすような真似するなよ
803ご冗談でしょう?名無しさん:2009/10/25(日) 19:53:20 ID:???
取れないのはバカだけでしょ
考え方と公式さえ分かっていれば物理なんてただの算数と同じ
804ご冗談でしょう?名無しさん:2009/10/25(日) 19:54:25 ID:???
その「考え方」が難しいんでしょ
805ご冗談でしょう?名無しさん:2009/10/25(日) 19:57:38 ID:???
普通に考えればわかることなのにそんなに難しいか?
少なくとも学部レベルまでなら理解に苦しむような内容はない
院レベルになると専門以外は少し厳しくなってくるが…
806ご冗談でしょう?名無しさん:2009/10/25(日) 19:58:46 ID:???
まあ、大抵は「実験室系」と呼ばれる系の問題に帰結するからね。
「系」そのものの対象を、どこにするのかだけでも、着眼点は変わってくる。

宇宙全体までの系になれば、一般相対性理論効果は無視できなくなる。
ある程度のマクロの系になれば、ニュートン物理や電磁気学、統計力学だけでもいける。
極小の系になれば、量子力学が顔を出す。

その辺りをキチンと理解しないと、難しいかも知れないけどね。
807ご冗談でしょう?名無しさん:2009/10/25(日) 20:00:05 ID:???
>>805
貴方、のんき坊主さんですね
808ご冗談でしょう?名無しさん:2009/10/25(日) 20:01:50 ID:???
マクスウェル方程式の相対論的効果ってどうやって導出すればいいんですか?
導入だけでいいので教えてください
809ご冗談でしょう?名無しさん:2009/10/25(日) 20:03:54 ID:???
簡単に書いておくよ。
ある変換に対して、普遍性を求めるのか?求めないのか?それがわかればいいよ。
810ご冗談でしょう?名無しさん:2009/10/25(日) 21:09:43 ID:???
相対性理論は間違っている
811ご冗談でしょう?名無しさん:2009/10/25(日) 21:15:53 ID:???

ふぁ〜あ・・・
812ご冗談でしょう?名無しさん:2009/10/25(日) 21:20:12 ID:???
J イナーシャって角加速度のことですか?単位も教えてください
813ご冗談でしょう?名無しさん:2009/10/25(日) 21:37:23 ID:???
inertia 慣性
moment of inertia 慣性モーメント
814ご冗談でしょう?名無しさん:2009/10/25(日) 21:41:56 ID:???
慣性モーメントってイニシャルのIで表していましたがなぜJなんでしょうか?
815ご冗談でしょう?名無しさん:2009/10/25(日) 21:48:38 ID:???

寝言?
816ご冗談でしょう?名無しさん:2009/10/25(日) 23:02:33 ID:???
>なぜJなんでしょうか?
Iが既に別の量に使われてるとか、1と紛らわしいからわざと使うのを避けたとか、
そんなところなんジャマイカ
817ご冗談でしょう?名無しさん:2009/10/25(日) 23:12:36 ID:???
量子力学の不確定性原理に関する質問はここでいいのでしょうか?

参考書の例題の一部なのですが導出過程が書かれていない箇所で
以下の部分の導出過程がわからないので教えていただきたいです。

エネルギーの不確定性を僵=10^(-3) [eV] とし、
K=(m*v^(2))/2
m=9.11*10^(-28) [g]
とすると速度vの不確定性况が
况=3*10^(4) [cm/sec]

googleで調べたところ不確定性は標準偏差のようなのですがどうも導出の手助けになるようなサイトは見つけられませんでした。
818ご冗談でしょう?名無しさん:2009/10/25(日) 23:12:43 ID:???
Initial の I かとおもった
819ご冗談でしょう?名無しさん:2009/10/25(日) 23:13:40 ID:???
>>817
量子力学の不確定性原理に関する質問じゃないよねこれ
820ご冗談でしょう?名無しさん:2009/10/25(日) 23:17:30 ID:???
>>819

不確定性原理に関する例題なのでそのように書いたのですが数学の質問にあたるだろう、という意味でしょうか?
そういうことならば数学板の方で質問すればよいのでしょうか?
821ご冗談でしょう?名無しさん:2009/10/25(日) 23:28:10 ID:???
>>820
知らんけど、古典力学の
僵〜∂K/∂v × 况
じゃないの?
822ご冗談でしょう?名無しさん:2009/10/25(日) 23:34:51 ID:qTbK8j+n
標準偏差だってわかってるなら
定義に入れて計算するだけじゃんか
Δv =√( <v^2>-<v>^2)
導出ってこれのこと??
823ご冗談でしょう?名無しさん:2009/10/25(日) 23:41:21 ID:???
>>821,822

返答ありがとうございます。
ただお二人の解法ではvの値が既知でないといけないと思うのですが
この例題でvの値は示されていないのです。
vは電子の速度なので既知という扱いではないと思います。
824592=596:2009/10/25(日) 23:44:23 ID:???
>>738
少しずつ見方が分かってきました

>>739
説明有り難う御座います
それで連結したバネの系の量子化とか、類推として有効になる訳ですね
朝永さん読んでみます

>>740
やはりそこの理解があやふやでした
>1粒子の量子力学でも、状態ベクトルそのもの と、
>それを粒子の位置演算子の固有基底で展開した成分 は、別物と考えた方がよいです。
演算子の方も、ヒルベルト空間で定義されたものと適当な基底のもとに書いたものとは別物という訳ですね

どうにか自分で勉強していけそうです
皆さん本当に有り難う御座いました
825ご冗談でしょう?名無しさん:2009/10/25(日) 23:48:24 ID:???
公式 ねじりばね定数=L/GJ
とあるのですが、Lは長さ、Gは横弾性係数です。
このJがよく分からないのですが、慣性モーメント・断面二次モーメント、断面二次極モーメントのどれでしょうか?
826ご冗談でしょう?名無しさん:2009/10/26(月) 00:02:19 ID:???
827ご冗談でしょう?名無しさん:2009/10/26(月) 00:03:47 ID:???
>>825

【機械】の質問スレッドはここだ!19
http://science6.2ch.net/test/read.cgi/kikai/1252797571/101

マルチとは…。
828ご冗談でしょう?名無しさん:2009/10/26(月) 00:34:33 ID:O0MujOvy
光は質量0なのにエネルギーがあるってきいたんですけど相対性理論のE=〜て式と矛盾しないんですか?
わかりやく教えてください。
829ご冗談でしょう?名無しさん:2009/10/26(月) 00:36:22 ID:???
着眼点はいいけど、エネルギーとはなにか、光子というキーワードを交えてもう一度調べた方がいいかもしれない
830ご冗談でしょう?名無しさん:2009/10/26(月) 00:40:07 ID:al9MEJTH
[1387]「理科系という宗教」、「アインシュタインという食わせ者のユダヤの大神官を創作した戦後物理学の
世界宗教化」というような本を出す私の意欲
投稿者:副島隆彦投稿日:2009/10/25(Sun) 06:17:23

「マイナス3(−3)」というのは、たとえば、それを実感で分かろうとすれば、それは、「私は、マイナス3円
という資産(お金)を持ってる」ということは、「私は、あなたに3円借りている」という意味ですから、
なんとか実感で分かる。しかし、それでも「マイナス3」という数量は実在しません。 

さらに、√−3(ルート・マイナス・3)という、虚数(きょすう、イマジナリー・ナンバー imaginary number 、
想像上の数字)は、この世(この世界)には、実在しません。虚数記号の i(アイ)を実数(じっすう)の後ろに
くっつけて、演算(えんざん)出来ることにして、使っているだけだ。虚数は、数学者たちが勝手に作って、
自分たちの「高級(高等)数学」論文(これらも、自分たちの作文のことだ)の道具、部品にして使っている。  

 この虚数を、実在(じつざい)である宇宙(space 、スペイス。私たちの目の前の空間から、ずっと続いている)
のことを説明するのに、ホーキングたちの宇宙論(ビッグバン理論)派は、この虚数をたくさん、自分たちの
「説明(のための)数学」の論文に多用しなければ済まないのか。
こういう、途方もない駄法螺=だぼら= をやって、今の世界の宇宙物理学というのは出来ている。
831ご冗談でしょう?名無しさん:2009/10/26(月) 00:55:35 ID:???
>>828
運動エネルギーは1/2mv~2って聞いたけど、
速度が0の物体にもエネルギーがあるのはおかしくないですか?

という質問といっしょ。
832ご冗談でしょう?名無しさん:2009/10/26(月) 00:58:26 ID:???
>>828
相対性理論の式はE^2=m^2c^4 + p^2c^2だよ。
833ご冗談でしょう?名無しさん:2009/10/26(月) 01:01:59 ID:???
834ご冗談でしょう?名無しさん:2009/10/26(月) 01:04:25 ID:???
>>832
E=mc^2じゃないんですか?
アインシュタインは間違ってた?
835ご冗談でしょう?名無しさん:2009/10/26(月) 01:09:04 ID:???
>>834
お前がな。
836ご冗談でしょう?名無しさん:2009/10/26(月) 01:12:06 ID:???
>>834
相対論の式が >>832 だと述べてあるだろ
837ご冗談でしょう?名無しさん:2009/10/26(月) 01:13:48 ID:???
アインシュタインてよく間違えてたことにされるよね(´・ω・`)
838ご冗談でしょう?名無しさん:2009/10/26(月) 01:14:33 ID:???
全国200万のしっと団が頑張ってるからな。
839ご冗談でしょう?名無しさん:2009/10/26(月) 01:14:38 ID:???
>>836
最新の理論だとそうなるってことですよね?
840ご冗談でしょう?名無しさん:2009/10/26(月) 01:15:22 ID:???
最新ってなんだ、アホが。
841ご冗談でしょう?名無しさん:2009/10/26(月) 01:21:13 ID:???
>>840
アホって言うな
>>834
よく考えてみるとアインシュタインの式はp=0の場合ですね
842ご冗談でしょう?名無しさん:2009/10/26(月) 01:25:30 ID:???
>>841
明日学校だろ?早く寝ろよ
843ご冗談でしょう?名無しさん:2009/10/26(月) 01:28:46 ID:???
あれ?でも光はm=0だからp=0の場合だとE=0になっちゃいますね
新しいほうの式を使うしかないみたいですね、納得いかないですが
844ご冗談でしょう?名無しさん:2009/10/26(月) 01:35:41 ID:O0MujOvy
828です。
みなさんのレベルが高くてついていけないくらいのバカですが、風自体にエネルギーはないけど風が吹くことによってエネルギーが発生するという様なことでいいんでしょうか?
845ご冗談でしょう?名無しさん:2009/10/26(月) 01:41:53 ID:???
自分で納得がいって、考えてる範囲でつじつまが合うのならそれでいいでそ
846ご冗談でしょう?名無しさん:2009/10/26(月) 01:45:49 ID:???
>>844
E=mc^2というのは静止エネルギー、つまり運動量が0場合の式
運動してるときは本来の形、E^2=m^2c^4 + p^2c^2を使えば矛盾してない
光の場合m=0だからE=pcという関係が出てくるでしょう
あなたの例えはなかなかいい線いってると思うよ
847ご冗談でしょう?名無しさん:2009/10/26(月) 02:52:08 ID:O0MujOvy
ありがとうございます。
逆に言えば光は静止する事ができないと考えても大丈夫ですか?
848ご冗談でしょう?名無しさん:2009/10/26(月) 03:03:44 ID:???
公式 ねじりばね定数=L/GJ
とあるのですが、Lは長さ、Gは横弾性係数です。
このJがよく分からないのですが、慣性モーメント・断面二次モーメント、断面二次極モーメントのどれでしょうか?
849ご冗談でしょう?名無しさん:2009/10/26(月) 07:20:20 ID:???
>>848
マルチ乙
850ご冗談でしょう?名無しさん:2009/10/26(月) 08:10:10 ID:al9MEJTH
佐藤勝彦(さとうかつひこ)と、池内了(うけうちさとる)という、心底、頭の悪い人間たちが、日本における
ビッグバン理論派の防衛隊の係りで、神官(あるいは、クソ坊主)たちだ。そのうち世界中で撃滅が始まる。
今のビッグバン派の宇宙論など50年後には、消えてなくなっているだろう。それは、江戸時代まで、
むずかしく荘厳(そう)な、何百もある経典(きょうてん)の、すなわち、アホダラ経のお経(きょう)を
唱えて、民衆を支配して、「ありがたい、ありがたい、お釈迦さまが説いた、真理の言葉なるぞ」と、
「この世の真理」なんかどうせ何も分からなかったくせに、長らく威張り腐って、一般大衆を騙して来た、
仏教各派の 大僧正(だいそうじょう)とか、座主(ざす)とか、門主(もんしゅ)とかの坊主どもと同じことだ。

フレッド・ホイルというケンブリッジ大学の宇宙物理学の学者が、ガモフらの大ウソつきが、「ビッグバン宇宙論」
を唱えた1956年頃と、全く同じ時から、「そんな宇宙膨張説など私は信じない。宇宙には、200億年前、
300億年前の星雲(銀河)があることが観測されている。個々の天体(星、スター、恒星)の成長(進化)と
消滅は見られるが、宇宙そのものの成長など考えられない。宇宙は、定常的に存在する」という、定常宇宙論
(ていじょううちゅうろん)の方が、ずっと、一貫して正しい。 

 今から、143億年前(?ころころ、とこの数字は変わる。おもしろい)に、宇宙の中心で、始原(しげん)の
大爆発があった、というビッグバン宇宙など、は無い。馬鹿どもの理論だ。ただの理論(セオリー)=仮説
(ハイポセティカル・セオリー)だ。これの元凶を作ったアインシュタインの相対論だって、今も、セオリーの
ままである。真偽判断の証明は出来ていない。だから、アルキメデスの原理のような原理や、定理になっていない。

数学の世界の公理(こうり、アクシアム)との整合性もつかないままだ。自分たちで、いいように勝手に数学
(数式)という道具を、勝手に作って、自分たちに都合のいい箇所だけ言いように使って、「ほら、証明できた」
と、自画自賛しているだけだ。(大)宇宙についての、証明など何も出来ていない。
851ご冗談でしょう?名無しさん:2009/10/26(月) 08:12:09 ID:al9MEJTH
ビッグバン派に、それでは、その始原の大爆発は、一体、観測できる宇宙のどの辺で起きたのか、説明してくれ、
というと、ムニャムニャと逃げる。「ビッグバンは、宇宙のどこで起きてもいいのです。数式が、空間のどこで
でも成立するのと同じことです」というような、人を喰ったようなことを言う。物理学が扱うのは、実在の世界だ、
ということの重大性をいい加減に、希薄にして逃げようとする。そうはさせるものか。
「実は、分からないのです、と言え。白状せよ」と、私は追及する。は、生命(ライフ)、物質(マター)について
の証明が、出来ていないのと同じことだ。しかし、生命や物質を扱っている連中は、まだ、少しは正直だ。
分子生物学(モレキュラー・バイオロジー)のノーベル賞独占体制はもうすぐ壊れるだろうが。これらと比較すると、
宇宙物理学の、自己欺瞞と、増長(ぞうちょう、膨脹?)ぶりは、目に余る。

 大宇宙については、人類は、まだ、何も確かなことは分かっていない。「小さな分かっていることしか(実験・
観測・観察のデータとしてしか)分かっていない」のです。あとは、自分勝手なモデル (仮説構成体)だ。
ビッグバン宇宙論という、ただの「モデル」だ。言いたい放題に過ぎない。
 人のことを、疑似科学(ぎじかがく)だ、と腐(くさ)している暇があったら、自分たちがやってきたことの
方が、もしかしたら疑似(シュードウ、プソイド)科学(サイエンス)ではないか、という恐怖に、真夜中に
ひとりで襲われるその恐怖感の方を、正直に書いたほうがいいと、私、副島隆彦は思います。
私は、しつこい人間です。やる、と決めたら、絶対にやる。何十年かかっても(もう、あと10年ぐらいで
死ぬつもりですが)やる、と言いながら、56歳まで生きてきた人間です。
852ご冗談でしょう?名無しさん:2009/10/26(月) 12:18:08 ID:???
ホイル説も所詮は『仮説』ぢゃないか(爆笑)
853ご冗談でしょう?名無しさん:2009/10/26(月) 12:19:51 ID:lDTaZyS1
ホイル説は観測事実と矛盾しないだろ。
ビッグバンは論理的矛盾で自滅してる。
854ご冗談でしょう?名無しさん:2009/10/26(月) 12:24:50 ID:???
議論の本質と直接関係のない罵倒で始まる文章を書く人間は、社会的にまず
信用するに足りない。
855ご冗談でしょう?名無しさん:2009/10/26(月) 13:34:29 ID:3GGmMaLz
ロスチャイルドの治し方は、サイコ暗示解きだよ。心理学者の専門のエキスパートはできるよ。
856ご冗談でしょう?名無しさん:2009/10/26(月) 15:00:24 ID:3GGmMaLz
ロマノフの治し方は、チコの実、クコの実を食べること。
857ご冗談でしょう?名無しさん:2009/10/26(月) 17:04:22 ID:???
電子の二重スリットの実験についての質問なんですが
空気がある状態で実験すると完全に粒子のようにふるまうのでしょうか?
858ご冗談でしょう?名無しさん:2009/10/26(月) 17:12:31 ID:???
光子の裁判
859ご冗談でしょう?名無しさん:2009/10/26(月) 22:44:59 ID:???
公式 ねじりばね定数=L/GJ
とあるのですが、Lは長さ、Gは横弾性係数です。
このJがよく分からないのですが、慣性モーメント・断面二次モーメント、断面二次極モーメントのどれでしょうか?
860ご冗談でしょう?名無しさん:2009/10/26(月) 22:46:55 ID:???
ねじりばね定数=GJ/L
Jは断面二次極モーメント
材料力学の本でも読めば?
861ご冗談でしょう?名無しさん:2009/10/26(月) 22:47:14 ID:???
862ご冗談でしょう?名無しさん:2009/10/27(火) 01:11:14 ID:???
>>857
あれは仮想実験であって、実際に実験しようとすると原子スケールの
スリットがいる(から気軽には実験できない)。電子のコンプトン波長とか
考えてみればわかるけど…。

途中で位置を観測すれば干渉模様がこわれることを、粒子的、と呼ぶなら
君の言うとおりだと思う。
863ご冗談でしょう?名無しさん:2009/10/27(火) 07:20:59 ID:???
>>862
NHK高校講座 物理
ttp://www.nhk.or.jp/kokokoza/library/2008/tv/butsuri/archive/resume020.html
光の回折と干渉 にて普通に"空気のある部屋"で
"原子スケールとはいえない"人間の目にも見える普通のスリットにて
その実験に立ち会ったおんにゃの子が、その目で、その現象をみているのは
夢まぼろしなのかい?
864ご冗談でしょう?名無しさん:2009/10/27(火) 07:26:46 ID:???
>>862
電子の2重スリット実験は実際に行われてるよ
865ご冗談でしょう?名無しさん:2009/10/27(火) 07:32:21 ID:???
>>863
節子それ電子とちゃう!電磁波や!
866衣ちゃん (咲 -Saki-):2009/10/27(火) 07:48:05 ID:???
                                             /:::〉
                                        /:::::::/
                                       '´::::::::::/
                                        /::::::::::_/
                                      /::::::厂   -‐―…――- 、___
                                   j/:::/ /:::::::::::::::::::::::::::::::::::::::::::/
                                  〈:::::{ /::::/ ̄ ̄ ̄ ̄ ̄ ̄ ̄
                                  ´>∨<`丶、        二二
                                   /   ´  ̄ ̄`   ヽ        ノ
                                 /   / -八  -\.  } \         ─
                              {八 { /、{\{ヽ、jヽ人\\        ̄厂
                                ∨ヽ{ ┃    ┃}∧ 、 \ ー―-
                                   / 人   、 ,   .ィヘ   \     \  テ
                               / /  >y‐rz‐<_八 \ \  丶  \}
                                 / / /\∨ハ∨/\丶 \ \  \   |
                              ′{厶イ トニ∧二>ト、{>\   \  ',∨
                              { 八  ∨{{    //) \\ \i\ }\|
                              \{\ /`ー'¨⌒`ー   }V\ノ  ヽ
                                  \        /
867天江衣 (咲 -Saki-):2009/10/27(火) 07:48:55 ID:???

          ,. '"  ,,. -‐ ''___`‐- .,,   ;ヽ;';,''          '';'';,,  ,, ,,
        /   /-‐''¨. . . . . . . ̄ ,,   ;;  ヽ  ..:::.  .: :. . .   ''  ''";'"
        ,.'    . . : : : : : : : : : : :.l: :,;'' :;,;''.       :  :  :  . .:  .      ;,
      /  . : . : : : : : :/://: : :;,: :l: :;''      .''::. :  .: .  . :   .     ;'
      ,' . . : : : : : : : :./:/!{: : :/:|: :l: ;'       : :. :  . .  . .  :.    ;''
       l ̄ 7 ̄/ ̄ ̄/l/! ll--{::::!: l! j'',,        : : .:  : .  :.:  :. . .  ;
       !_ ../..-‐.:7:`''lッl/,,l_|ヽ: l:::l:./:/;,: ';..       : .: :  : :  :::  :.' :  ;'
      ', : l: : :.l!:ト、ヽ|,,===ミ、`、!.j///,' '';;     .'  :.;   ...:  ::   :::  : ';,
      '、:ト、: :l:、!:.〉 {{ l:::():::!l::::::::::::::'丿.イ'';..   :  ::   ''  '     : ;'
.       ``/>‐:`ヘ:',.``ー‐''     '" / '',,    :               :::;'
       / : :./:ゝ.、      '  __, /  '';,    :            . ;'  
.     ,. ' ' . : : /: : : :;>. ._. ` ̄-‐ ´.'  ../;    :           . ;' これは光=電磁波であって
.    / . : : : /: : : : /: : : :/.>.--/ . .:/: : ',   :            ;;' 電子じゃないよ !!!
.   /  . : : : ./: : ,; -‐_‐_ v_/〉:.:/ . ,: :': : : / ';  :           ;'
../  . : : ; ':/l¨¨'´ ̄ ...../: :._/. : '': : : :; ''  / ;             ;'
´ . : : ; ' /: :l:!   ..::;  ''": : : : : :;, '}::}   /  ',            ;''\
. .: : ; ',/: : : 人,, ィ'.". : : : : : :, ''"  )::l   },ィ''"l ';          ;'\  \
: : ;'/: : : : ://: : : : : : : : /\ .   /:/. -==/::|, ┴ '''       ;; '' \:\  \
868ご冗談でしょう?名無しさん:2009/10/27(火) 08:07:35 ID:???
いや電子じゃないだろ。
869ご冗談でしょう?名無しさん:2009/10/27(火) 12:12:30 ID:Z1Lwy56t
質問させて下さい。
常識的なことだと思うのですが少し混乱しています。

水を張ったお茶碗の底に10円玉を置き、斜め上方から
見ると、10円玉から出た光が屈折してから目に到達する
ため、実際よりも浅い位置にあるように見えます。

これについて物理の教科書や参考書に書かれている図では、
人間の目から水面まで延びている光の線をそのまま水中に
向かって延長した直線(これを直線mとします)と、
10円玉から真上に向かって引いた直線(直線nとします)
との交点に10円玉があるように見えると解説されています。

ここで私が疑問に思っているのは、10円玉が直線m上に
あるように見えるというところまでは納得がいくのですが、
直線n上にある理由が分からないのです。
これは私の単純な考えなのですが、光が水面と交わる点を
中心とし、そこから実際の10円玉がある場所までの距離を
半径として円を描き、それと直線mが交わる点にあるように
見えるのではないかと考えています。

どなたかこの問題について教えて頂ける方がいらっしゃい
ましたらどうかよろしくお願いします。ここ5年ほど
疑問に思い続けております。
870ご冗談でしょう?名無しさん:2009/10/27(火) 12:46:46 ID:YS6NEW/C
もしもの話なんですけど
直径1mの電車が光速度の90%で、電車に対して静止している直径1mのトンネルをくぐる時
電車がトンネルをくぐってる時間は何秒でしょうか?

トンネルも光速度の90%で電車に向かってきた場合のくぐってる時間も教えてください。
871ご冗談でしょう?名無しさん:2009/10/27(火) 12:49:16 ID:???
>>870  トンネルの 長さも出さず しつもんか
872ご冗談でしょう?名無しさん:2009/10/27(火) 12:51:05 ID:???
>>870 電車にも 長さがいるぞ ねんのため
873ご冗談でしょう?名無しさん:2009/10/27(火) 12:51:15 ID:???
直径とかワロタ
874ご冗談でしょう?名無しさん:2009/10/27(火) 13:09:04 ID:???
>>869
よく気がつくな、今まで考えたこともなかった
どこに見えるかは2つ考えられる
(1)両眼視によるもの
(2)焦点距離によるもの
普通、距離は両眼視で測ってると思う
このときは、右目でできる直線をm1, 左目でできる直線をm2とすれば、
2直線を真上からみたとき、ちょうど10円玉のところで交わるでしょう?
(2)のほうは自分の計算だと、むしろさらに手前に見えるみたい(正しいかはわからない)
875ご冗談でしょう?名無しさん:2009/10/27(火) 13:52:38 ID:YS6NEW/C
トンネルも電車も1メートルです
876ご冗談でしょう?名無しさん:2009/10/27(火) 14:01:45 ID:???
>>875
唐突に何を言い出すのだ。
877ご冗談でしょう?名無しさん:2009/10/27(火) 14:52:47 ID:???
>>869
10円玉を水の中に入れて実験しました。
でも浮き上がったよーには見えませんでした。^^;

だけどそーですよね。
幾何光学で、レンズなんかでは、どこにどんな風に見えるか?は
中学校でも散々作図しますよね。
見る人の目がどこにあるか? ですよね。
水の屈折ではイー加減にやってますけど、
水の屈折だってやっぱり同じですよね。

教科書作者に文句を言いましょう。
878ご冗談でしょう?名無しさん:2009/10/27(火) 15:05:43 ID:???
>>869
光の屈折に関して根本的な勘違いをしてる。
879869:2009/10/27(火) 15:55:52 ID:???
>>874
ご回答ありがとうございます。
私もひょっとすると人間は両目で物を見るという点が
この問題に関わってくるのかもしれないと思ったの
ですが、仰る通り確かに(1)の考え方に従って真上から
見た図を書き、10円玉が手前に見えるのか奥に見えるのか
という2次元的な情報についてだけ考えると、直線m1、
直線m2、そして人間の両目を結んだ三角形ができ、
本来の10円玉と同じ位置に見えることになりますね。
そうすると、教科書や参考書に書かれている位置
(本来の10円玉より浅い位置で、手前や奥ではなく
真上の位置)に見えるということになりますね。
ああ、なるほど。ここ5年の中ではかなり納得が
行く説明を聞いた気がします。

>>877
実際に実験されたんですか。どうもありがとうございます。
浮き上がったように見えませんでしたか?見えるか見えない
かという点に関して言えば、私自身は経験則的に水中のものは
浅く見える(特に川などで)ということは間違いないと思って
おりますが、小さな茶碗に入れた10円玉などではあまり大きな
影響がないのかもしれませんね。

>>878
もしよろしければ少し解説して頂けませんでしょうか。
この問題についての答を知りたいので、何か参考意見が
あればどんなものでも伺いたいと思っております。
880ご冗談でしょう?名無しさん:2009/10/27(火) 16:09:11 ID:???
10円玉いれるのは汚いし、どんぶりと箸でやるのがお勧め
斜めにした箸は折れて見えるし、
真上に〜というのも、水に入れた一本の箸の先端にもう一本の箸の先を
上からぶつけようとしてみると、真上に見えてることが実感できると思う
881ご冗談でしょう?名無しさん:2009/10/27(火) 16:18:46 ID:???
>>879
紙に矢印を描いてその上に厚いガラスを乗せて、
で両端から目に来る光を作図し、
それから直線だとして投影すればこれはいいんですね。
投影された矢印は垂直には上がらず、遠くにズレて浮き上がりますよね。

でも教科書作者に文句を言うときには、水の屈折率で作図しましょう^^;
882ご冗談でしょう?名無しさん:2009/10/27(火) 16:42:05 ID:???
ちゃんとした教科書だと「上に浮き上がって見える」というのはあくまで
「真上に近い位置から見下ろしたという極限での計算」だと明記してある
んだけどね。
883ご冗談でしょう?名無しさん:2009/10/27(火) 16:46:00 ID:???
これ実際にはどうなの?面白い問題じゃない?
両眼視だと真上、幾何光学で手前
やってみると真上に見えるけど
884ご冗談でしょう?名無しさん:2009/10/27(火) 16:53:07 ID:???
>>883
> 幾何光学で手前
どうして?
885ご冗談でしょう?名無しさん:2009/10/27(火) 18:35:35 ID:???
886ご冗談でしょう?名無しさん:2009/10/27(火) 18:50:34 ID:???
>>885
両目を上下に並べて見ると手前に見えるってことかなあ
887ご冗談でしょう?名無しさん:2009/10/28(水) 01:16:24 ID:???
量子論に於ける波動関数が連続であるという条件は、どういう物理的な要請から現れているのでしょうか。
それとも基礎方程式としてSchroedinger方程式を仮定した時点で付随してくるものなのでしょうか。
ご教示おねがいします。
888ご冗談でしょう?名無しさん:2009/10/28(水) 01:55:40 ID:egwuMjkD
なぜ質量があると空間はゆがむのですか?
889ご冗談でしょう?名無しさん:2009/10/28(水) 02:19:10 ID:???
>>887
シュレディンガー方程式は、空間変数についての2階微分を含んでいるから、
波動関数には(弱い意味で)2階微分が存在しないといけない。

だから、大雑把にいうと、波動関数の一階微分が連続になる。
もちろん、波動関数自体は連続。
890ご冗談でしょう?名無しさん:2009/10/28(水) 02:23:24 ID:???
>>888
宇宙はそういう風にできてるらしい、としか言いようがない。
891ご冗談でしょう?名無しさん:2009/10/28(水) 08:33:08 ID:???
君達馬鹿?考え方が逆だっての。だから質量があると何で空間が歪むの?とか時間って何?とか重力は何であるの?とか意味不明な問いがでてくる。
892ご冗談でしょう?名無しさん:2009/10/28(水) 10:10:22 ID:???
>>891
で、どういう考え方なら正しいの?
俺は>>890同様「そういう風にできているらしいとしか言いようがない」と
思うけど。
893ご冗談でしょう?名無しさん:2009/10/28(水) 10:13:33 ID:???
891に問うても時間の無駄。
イチャモンは付けるが、自分の意見は言わない典型的な知ったかクン。
894ご冗談でしょう?名無しさん:2009/10/28(水) 11:05:28 ID:qww+2h3W
ホースの摩擦損失の単位は kg/cuで表しますが、なぜですか?
摩擦による損失が圧力の単位で表すことが理解できません…
ジュールならまだしも、なぜ、平方センチメートルに対しての圧力なのでしょうか?

どなたか回答をお願いします!
895ご冗談でしょう?名無しさん:2009/10/28(水) 11:21:52 ID:???
>>894
摩擦損失の定義ってナニ?
「入り口にかけた圧力」と「出口での圧力」の差なら、圧力が単位で当たり前
だけど。
896ご冗談でしょう?名無しさん:2009/10/28(水) 11:34:48 ID:???
「摩擦損失 定義」でぐぐったら、まさに「入口と出口の圧力の差」と出てきたが。
897ご冗談でしょう?名無しさん:2009/10/28(水) 11:38:14 ID:89QSe00n
>>891
空間がゆがむから質量ができる?
898ご冗談でしょう?名無しさん:2009/10/28(水) 11:48:22 ID:???
卵が先か ニワトリが先か の議論になりそうだからこのネタは以降却下
899ご冗談でしょう?名無しさん:2009/10/28(水) 11:51:58 ID:???
卵 or ニワトリの問題は「卵が先」との結論がついている
900ご冗談でしょう?名無しさん:2009/10/28(水) 12:23:30 ID:???
空間の歪みと質量、どっちが先かと言えば「同時」だろ。電荷と電場とか、
電流と磁場とかと一緒。
901ご冗談でしょう?名無しさん:2009/10/28(水) 12:25:22 ID:???
という点で、鶏と卵とは全然問題の質が違う。
902ご冗談でしょう?名無しさん:2009/10/28(水) 12:34:52 ID:???
>>899
オマエ親子丼を食ったことないのか? 卵は後ででとじるんだよ
つーか、およその卵が入る料理では、卵は最後に入れるんだ アホ
903ご冗談でしょう?名無しさん:2009/10/28(水) 12:43:13 ID:???
>>902
めちゃめちゃ寒い
904ご冗談でしょう?名無しさん:2009/10/28(水) 12:44:34 ID:???
バカは取りそろえていると思ったが、下には限りがないな。
905ご冗談でしょう?名無しさん:2009/10/28(水) 12:54:01 ID:???
ニワトリが産んだ卵を
ニワトリの卵とするか
ニワトリが生まれる卵を
ニワトリの卵とするかで
変わるだよね
906ご冗談でしょう?名無しさん:2009/10/28(水) 12:55:56 ID:???
>>905
座布団3枚
907ご冗談でしょう?名無しさん:2009/10/28(水) 12:57:29 ID:???
>>905
どちらが先かの話しでニワトリが生まれる卵以外をニワトリの卵とする奴がいたら異常だよ。
908ご冗談でしょう?名無しさん:2009/10/28(水) 13:10:17 ID:???
え、なんで?
909ご冗談でしょう?名無しさん:2009/10/28(水) 13:10:54 ID:???
ニワトリが産んだ卵、は誰が見てもニワトリの卵
ニワトリが生まれる卵は、しかし、
誰もそれをニワトリが生まれる卵だとは言い切れない

ニワトリ無くして卵は無い
910ご冗談でしょう?名無しさん:2009/10/28(水) 13:16:17 ID:???
アヒルが卵胎生でニワトリを生んだら、しかしそれはニワトリだ
卵から孵ったアヒルは、それがニワトリが産んだ卵だってもアヒルだ

ニワトリが先だ
911ご冗談でしょう?名無しさん:2009/10/28(水) 13:21:18 ID:???

おまいら、生物板にでも逝け
912ご冗談でしょう?名無しさん:2009/10/28(水) 13:28:43 ID:???
空間が歪むから質量ができる? となると
じゃあ誰が最初に(どこかの神様 ???)空間を歪めたの?の議論になる
913ご冗談でしょう?名無しさん:2009/10/28(水) 13:34:44 ID:???
神様が最初に空間を歪めたもうた

よって、ガスが集まって質量ができた

さらに卵を お造りたもうた

ついでに自然数も お造りたもうた

↓・・・ウン百億年後

今現在に至る
914ご冗談でしょう?名無しさん:2009/10/28(水) 13:38:39 ID:???
途中、神様が気まぐれに(どこから ???)巨大隕石を地球に投げ込んだ

大量絶滅、恐竜時代の終焉
915ご冗談でしょう?名無しさん:2009/10/28(水) 13:42:06 ID:???
観鈴ちんは、ヒヨコが大きくなったら恐竜さんになると思ってました
916ご冗談でしょう?名無しさん:2009/10/28(水) 13:44:02 ID:???
が、がお…
917ご冗談でしょう?名無しさん:2009/10/28(水) 13:46:46 ID:???
>>912
その話はもう>>900で終わっている。
918ご冗談でしょう?名無しさん:2009/10/28(水) 13:47:32 ID:???
900越えると更にもましてgdgdになるのがいつもの例w
919ご冗談でしょう?名無しさん:2009/10/28(水) 13:49:18 ID:???
>>915-916
ごめん、元ネタ分からん
920ご冗談でしょう?名無しさん:2009/10/28(水) 13:50:41 ID:???
ボクはミドリガメは大きくなったらガメラになるからと、
今も大切に育てています。
921ご冗談でしょう?名無しさん:2009/10/28(水) 13:52:01 ID:???
>>920
ごめん、元ネタ分からん
922ご冗談でしょう?名無しさん:2009/10/28(水) 13:56:10 ID:???
それは報告しなくていいです
923ご冗談でしょう?名無しさん:2009/10/28(水) 14:01:04 ID:???
さよか
924ご冗談でしょう?名無しさん:2009/10/28(水) 14:12:36 ID:???
数学なのか物理のどっちかなのか迷ったんですが、ここで質問します
正面、真上、真横の三方向からまん丸に見える物体があります
これはまん丸な球体以外ありえませんか?
925ご冗談でしょう?名無しさん:2009/10/28(水) 14:15:51 ID:???
>>924
円板を3つ互いに直角に組み合わせた物体とか、いろんな可能性があるな。
926ご冗談でしょう?名無しさん:2009/10/28(水) 14:17:23 ID:???
球以外でも可能。
名前は付いてないがね。
927ご冗談でしょう?名無しさん:2009/10/28(水) 14:18:26 ID:???
>>924
互いに垂直な円柱が交差する部分の立体とか。
大学入試とかでその体積を求めろとかあるじゃん。
928ご冗談でしょう?名無しさん:2009/10/28(水) 15:08:29 ID:2zZPZxIb
暗黒物質の仮説を数式で示してください
929ご冗談でしょう?名無しさん:2009/10/28(水) 16:49:12 ID:???
その仮説に関して、私は真に驚くべき数式を見つけたが
この余白はそれを書くには狭すぎる

by 萌える・ド・ブルマー
930ご冗談でしょう?名無しさん:2009/10/28(水) 17:30:03 ID:89QSe00n
>>929
フェルマー?
931ご冗談でしょう?名無しさん:2009/10/28(水) 18:24:08 ID:???
どう見てもピエール・ド・フェルマーをもじった名前なわけだが
932ご冗談でしょう?名無しさん:2009/10/28(水) 18:28:37 ID:???
12 名前: 132人目の素数さん 投稿日: 02/04/15 12:12

「ブルマーの定理」なら、

[1] 小野敏洋,「太陽の少女インカちゃん」,第1巻,電撃comics(EX),
メディアワークス,主婦の友社,\880,1996

という文献(てゆーかコミック本ですが^^;)に詳しく紹介されてい
ますが…。(笑)
 たしか裏表紙にまで「証明」の一部が書いてあったような記憶が。
933ご冗談でしょう?名無しさん:2009/10/28(水) 18:52:37 ID:GpwJ5LKw
地球は自転しながら太陽の周りを公転しています。
そして太陽系は銀河の中心を軸に周り、その銀河も移動しています。
ならば、銀河の外のある座標から我々を見たとき、いったいどのくらいの速さで動いているのでしょうか?
当方文系なもので難しいことは分かりかねますが、御回答宜しくお願いします。
934三流理系:2009/10/28(水) 19:21:05 ID:???
銀河の外のある座標とは?そんなの任意では?
アンドロメダ星雲M31とは時速約50万kmで近づいてきており、
30億年後くらいには衝突するらしいです。アンドロメダ星雲含む43個の銀河
は局部局部銀河群を構成しているそうです。43個の銀河の重心座標系とれば
いかも?詳しくは以下のサイト。
http://astro.ysc.go.jp/andromeda.html
935ご冗談でしょう?名無しさん:2009/10/28(水) 19:29:27 ID:???
銀河の外のある座標から「我々」ってどれ?
地球?太陽?天の川銀河?
どっち?

そうとうなスピードで動いているそうだ NHKでやってた

ナン億年後かはしらないが、お隣のアンドロメダ銀河と ぶつかるらしい
936ご冗談でしょう?名無しさん:2009/10/28(水) 19:36:53 ID:???
まあぶつかったって、恒星と恒星の間隔は著しく遠いから、
互いに重力で相互作用しながらすり抜けてしまうのがほとんどで、
恒星同士衝突するのは極めてまれな現象。

星間ガス同士の衝突の方が見ものになるだろう。
937ご冗談でしょう?名無しさん:2009/10/28(水) 19:39:55 ID:???
銀河同士の衝突はままあるそうだ
観測されている"変な"形の銀河はまさに衝突中だとか
ただその状況が地球に光として届き観測されるまでウン万年かかるがな
938ご冗談でしょう?名無しさん:2009/10/28(水) 19:41:24 ID:???
天文・気象板でやれ
939ご冗談でしょう?名無しさん:2009/10/28(水) 19:47:08 ID:???
推測の域だけど

私たちの天の川銀河も
実は元々小さな銀河だったのかもしれない

過去に何度か周り(?)の銀河たちと衝突し合って大きくなって
今のようなものになったのかと・・・

そして今度は(比較的近い)お隣のアンドロメダ銀河に迫ろうとしている
940ご冗談でしょう?名無しさん:2009/10/28(水) 19:51:45 ID:???
食いしん坊だな
941ご冗談でしょう?名無しさん:2009/10/28(水) 19:53:21 ID:???
>>936
それは言い切れない。車輪銀河の例がある。
天の川銀河も車輪銀河のように綺麗に壊れるかも知れない。
ちなみに、アンドロメダ銀河の中心には核が二つあり、
そのうちのひとつは遠い昔にアンドロメダに飲み込まれた銀河だ、
とゆー説もあるそーだ。
942887:2009/10/28(水) 19:54:13 ID:???
>>889
ありがとうございます。
1階微分の連続性はまだちょっと微妙なところなので、もう少し考えてみます。
943ご冗談でしょう?名無しさん:2009/10/28(水) 19:58:34 ID:???
>>941
は?なに寝言言ってるの?
重力相互作用は否定してないんだけど。
恒星同士の衝突なんて滅多に起きんと言ってんの。
944ご冗談でしょう?名無しさん:2009/10/28(水) 20:02:03 ID:???
>>943
とりあえず「車輪銀河」を検索すること。
945ご冗談でしょう?名無しさん:2009/10/28(水) 20:08:44 ID:???
検索するとwikiにわざわざ
>ただし、銀河を構成する恒星の密度は非常に小さいため、銀河同士が衝突しても星の衝突が起こることはめったにない。
と書いてあるな
946ご冗談でしょう?名無しさん:2009/10/28(水) 20:11:42 ID:???
wikiはだめだ(笑)

先ず車輪銀河の写真を見て欲しいんだわさ。
947ご冗談でしょう?名無しさん:2009/10/28(水) 20:16:39 ID:egwuMjkD
矛盾とパラドクスの違いはなんですか?
948ご冗談でしょう?名無しさん:2009/10/28(水) 20:19:45 ID:???
同じ。でもパラドクスの方は、一見矛盾してるようだが矛盾ではない、という意味が殆ど。
949ご冗談でしょう?名無しさん:2009/10/28(水) 20:25:33 ID:???
同じ。
でもパラドクスの方は、
一見矛盾してるようだが矛盾ではないと、
子供騙しの解説をして利口ぶってる気になるバカな人が殆ど。(笑)
950ご冗談でしょう?名無しさん:2009/10/28(水) 20:45:48 ID:???
>>946
恒星同士の衝突がわかるほどの解像度の車輪銀河の写真はどこにあるの?
951ご冗談でしょう?名無しさん:2009/10/28(水) 20:50:56 ID:???
>>946
車輪銀河くらい自分でシミュレーションしたこともあるくらい知ってるよ。
キモはなんなの?
952ご冗談でしょう?名無しさん:2009/10/28(水) 20:52:21 ID:???
もう銀河が突き抜けちゃったあとだもん(笑)
でもNASAあたりのHPなら
ハッブルで撮ったかなり解像度の高い写真があると思うけど。
だけどどっちみち、
個々の星にまで分解できた写真は無いだろうけどね。
953ご冗談でしょう?名無しさん:2009/10/28(水) 20:55:36 ID:???
>>951
銀河衝突のシミュレーションなんて、
個人所有レベルのパソコンじゃとてもとてもでできないよ。(笑)
954ご冗談でしょう?名無しさん:2009/10/28(水) 21:01:17 ID:???
xlock -mode galaxy っと
955ご冗談でしょう?名無しさん:2009/10/28(水) 21:05:08 ID:???
>>953
スパコンでも無理だけどね
956ご冗談でしょう?名無しさん:2009/10/28(水) 21:08:22 ID:???
>>946
wikiだからってだけで受け入れないのは、wikiだからってだけで受け入れるのと同程度にマヌケ

国立科学博物館 宇宙の質問箱 銀河編から
http://www.kahaku.go.jp/exhibitions/vm/resource/tenmon/space/galaxy/galaxy06.html
>星と星のあいだはひじょうに離れているので、銀河どうしが衝突しても、おたがいの銀河の星どうしがぶつかることはありませんが、

いい加減に「間違ってましたごめんなさい」すればいいのに
957ご冗談でしょう?名無しさん:2009/10/28(水) 21:10:05 ID:???
スパコン持ってないから分からない。(笑)
958ご冗談でしょう?名無しさん:2009/10/28(水) 21:13:00 ID:???
それ以前にWikipediaをナレッジデータベースを構築するIT技術の一手法である
"wiki"と略すのが頭悪そうに思えるのだけど。
959933:2009/10/28(水) 21:41:15 ID:b68pLbZ4
説明不足ですいません。ウィキの言葉を借りると、
「宇宙空間における物体の絶対速度」というのが知りたかったわけです。
天の川銀河を見渡せるような某静止したポイントから「日本」を見渡した場合、
どのくらいのスピードで動いているのだろうと。
特殊相対性理論によるとこの考え方に意味は無いようですが・・・

日本の自転速度は1400km/h
地球の公転速度は10万km/h、秒速=30km/s
銀河系は約600km/sの速度で宇宙空間を運動している。
我々(銀河)は1日に5184万km移動しており、1年では189億km動くことになる。

とのことです。
日本に住んでいる我々はどのくらいのスピードで動いているか、
計算方法の見当も付きません。

960ご冗談でしょう?名無しさん:2009/10/28(水) 21:43:41 ID:???
ソーイングマシンをマシン(ミシン)と略すのも頭悪そう?
961933:2009/10/28(水) 21:47:29 ID:b68pLbZ4
太陽系の移動速度が抜けてました。何度も申し訳ないです。

陽系の軌道速度は217km/sで、約1,400年で1光年、8日で1天文単位進む。

らしいです。
これって数学の守備範囲なんですかね?
962ご冗談でしょう?名無しさん:2009/10/28(水) 21:47:54 ID:???
> 「宇宙空間における物体の絶対速度」というのが知りたかったわけです。
> 天の川銀河を見渡せるような某静止したポイントから「日本」を見渡した場合、
> どのくらいのスピードで動いているのだろうと。

音読68回したら、もう一度質問を書いてね(はぁと
963ご冗談でしょう?名無しさん:2009/10/28(水) 21:57:25 ID:2zZPZxIb
>>955
フツーにディスカバリーチャンネルでシミュレートしてましたね
964ご冗談でしょう?名無しさん:2009/10/28(水) 22:02:51 ID:???
>>959
> 特殊相対性理論によるとこの考え方に意味は無いようですが・・・
ガリレイの頃から意味が無い
965ご冗談でしょう?名無しさん:2009/10/28(水) 22:03:31 ID:???
>>959
「静止した某ポイント」なんてものはないっ。

自分でも「相対性理論によれば」とか書いていて、なんでそんな「無いもん買い」
をしようとするんだ。
966ご冗談でしょう?名無しさん:2009/10/28(水) 22:23:35 ID:???
>>963
恒星の数の桁を大きく端折ってるのさ。

10^11個の質点の相互作用とか無理だから。
演算量はその自乗のオーダーだし。
967ご冗談でしょう?名無しさん:2009/10/28(水) 22:56:41 ID:???
>>959
>「宇宙空間における物体の絶対速度」

と言うものは存在しえませんが、あなたの考えに近いものに「宇宙背景放射に対する速度」というものが観測上得られています。
COBEによる観測によると、(地球から観測した)背景放射には相対速度700km/sの双極成分(ドップラーシフト)がみられるとの事。
968ご冗談でしょう?名無しさん:2009/10/29(木) 00:39:18 ID:erBOf1PF
日光を半透明の色紙にすかすと、色の付いた影ができますよね。
どうして色がついた影ができるのですか?
969ご冗談でしょう?名無しさん:2009/10/29(木) 00:57:32 ID:???
例えば超銀河団の中心との相対速度でいいのではないの
970ご冗談でしょう?名無しさん:2009/10/29(木) 01:00:03 ID:???
ニワトリがゲシュタルト崩壊中
971ご冗談でしょう?名無しさん:2009/10/29(木) 02:25:15 ID:???
銀河同士の衝突シミュレート動画
http://www.youtube.com/watch?v=ZTWsfPHcVHg
972DCCLXXVI ◆E5JgEsWY1w :2009/10/29(木) 18:35:18 ID:???
>>966
N個の質点ならNlogNのオーダーだが。
973ご冗談でしょう?名無しさん:2009/10/29(木) 19:34:08 ID:???
>>968
俺でも答えられるようなカス質問すんな
974ご冗談でしょう?名無しさん:2009/10/29(木) 20:00:00 ID:???
>>972
くわしく
975ご冗談でしょう?名無しさん:2009/10/29(木) 20:06:20 ID:???
空間にはもともと広がる性質がありますか?
976ご冗談でしょう?名無しさん:2009/10/30(金) 01:08:19 ID:???
>>975
ない。
977DCCLXYIV ◆E5JgEsWY1w :2009/10/30(金) 08:23:47 ID:???
相互作用の数はN^2のオーダーだが、
アルゴリズムや、相互作用の近似などでNlogNのオーダーまで計算回数を減らせる。
詳しくは分子動力学の教科書。
978ご冗談でしょう?名無しさん:2009/10/30(金) 09:47:54 ID:HWm03QHt
ある半導体のエネルギーギャップが1eVだとします。
温度に換算すると約1万Kです。
こんなにも差があるのに室温で半導体は熱的励起されるんですか?
979ご冗談でしょう?名無しさん:2009/10/30(金) 13:37:16 ID:WnuATHUN
電子が無数に同時に存在してるならこの宇宙は1つの電子でできているということでしょうか?
980ご冗談でしょう?名無しさん:2009/10/30(金) 13:41:00 ID:K18+w37f
常に今しか存在しないのに、自分自身や身の回りの物が過去、現在、未来と存在を保ってられるのは何故ですか?何か空間のDNA的なものが存在するのでしょうか?高卒ライン工勤務の俺に教えて下さい。
981ご冗談でしょう?名無しさん:2009/10/30(金) 13:51:25 ID:ffmejiJA
>>979
そうだよ。
982ご冗談でしょう?名無しさん:2009/10/30(金) 13:52:21 ID:ffmejiJA
>>980
神を信じなさい。さすれば、全ての謎は氷解する。
983ご冗談でしょう?名無しさん:2009/10/30(金) 15:26:18 ID:HWm03QHt
KケルビンってMKS単位系で表すとどうなりますか?
984ご冗談でしょう?名無しさん:2009/10/30(金) 15:38:10 ID:???
>>983
K (ケルビン)はエネルギーだから、しいて言えば J (ジュール)かな。

1 [K] = a [J]

aは水の3重点とか使って便利なように決めてるね。
985ご冗談でしょう?名無しさん:2009/10/30(金) 15:44:36 ID:???
>>978
約1eVは不純物を含まない真性半導体の
伝導帯と価電子帯のギャップであって

不純物を含んだ半導体の、ドナー準位とかアクセプター準位とか
とのギャップは、50〜100mVくらいなんじゃないの?
986ご冗談でしょう?名無しさん:2009/10/30(金) 16:03:27 ID:HWm03QHt
>>985
真性半導体は1万Kにならないと熱だけでは励起できないということですか?
987ご冗談でしょう?名無しさん:2009/10/30(金) 16:12:48 ID:???
>>984
ボルツマン定数かけないでそんなことしていいの?
988ご冗談でしょう?名無しさん:2009/10/30(金) 18:52:47 ID:uo8LaAMO
>>987
landauの単位系じゃないの?
989ご冗談でしょう?名無しさん:2009/10/30(金) 19:19:35 ID:???
刃は、力がせまい一点に集中するから切れる、と聞きました。
しかし、カッターなどを上から押しつけるだけではほとんど切れません。
「引かないと切れない」のはなぜなのですか??
990ご冗談でしょう?名無しさん:2009/10/30(金) 20:49:16 ID:??? BE:605894988-DIA(353535)
次スレ立てました
■ちょっとした疑問や質問はここに書いてね119■
http://science6.2ch.net/test/read.cgi/sci/1256903255/
991ご冗談でしょう?名無しさん:2009/10/30(金) 21:12:45 ID:???
>>989
引くことで刃先角度が実質的に小さくなる
992ご冗談でしょう?名無しさん:2009/10/30(金) 21:20:19 ID:???
>>989
刃は細かいノコギリ状になっているという要因もあるらしい
993ご冗談でしょう?名無しさん:2009/10/30(金) 23:33:49 ID:HWm03QHt
p軌道に2N個電子がはいったら状態数は6N個である
と教科書にありました?
状態数が電子の数を越えるなんてあり得るんですか?
994ご冗談でしょう?名無しさん:2009/10/31(土) 00:27:06 ID:???
>>984
K(ケルビン)は温度単位、MKS単位系でもKのまま、CGS単位系では℃だったと思う。
995ご冗談でしょう?名無しさん:2009/10/31(土) 00:36:34 ID:???
>>994
SI単位系ではケルビンは基本単位の一つだけど、MKSやCGS単位系に温度の単位があるの?
996ご冗談でしょう?名無しさん:2009/10/31(土) 01:04:17 ID:9nEE6LGc
あるに決まってるだろ。
997ご冗談でしょう?名無しさん:2009/10/31(土) 01:36:26 ID:???
http://www.wdic.org/w/SCI/SI単位系

SI単位系はMKSA単位系を発展させたもので、MKSA単位系から作ることができない温度(K)、物質量(mol)、光度(cd)、角度(rad)、立体角(sr)を追加して拡張したものが基本となっている。
998ご冗談でしょう?名無しさん:2009/10/31(土) 07:14:18 ID:???
ブルーレイに使う物質を、フォトジェニックにすると、高次物質、超電導物質(三菱のものより劣るが)、面白い特性を持つよ。クリスタルエネジー)が作れるよ。EUの知識だが。 
特許

上記でできた物質を、マテリアルに変換すると、高次マテリアルができるよ。
特許

アメリカには、もうジュエル(工業専門用語だが)があるよ。スーパートップシークレットだが。
999ご冗談でしょう?名無しさん:2009/10/31(土) 07:16:43 ID:Ozz2xU23
パソコンに付いているバンク(銀行)機能を、逆にしてみれば、放出系の機能ができるよ。
特許

フォトジェニック以外でも、試すと面白いものができるよ。
1000ご冗談でしょう?名無しさん:2009/10/31(土) 07:17:16 ID:9nEE6LGc
ウンコをするのも面白い。
10011001
このスレッドは1000を超えました。
もう書けないので、新しいスレッドを立ててくださいです。。。